Hostname: page-component-76fb5796d-vvkck Total loading time: 0 Render date: 2024-04-27T10:49:02.901Z Has data issue: false hasContentIssue false

ON COHEN AND PRIKRY FORCING NOTIONS

Part of: Set theory

Published online by Cambridge University Press:  11 September 2023

TOM BENHAMOU*
Affiliation:
SCHOOL OF MATHEMATICAL SCIENCES RAYMOND AND BEVERLY SACKLER FACULTY OF EXACT SCIENCE, TEL-AVIV UNIVERSITY RAMAT AVIV 69978 ISRAEL E-mail: gitik@tauex.tau.ac.il
MOTI GITIK
Affiliation:
SCHOOL OF MATHEMATICAL SCIENCES RAYMOND AND BEVERLY SACKLER FACULTY OF EXACT SCIENCE, TEL-AVIV UNIVERSITY RAMAT AVIV 69978 ISRAEL E-mail: gitik@tauex.tau.ac.il
Rights & Permissions [Opens in a new window]

Abstract

  1. (1) We show that it is possible to add $\kappa ^+$-Cohen subsets to $\kappa $ with a Prikry forcing over $\kappa $. This answers a question from [9].

  2. (2) A strengthening of non-Galvin property is introduced. It is shown to be consistent using a single measurable cardinal which improves a previous result by S. Garti, S. Shelah, and the first author [5].

  3. (3) A situation with Extender-based Prikry forcings is examined. This relates to a question of H. Woodin.

Type
Article
Creative Commons
Creative Common License - CCCreative Common License - BYCreative Common License - NCCreative Common License - SA
This is an Open Access article, distributed under the terms of the Creative Commons Attribution-NonCommercial-ShareAlike licence (https://creativecommons.org/licenses/by-nc-sa/4.0/), which permits non-commercial re-use, distribution, and reproduction in any medium, provided the same Creative Commons licence is included and the original work is properly cited. The written permission of Cambridge University Press must be obtained for commercial re-use.
Copyright
© The Author(s), 2023. Published by Cambridge University Press on behalf of The Association for Symbolic Logic

0 Introduction

0.1 Intermediate models of the tree-Prikry forcing

In many mathematical theories, such as groups, vector spaces, topological spaces, and graphs, the study of submodels of a given model is indispensable to the understanding of the model and in some sense measures its complexity. In forcing theory, subforcings of a given forcing generate intermediate models to a generic extension by the forcing. Hence, the study of intermediate models is somehow parallel to the one regarding subforcings. There are numerous classification results in this spirit, for example, some forcing such as the Sacks forcing [Reference Sacks34] and variants of the tree-Prikry forcing [Reference Koepke, Rasch and Schlicht25] do not have proper intermediate models. Other forcings such as the Cohen forcing [Reference Kanamori24], Random forcing [Reference Maharam27], Prikry forcing [Reference Gitik, Kanovei and Koepke20], and Magidor forcing [Reference Benhamou and Gitik6, Reference Benhamou and Gitik8] have intermediate models of the same type. A tree-Prikry forcing or its particular case, which will be central for us in this paper, the Prikry forcing with a non-normal ultrafilter can behave differently. For example, under suitable large cardinal assumptions, every $\kappa $ -distributive forcing of cardinality $\kappa $ is a projection of this forcing. Actually, more is true, under the assumption that $\kappa $ is $\kappa $ -compact there is a single Prikry-type forcing which absorbs all the $\kappa $ -distributive forcings of cardinality $\kappa $ (see [Reference Gitik19]). In the absence of very large cardinals, the situation changes; indeed, Hayut and the authors [Reference Benhamou, Gitik and Hayut9] proved that if a certain $<\kappa $ -strategically closed forcing of cardinality $\kappa $ is a projection of the tree-Prikry forcing then it is consistent that there is a cardinal $\lambda $ with high Mitchell order, namely $o(\lambda )>\lambda ^+$ . In [Reference Benhamou and Gitik6], the authors proved that starting from a measurable cardinal (which is the minimal large cardinal assumption in the context of Prikry forcing) it is consistent that there is a (non-normal) ultrafilter U, such that the Prikry forcing with U projects onto the Cohen forcing $\operatorname {\mathrm {Cohen}}(\kappa ,1)$ ; this was improved later in [Reference Benhamou, Gitik and Hayut9] to a larger class of forcing notions called Masterable forcings. In the context of Prikry-type forcings, the existence of such embeddings and projections allows one to iterate distributive forcing notions on different cardinals (see [Reference Gitik, Foreman and Kanamori17, Section 6.4]).

It remained open whether it is possible to get more Cohen subsets of $\kappa $ after forcing with the Prikry forcing with a $\kappa $ -complete ultrafilter U over $\kappa $ . This was asked explicitly in [Reference Benhamou, Gitik and Hayut9].

The basic difficulty is that the size of $\operatorname {\mathrm {Cohen}}(\kappa ,\kappa ^+)$ is $\kappa ^+$ and it is not hard to see (Proposition 2.9) that this cannot happen, if U has the Galvin property.

We formulate a certain strengthening of the negation of the Galvin property, show its consistency starting with a measurable cardinal, and finally apply it in order to construct an ultrafilter U such that the Prikry forcing (for a formal definition of the Prikry forcing with non-normal ultrafilter, see Definition 1.2) with it adds a generic subset to $\operatorname {\mathrm {Cohen}}(\kappa ,\kappa ^+)$ .

0.2 Extender-based Prikry forcing and a question of Woodin

Magidor and the second author developed the Extender-based Prikry forcing in [Reference Gitik and Magidor21] to violate the SCH under mild large cardinal assumptions. Later Merimovich [Reference Merimovich29, Reference Merimovich30] presented a variation of this forcing which will be used in this paper.

H. Woodin askedFootnote 1 in the early $90$ s whether, assuming that there is no inner model with a strong cardinal, it is possible to have a model M in which $2^{\aleph _\omega }\geq \aleph _{\omega +3}$ , GCH holds below $\aleph _\omega $ , there is an inner model N such that $\kappa =(\aleph _\omega )^M$ is a measurable and $2^{\kappa }\geq (\aleph _{\omega +3})^M$ .

A natural approach to tackle Woodin’s question is to use the Extender-based Prikry with interleaved collapses forcing, defined by the second author and Magidor in [Reference Gitik and Magidor21]. This forcing collapses a measurable cardinal to $\aleph _\omega $ and simultaneously blows up the powerset of that measurable. Hence, if one can show that a generic extension by the Extender-based Prikry forcing has an intermediate model where $\kappa $ stays measurable and $2^{\kappa }$ is large, this will provide a positive answer to Woodin’s question. In this paper we show that this approach is doomed. More precisely, we address in general the question whether it is possible to add many subsets of $\kappa \ {\langle } x_\alpha \mid \alpha <\lambda {\rangle }, \ \lambda \geq \kappa ^{++}$ with the Extender-based Prikry forcing over $ \kappa $ such that $\kappa $ remains a regular cardinal in $V[ {\langle } x_\alpha \mid \alpha <\lambda {\rangle }]$ . We give a negative answer to this question with respect to the Extender-based Prikry forcing as defined in [Reference Gitik and Magidor21] and the Merimovich version of the forcing presented in [Reference Merimovich30, Reference Merimovich31]. In particular, as a consequence of our results (Theorems 4.5 and 4.6), the Extender-based Prikry forcing cannot be used to answer Woodin’s question.

0.3 The Galvin property

F. Galvin [Reference Baumgartner, Ha̧jņal and Mate2], in the 70s, showed that if $\kappa ^{<\kappa }=\kappa $ and F is a normal filter over $\kappa $ then the following combinatorial property holds:

$$ \begin{align*}\text{ For every }\{X_i\mid i<\kappa^+\}\subseteq F\text{ there is }I\subseteq [\kappa^+]^{\kappa}\text{ such that } \cap_{i\in I}X_i\in F.\end{align*} $$

We denote this statement by $Gal(F,\kappa ,\kappa ^+)$ . In particular, this holds for the club filter $Cub_{\kappa }$ as it is a normal filter over a cardinal $\kappa $ .

In [Reference Abraham and Shelah1], Abraham and Shelah constructed a model where $ Gal(Cub_{\kappa ^+},\kappa ^+,\kappa ^{++})$ fails for a regular $\kappa $ . Garti [Reference Garti13, Reference Garti14] and later together with the first author and Poveda [Reference Benhamou, Garti and Poveda4] continued the investigation of the Galvin property for the club filter. The Galvin property for $\kappa $ -complete ultrafilters over a measurable cardinal $\kappa $ was used recently in [Reference Benhamou and Gitik7, Reference Gitik18]. The question of failure of the Galvin property for such ultrafilters was shown to be independent. Namely, in [Reference Benhamou and Gitik7] the authors observed that in $L[U]$ every $\kappa $ -complete ultrafilter has the Galvin property, and Garti, Shelah, and the first author, starting with a supercompact cardinal, produced a model with a $\kappa $ -complete ultrafilter which contains $Cub_\kappa $ and fails to satisfy the Galvin property.

In Section 2, we isolate a property of sequences we call a strong witness for the failure of Galvin’s property which implies in particular the failure of Galvin’s property. This property is used in Theorem 2.6, where we start from a single measurable cardinal, and construct a model with an ultrafilter which fails to satisfy the Galvin property. This improves the initial large cardinal assumption of [Reference Benhamou, Garti and Shelah5].

Later in Theorem 2.10, we were able to slightly modify the construction of Theorem 2.6, construct an ultrafilter W and a strong witness for the failure of the Galvin property for it, which serves to glue together initial segments of functions, and obtain $\kappa ^+$ -mutually generic Cohen function on $\kappa $ . This idea is generalized to longer sequences (and in turn to more Cohen functions) in Theorems 3.1 and 3.3.

Our main results are:

Theorem 2.6. Assume GCH and let $\kappa $ be measurable in V. Then there is a cofinality preserving forcing extension $V^*$ in which there is a $\kappa $ -complete ultrafilter W over $\kappa $ which concentrates on regulars, extends $Cub_\kappa $ , and has a strong witness for the failure of Galvin’s property.

Theorem 2.10. Assume $GCH$ and that $\kappa $ is a measurable cardinal in V. Then there is a cofinality preserving forcing extension $V^*$ in which $GCH$ still holds, and there is a $\kappa $ -complete ultrafilter $U^*\in V^*$ over $\kappa $ such that forcing with Prikry forcing $Prikry(U^*)$ introduces a $V^*$ -generic filter for $Cohen^{V^*}(\kappa ,\kappa ^+)$ .

Theorem 3.1. Assume GCH and that there is a $(\kappa ,\kappa ^{++})$ -extender over $\kappa $ in V. Then there is a cofinality preserving forcing extension $V^*$ such that $V^*\models 2^{\kappa }=\kappa ^{++}$ , in $V^*$ there is a $\kappa $ -complete ultrafilter W over $\kappa $ which concentrates on regulars, extends $Cub_\kappa $ , and has a strong witness of length $\kappa ^{++}$ for the failure of Galvin’s property.

Theorem 3.3. Assume $GCH$ and that E is a $(\kappa ,\kappa ^{++})$ -extender in V. Then there is a cofinality preserving forcing extension $V^*$ in which $2^{\kappa }=\kappa ^{++}$ and a non-Galvin ultrafilter $W\in V^*$ such that forcing with $\operatorname {\mathrm {Prikry}}(W)$ introduces a $V^*$ -generic filter for $Cohen^{V^*}(\kappa ,\kappa ^{++})$ -generic filter.

Theorem 4.5. Let ${\mathcal P}_E$ be the Extender-based Prikry forcing of [Reference Gitik and Magidor21], and $G\subseteq {\mathcal P}$ be a generic. Suppose that $A\in V[G]\setminus V$ is a subset of $\kappa $ . Then $\kappa $ changes its cofinality to $\omega $ in $V[A]$ .

Theorem 4.6. Assume GCH, let E be an extender over $\kappa $ , and let $\mathbb {P}_E$ be the Merimovich version of the Extender-based Prikry forcing of [Reference Merimovich29Reference Merimovich31]. Let G be a generic subset of $\mathbb {P}_E$ and let ${\langle } {A}_\alpha \mid \alpha <\kappa ^{++} {\rangle }$ be different subsets of $\kappa $ in $V[G]$ . Then there is $I\subseteq \kappa ^{++}, I\in V, |I|=\kappa $ such that $\kappa $ is a singular cardinal of cofinality $\omega $ in $V[{\langle } {A}_\alpha \mid \alpha \in I{\rangle }]$ . In particular, there is no intermediate model of $V[G]$ where $\kappa $ is measurable and $2^{\kappa }>\kappa ^+$ .

This paper is organized as follows:

  • Section 1: We provide the basic definitions and background for this paper.

  • Section 2: We prove Theorems 2.6 and 2.10.

  • Section 3: We prove Theorems 3.1 and 3.3.

  • Section 4: We prove Theorems 4.5 and 4.6.

1 Basics

1.1 The forcing notions

In our notations $p\leq q$ means that q is stronger than p. We assume that the reader is familiar with the forcing method and iterated forcing. Most of our notations are inspired by [Reference Cummings, Foreman and Kanamori12, Reference Gitik, Foreman and Kanamori17] where we refer the reader for more information regarding forcing and iterations. Let us present the definitions of the forcing we intend to use:

Definition 1.1. The forcing adding $\lambda $ -many Cohen functions to $\kappa $ denoted by $\operatorname {\mathrm {Cohen}}(\kappa ,\lambda )$ consists of all partial functions $f:\kappa \times \lambda \rightarrow \{0,1\}$ such that $|f|<\kappa $ . The order is defined by $f\leq g$ iff $f\subseteq g$ .

Definition 1.2. Let U be a $\kappa $ -complete non-trivial ultrafilter over $\kappa $ and let $\pi :\kappa \rightarrow \kappa $ be the function representing $\kappa $ in the $\mathrm {Ult}(V,U)$ . The Prikry forcing with U, denoted by $\operatorname {\mathrm {Prikry}}(U)$ , consists of all sequences ${\langle }\alpha _1,...,\alpha _n, A{\rangle }$ such that:

  1. (1) ${\langle }\alpha _1,...,\alpha _n{\rangle }$ is a $\pi $ -increasing sequence of ordinals below $\kappa $ , i.e., for every $1\leq i<n$ , $\alpha _i<\pi (\alpha _{i+1}),$

  2. (2) $A\in U$ , $\pi (\min (A))>\alpha _n$ .

The order is defined by ${\langle }\alpha _1,...,\alpha _n,A{\rangle }\leq {\langle }\beta _1,...,\beta _m,B{\rangle }$ iff:

  1. (1) $n\leq m$ and for every $i\leq n$ , $\alpha _i=\beta _i$ ,

  2. (2) for every $n<i\leq m$ , $\beta _i\in A$ ,

  3. (3) $B\subseteq A$ .

If $n=m$ we say that q directly extends p and denote it by $p\leq ^* q$ .

If U is normal then we can take $\pi =id$ and the forcing $\operatorname {\mathrm {Prikry}}(U)$ is the standard Prikry forcing. The requirement that the sequence is $\pi $ -increasing ensures that the forcing $\operatorname {\mathrm {Prikry}}(U)$ is forcing equivalent to the tree-Prikry forcing defined in [Reference Gitik, Foreman and Kanamori17]. Also, it enables to define a diagonal intersection suitable for the non-normal case, namely, for $\{A_i\mid i<\kappa \}\subseteq U$ define

$$ \begin{align*}\Delta^*_{i<\kappa}A_i:=\{\alpha<\kappa\mid\forall i<\pi(\alpha).\alpha\in A_i\}.\end{align*} $$

This kind of diagonal intersection instead of the standard one is used to prove the Prikry property of $\operatorname {\mathrm {Prikry}}(U)$ .

Later we will need the easy direction of the Mathias criterion [Reference Mathias28] for Prikry-generic sequences, and the proof can be found in [Reference Benhamou3, Corollary 4.22]:

Lemma 1.3. Let $G\subseteq \operatorname {\mathrm {Prikry}}(U)$ be a generic filter producing a Prikry sequence $\{c_n\mid n<\omega \}$ . Then for every $A\in U$ , there is $N<\omega $ such that for every $n\geq N$ , $c_n\in A$ .

For more information regarding the tree-Prikry forcing see [Reference Gitik, Foreman and Kanamori17] or [Reference Benhamou3]. In the following, we define the notion of lottery sum. The terminology “lottery sum” is due to Hamkins, although the concept of the lottery sum of partial orderings has been around for quite some time and has been referred to, for example, as “disjoint sum of partial orderings”:

Definition 1.4. Let $\mathbb {P}_0,\mathbb {P}_1$ be two forcing notions. The lottery sum of $\mathbb {P}_0$ and $\mathbb {P}_1$ denoted by $\operatorname {\mathrm {LOTT}}(\mathbb {P}_0,\mathbb {P}_1)$ is the forcing whose underlining set is $\mathbb {P}_0\times \{0\}\cup \mathbb {P}_1\times \{1\}$ and the order is define by ${\langle } p,i{\rangle }\leq {\langle } p',j{\rangle }$ iff $i=j$ and $p\leq _{\mathbb {P}_i} p'$ .

The forcing $\operatorname {\mathrm {LOTT}}(\mathbb {P}_0,\mathbb {P}_1)$ generically chooses $\mathbb {P}_0$ or $\mathbb {P}_1$ and adds a V-generic filter for it. As Hamkins observed in [Reference Hamkins22], iterating such forcing notions leaves a certain amount of freedom when lifting ground model embeddings; this will be exploited in most of our construction.

In Section 4 we will discuss the Extender-based Prikry forcing which was originally defined by Magidor and the second author in [Reference Gitik and Magidor21]. A more recent variation of it is due to Merimovich [Reference Merimovich29Reference Merimovich31].

Let us present the two versions. Let E be a $(\kappa ,\lambda )$ -extender and $j=j_E:V\rightarrow M_E\simeq Ult(V,E)$ the natural elementary embedding (see [Reference Jech23] for the definition of extenders and related constructions) and suppose that $f_\lambda :\kappa \rightarrow \kappa $ is a function such that $j(f_\lambda )(\kappa )=\lambda $ (our result uses $\lambda =\kappa ^{++}$ and we can simply take $f_\lambda (\nu )=\nu ^{++}$ ). Let us first present the Merimovich version of the Extender-based Prikry forcing.

For each set of cardinality $\leq \kappa $ , $d\in [\lambda \setminus \kappa ]^{\leq \kappa }$ with $\kappa \in d$ . Define

$$ \begin{align*}E(d)=\{X\in V_\kappa\mid (j\restriction d)^{-1}\in j(X)\}.\end{align*} $$

If $A\in E(d)$ we can assume that for every $\nu ,\mu \in A$ , $\nu :d\rightarrow \kappa $ is order preserving, $\kappa \in \operatorname {\mathrm {dom}}(\nu )$ , $|\nu |\leq \nu (\kappa )$ , $\nu (\kappa )=\mu (\kappa )\rightarrow \operatorname {\mathrm {dom}}(\nu )=\operatorname {\mathrm {dom}}(\mu )$ . Merimovich calls such a set a good set.

Definition 1.5. The conditions of $\mathbb {P}_E$ are pairs $p={\langle } f^p,A^p{\rangle }$ such that:

  1. (1) $f^p:d\rightarrow [\kappa ]^{<\omega }$ is the “Cohen Part” of the condition, $d\in [\lambda \setminus \kappa ]^{<\omega }$ , $\kappa \in d$ .

  2. (2) $A^p\in E(d)$ is a good set.

  3. (3) For every $\nu \in A^p$ and $\alpha \in \operatorname {\mathrm {dom}}(\nu )$ , $\max (f^p(\alpha ))<\nu (\kappa )$ .

The order of $\mathbb {P}_E$ is defined in two steps: a direct extension is defined by ${\langle } f,A{\rangle }\leq ^* {\langle } g,B{\rangle }$ if:

  1. (1) $f\subseteq g$ .

  2. (2) $B\restriction \operatorname {\mathrm {dom}}(f):=\{\nu \restriction \operatorname {\mathrm {dom}}(\nu )\cap \operatorname {\mathrm {dom}}(f)\mid \nu \in B\}\subseteq A$ .

A one-point extension of $p={\langle } f,A{\rangle }$ for $\nu \in A$ is defined by $p^{\smallfrown }\nu ={\langle } g,B{\rangle }$ where:

  1. (1) $\operatorname {\mathrm {dom}}(g)=\operatorname {\mathrm {dom}}(f)$ .

  2. (2) For every $\alpha \in \operatorname {\mathrm {dom}}(g),$

    $$ \begin{align*}g(\alpha)=\begin{cases} f(\alpha)^{\smallfrown}\nu(\alpha), & \alpha\in\operatorname{\mathrm{dom}}(\nu),\\ f(\alpha), & \text{else}.\end{cases}\end{align*} $$
  3. (3) $B=\{\mu \in A\mid \sup _{\alpha \in \operatorname {\mathrm {dom}}(\nu )}(\nu (\alpha )+1)\leq \mu (\kappa )\}$ .

An n-point extension $p{}^{\smallfrown }\vec {\nu }$ is defined recursively by consecutive one-point extensions. A general extension is defined by $p\leq q$ iff for some $\vec {\nu }\in [A^p]^{<\omega }$ , $p^{\smallfrown }\vec {\nu }\leq ^* q$ .

As in [Reference Merimovich30], we will sometime replace the large set A in a condition ${\langle } f,A{\rangle }$ with a Tree T which is $E(\operatorname {\mathrm {dom}}(f))$ -fat.

Let us now present the original version defined by Magidor and the second author from [Reference Gitik and Magidor21]. Define for every $\kappa \leq \alpha <\lambda $ :

$$ \begin{align*}U_\alpha:=\{X\subseteq\kappa\mid \alpha\in j(X)\}.\end{align*} $$

These are P-point ultrafilters. For every $\alpha \leq \beta <\lambda $ we define that $\alpha \leq _E\beta $ if there is some $f:\kappa \rightarrow \kappa $ , $j(f)(\beta )=\alpha $ . This implies that f Rudin–Keisler projects $U_\beta $ onto $U_\alpha $ . For every such pair $\alpha \leq _E\beta $ fix such a projection $\pi _{\beta ,\alpha }$ such that $\pi _{\alpha ,\alpha }=id$ . The projections to the normal measure $U_\kappa $ have a uniform definition, $\pi _{\alpha ,\kappa }(\nu )=\nu ^0$ where $\nu ^0$ is the maximal inaccessible $\nu ^*\leq \nu $ such that $f_\lambda \restriction \nu ^*:\nu ^*\rightarrow \nu ^*$ , $f_\lambda (\nu ^*)>\nu $ , and $\pi _{\alpha ,\kappa }(\nu )=0$ if there is no such $\nu ^*$ . Suppose that the system ${\langle } U_\alpha ,\pi _{\alpha ,\beta }\mid \alpha \leq \beta <\lambda , \alpha \leq _E\beta {\rangle }$ is a nice system (see [Reference Gitik and Magidor21] or [Reference Gitik, Foreman and Kanamori17, Discussion after Lemma 3.5]). Let us say that $\nu $ is permitted for $\nu _0,...,\nu _n$ is $\nu ^0>\max _{i=0,...,n}\nu ^0_i$ .

Definition 1.6. The conditions of the forcing $\mathcal {P}_E$ are pairs $p={\langle } f,T{\rangle }$ such that:

  1. (1) $f:\lambda \setminus \kappa \rightarrow [\kappa ]^{<\omega }$ , $\kappa \in \operatorname {\mathrm {dom}}(f)$ , $|f|\leq \kappa $ .

  2. (2) For each $\alpha \in \operatorname {\mathrm {Supp}}(p):=\operatorname {\mathrm {dom}}(f)$ , $\pi _{\alpha ,\kappa }^{\prime \prime }f(\alpha )$ is a finite increasing sequence.

  3. (3) The domain of f has a $\leq _E$ -maximal element $mc(p):=\alpha =\max (\operatorname {\mathrm {Supp}}(p))$ .

  4. (4) $\pi _{mc(p),\kappa }"f(mc(p))=f(\kappa )$ .

  5. (5) For every $\gamma \in \operatorname {\mathrm {Supp}}(p)$ , $\pi _{mc(p),\gamma }(\max (f(mc(p)))$ is not permitted to $f(\gamma )$ .

  6. (6) T is a $U_{mc(p)}$ -splitting tree with stem $f(mc(p))$ , namely, for $s\in T$ , either $s\leq t$ , or $s\geq t$ and $\operatorname {\mathrm {Succ}}_T(s):=\{\alpha <\kappa \mid s{}^{\smallfrown }\alpha \in T\}\in U_{mc(p)}$ .

  7. (7) For every $\nu \in \operatorname {\mathrm {Succ}}_T(f(mc(p)))$ ,

    $$ \begin{align*}|\{\gamma\in \operatorname{\mathrm{Supp}}(p)\mid \nu\text{ is permitted to } f(\gamma)\}|\leq \nu^0.\end{align*} $$

The order is defined $p\leq q$ if:

  1. (1) $\operatorname {\mathrm {Supp}}(p)\subseteq \operatorname {\mathrm {Supp}}(q)$ .

  2. (2) For $\gamma \in \operatorname {\mathrm {Supp}}(p)$ , $f^q(\gamma )$ is an end-extension of $f^p(\gamma )$ .

  3. (3) $f^q(mc(p))\in T^p$ .

  4. (4) For $\gamma\hspace{-0.5pt} \in\hspace{-0.5pt} \operatorname {\mathrm {Supp}}(p)$ , $f^q(\gamma )\setminus f^p(\gamma )\hspace{-0.5pt}=\hspace{-0.5pt}\pi _{mc(p),\gamma }^{\prime \prime }f^q(mc(p))\hspace{-0.5pt}\setminus\hspace{-0.5pt} f^p(mc(p))\restriction (i+1)$ , where i is maximal such that $f^q(mc(p))$ is not permitted for $f^p(\gamma )$ .

  5. (5) $\pi _{mc(q),mc(p)}^{\prime \prime }T^q\subseteq T^p$ .

  6. (6) For every $\gamma \in \operatorname {\mathrm {Supp}}(p)$ , and $\nu \in \operatorname {\mathrm {Succ}}_{T^q}(f^q(mc(q)))$ , such that $\nu $ is permitted for $f^q(\gamma )$ (so by condition (7) there are only $\nu ^0$ -many such $\gamma $ ’s) then $\pi _{mc(q),\gamma }(\nu )=\pi _{mc(p),\gamma }(\pi _{mc(q),mc(p)}(\nu )).$

1.2 Canonical functions

The main construction of this paper uses the notion of canonical functions:

Definition 1.7. For every limit ordinal $\delta <\kappa ^+$ , fix a cofinal sequence $\bar {\delta }=\{ \delta _i\mid i<cf(\delta )\}$ . Let us define inductively functions $\tau _\alpha :\kappa \rightarrow \kappa $ for $\alpha <\kappa ^+$ :

$$ \begin{align*}\tau_0(x)=0,\end{align*} $$
$$ \begin{align*}\tau_{\alpha+1}(x)=\tau_\alpha(x)+1,\end{align*} $$
$$ \begin{align*}\text{For limit }\delta, \ \tau_{\delta}(x)=\sup_{y<\min(x,cf(\delta))}\tau_{\delta_y}(x).\end{align*} $$

Proposition 1.8. Let $\lambda \leq \kappa $ be a regular cardinal. Then:

  1. (1) For every $\alpha <\beta <\lambda ^+$ , $\{\nu \mid \tau _{\alpha }(\nu )\geq \tau _{\beta }(\nu )\}$ is bounded in $\lambda $ .

  2. (2) For every any $\alpha <\lambda ^+$ , $\tau _{\alpha }:\lambda \rightarrow \lambda $ .

  3. (3) For every normal measure $\mathcal {V}$ on $\lambda $ , and for every $\alpha <\lambda ^+$ , $[\tau _{\alpha }]_{\mathcal {V}}=\alpha $ .

  4. (4) If $\lambda <\kappa $ , then for every $\beta $ , $\tau _{\beta }(\lambda )<\lambda ^+$ .

Proof For $(1)$ , we prove inductively on $\beta <\lambda ^+$ that for every $\alpha <\beta $ , $(1)$ holds. For $\beta =0$ this is vacuous. The successor stage is also easy since for every x, $\tau _{\beta }(x)<\tau _{\beta +1}(x)$ so if $\alpha <\beta $ then by induction hypothesis there is $\xi <\lambda $ from which $\tau _{\beta }$ dominates $\tau _{\alpha }$ , i.e., $\forall \nu \in (\xi ,\lambda ).\tau _\alpha (\nu )<\tau _{\beta }(\nu )$ . It follows that for the same $\xi $ , $\tau _{\alpha }(\nu )<\tau _{\beta +1}(\nu )$ . As for limit points $\delta $ . Fix any $\alpha <\delta $ , then there is $i< cf(\delta )\leq \lambda $ such that $\delta _i>\alpha $ . By induction hypothesis there is $\xi _i<\lambda $ such that $\tau _{\delta _i}(\nu )>\tau _{\alpha }(\nu )$ for every $\nu \in (\xi _i,\lambda )$ . Let $\xi ^*:=\max \{\xi _i, i\}+1<\lambda $ . It follows that for every $\nu \in (\xi ^*,\lambda )$ , $\nu>i$ , and hence

$$ \begin{align*}\tau_{\delta}(\nu)=\sup_{y<\min(\nu,cf(\delta))}\tau_{\delta_y}(\nu)\geq \tau_{\delta_i}(\nu)>\tau_{\alpha}(\nu).\end{align*} $$

Prove $(2)$ $(4)$ by induction on $\alpha <\lambda ^+$ . For $\alpha =0$ this is trivial. Suppose that $(2)$ $(4)$ hold for $\alpha $ then clearly by induction hypothesis $\tau _{\alpha +1}:\lambda \rightarrow \lambda $ , and $\tau _{\alpha +1}(\lambda )=\tau _{\alpha }(\lambda )+1<\lambda ^+$ , namely $(2)$ and $(4)$ follow. Also, $\lambda =\{\nu <\lambda \mid \tau _{\alpha }(\nu )+1=\tau _{\alpha +1}(\nu )\}\in \mathcal {V}$ , ane hence by the Lós theorem and the induction hypothesis:

$$ \begin{align*}\alpha+1=[\tau_{\alpha}]_{\mathcal{V}}+1=[\tau_{\alpha+1}]_{\mathcal{V}}.\end{align*} $$

Suppose that $\delta <\lambda ^+$ is limit, then by induction hypothesis, for every $x<\lambda $ and $y<\min (x,cf(\delta ))<\lambda $ , $\tau _{\delta _y}(x)<\lambda $ . It follows from the regularity of $\lambda $ that

$$ \begin{align*}\tau_\delta(x)=\sup_{y<\min(x,cf(\delta))}\tau_{\delta_y}(x)<\lambda.\end{align*} $$

This concludes $(2)$ . Also, $(4)$ follows similarly using the regularity of $\lambda ^+$ . As for $(3)$ , we use $(1)$ to conclude that for every $\alpha <\delta $ , $\{\nu <\lambda \mid \tau _{\alpha }(\nu )\geq \tau _{\delta }(\nu )\}$ is bounded. Hence by induction $\alpha =[\tau _{\alpha }]_{\mathcal {V}}<[\tau _{\delta }]_{\mathcal {V}}$ . It follows that $\delta \leq [\tau _{\delta }]_{\mathcal {V}}$ . For the other direction, suppose that $[f]_{\mathcal {V}}<[\tau _{\delta }]_{\mathcal {V}}$ , then

$$ \begin{align*}E:=\{x<\lambda\mid f(x)<\tau_{\delta}(x)\}\in\mathcal{V}.\end{align*} $$

By definition of $\tau _{\delta }$ , for every $x\in E$ , there is $y_x<\min (x,cf(\delta ))$ such that $\tau _{\delta _{t_x}}(x)>f(x)$ . The function $x\mapsto y_x$ is regressive, and by normality we conclude that there is $y^*<cf(\delta )$ and $E'\subseteq E$ such that for every $x\in E'$ , $f(x)<\tau _{\delta _{y^*}}(x)$ . Hence $[f]_{\mathcal {V}}<[\tau _{\delta _{y^*}}]_{\mathcal {V}}=\delta _{y^*}<\delta $ and in turn $\delta =[\tau _{\delta }]_{\mathcal {V}}$ .

2 The results where GCH holds

2.1 Non-Galvin ultrafilter from optimal assumption

In [Reference Benhamou, Garti and Shelah5], Garti, Shelah, and the first author constructed a model with a $\kappa $ -complete ultrafilter which contains $Cub_\kappa $ and fails to satisfy the Galvin property. The initial assumption was a supercompact cardinal and the construction went through adding slim Kurepa trees.

Here we present a different construction. Our initial assumption will be a measurable cardinal and the property obtained will be a certain strengthening of the negation of the Galvin property. It will be used further to produce many Cohens.

Let us first present the stronger form of negation:

Definition 2.1. Let U be a $\kappa $ -complete ultrafilter non-normal over $\kappa $ . We call a family $\{ A_\alpha \mid \alpha <\kappa ^+\}\subseteq U$ a strong witness for the failure of the Galvin property iff for every subfamily ${\langle } A_{\alpha _\xi } \mid \xi <\kappa {\rangle }$ of size $\kappa $ the following holds:

$$ \begin{align*}\text{ for every }\zeta, \kappa\leq \zeta< [id]_U, \ [id]_U \not \in A^{\prime}_{\alpha_\zeta},\end{align*} $$
$$ \begin{align*}\text{where }{\langle} A^{\prime}_{\alpha_\zeta} \mid \zeta<j_U(\kappa){\rangle}=j_U({\langle} A_{\alpha_\xi} \mid \xi<\kappa{\rangle}).\end{align*} $$

Remark 2.2.

  1. (1) Note that the interval $[\kappa , [id]_U)$ is non-empty since U is not normal.

  2. (2) The family $\{ A_\alpha \mid \alpha <\kappa ^+\}$ witnesses the failure of the Galvin property for U.

Proof Since whenever ${\langle } A_{\alpha _\xi } \mid \xi <\kappa {\rangle }$ is a subfamily of size $\kappa $ , then $\bigcap _{\xi <\kappa } A_{\alpha _\xi }$ is not in U. Otherwise, suppose that $\bigcap _{\xi <\kappa } A_{\alpha _\xi }=B \in U$ . Then $[id]_U \in j_U(B)$ , but $j_U(B)=\bigcap _{\zeta <j_U(\kappa )}A^{\prime }_{\alpha _\zeta }$ . However, $[id]_U \not \in A^{\prime }_{\alpha _\zeta }$ , for every $\zeta , \kappa \leq \zeta < [id]_U$ . Contradiction.

Lemma 2.3. Suppose that $\{ A_\alpha \mid \alpha <\kappa ^+\}$ is a strong witness for the failure of the Galvin property of the ultrafilter U over $\kappa $ . Let $U^0=\{X\subseteq \kappa \mid \kappa \in j_U(X)\}$ be a projection of U to a normal ultrafilter, $\nu \mapsto \pi _{nor}(\nu )$ a projection map, and $k: M_{U^0}\to M_U$ the corresponding elementary embedding. Assume that $crit(k)=j_{U^0}(\kappa )=[id]_U$ . Then $[id]_U \not \in B$ , for every $B \in j_U(\{ A_\alpha \mid \alpha <\kappa ^+\})$ which is in $\operatorname {\mathrm {rng}}(k)\setminus \operatorname {\mathrm {rng}}(j_U)$ .

Proof Let B be as in the statement of the lemma. Pick $A'\subseteq j_{U^0}(\kappa )$ such that $k(A')=B$ . Then $A' \not \in \operatorname {\mathrm {rng}}(j_{U^0})$ , since otherwise its image B will be in the range of $j_U=k\circ j_{U^0}$ . Denote by

$$ \begin{align*}\{A^{\prime}_{\nu}\mid \nu<j_{U^0}(\kappa^+)\}=j_{U^0}(\{A_i\mid i<\kappa^+\}),\end{align*} $$
$$ \begin{align*}\{A^{\prime\prime}_{\nu}\mid \nu<j_{U}(\kappa^+)\}=j_{U}(\{A_i\mid i<\kappa^+\}).\end{align*} $$

Since $U^0$ is normal, there is $f:\kappa \rightarrow \kappa ^+$ such that $A'=A^{\prime }_{j_{U^0}(f)(\kappa )}$ and thus

$$ \begin{align*}B=k(A')=k(A^{\prime}_{{j_{U^0}(f)(\kappa)}})=A^{\prime\prime}_{j_{U}(f)(\kappa)}.\end{align*} $$

Since B is not in the range of k, f is not constant. Recall that $\{ A_\alpha \mid \alpha <\kappa ^+\}$ is a strong witness for U being non-Galvin ultrafilter over $\kappa $ . Apply this to the family $\{ A_{f(\nu )} \mid \nu <\kappa \}.$ It follows that $[id]_U\not \in A^{\prime \prime }_{j_{U}(f)(\kappa )}=B.$

Before proving the main result of this section we present two preservation theorems for being a strong witnesses for the failure of the Galvin property. These theorems are not used later and the reader can proceed directly to Theorem 2.6.

Theorem 2.4. Assume $2^{\kappa }=\kappa ^+$ . Suppose that the family $\{ A_\alpha \mid \alpha <\kappa ^+\}$ is a strong witness for U being a non-Galvin ultrafilter over $\kappa $ . Let $U^0=\{X\subseteq \kappa \mid \kappa \in j_U(X)\}$ be a projection of U to a normal ultrafilter, $\nu \mapsto \pi _{nor}(\nu )$ a projection map, and $k: M_{U^0}\to M_U$ the corresponding elementary embedding. Assume that $crit(k)=j_{U^0}(\kappa )$ and $[id]_U=j_{U^0}(\kappa )$ .

Suppose that $V^*$ is an extension of V in which all the embeddings $j_{U^0},j_U, k$ extend to an elementary embedding $j^{0*}:V^*\to M^{0*},j^*:V^*\to M^*$ , $k^*:M^{0*}\to M^*$ . Define $U^*=\{X \subseteq \kappa \mid [id]_U \in j^*(X)\}$ .

Then $\{ A_\alpha \mid \alpha <\kappa ^+\}$ is a strong witness that $U^*$ is a non-Galvin ultrafilter over $\kappa $ .

Proof Note that $(\kappa ^+)^{V^*}=(\kappa ^+)^V$ . Just otherwise, $(\kappa ^{++})^V $ will be $\leq (\kappa ^+)^{V^*}$ , and then, $j^*(\kappa )>(\kappa ^{++})^V$ . This is impossible, since $j^*$ extends $j_U$ . The rest follows from the previous lemma and the fact that $[\kappa , [id]_U)\subseteq \operatorname {\mathrm {rng}}(k)\setminus \operatorname {\mathrm {rng}}(j_{U})$ since $crit(k)=j_{U^0}(\kappa )=[id]_U$ .

Theorem 2.5. Assume $2^{\kappa }=\kappa ^+$ . Suppose that $\{ A_\alpha \mid \alpha <\kappa ^+\}$ is a strong witness for U being a non-Galvin ultrafilter over $\kappa $ which contains $Cub_\kappa $ and be a witnessing family.

Let $V^*$ be a $\kappa $ -c.c. extension of V in which $j_U$ extends to an elementary embedding $j^*:V^*\to M^*$ , where $M^*$ is a corresponding extension of $M_U$ .

Define $U^*=\{X \subseteq \kappa \mid [id]_U \in j^*(X)\}$ .

Then $\{ A_\alpha \mid \alpha <\kappa ^+\}$ is a strong witness that $U^*$ is a non-Galvin ultrafilter over $\kappa $ .

Proof Suppose now that ${\langle } A_{\alpha _\xi } \mid \xi <\kappa {\rangle }$ is a subfamily of $\{ A_\alpha \mid \alpha <\kappa ^+\}$ of size $\kappa $ in ${V^*}$ .

Work in V. Let be a name of $\alpha _\xi $ . By $\kappa $ -c.c., then for every $\xi <\kappa $ there will be $s_\xi \subseteq \kappa ^+$ of cardinality less than $\kappa $ , such that .

Let $S=\sup _{\xi <\kappa }s_\xi $ . Enumerate $S={\langle } \beta _i\mid i<\kappa {\rangle }$ such that we if $\beta _i\in s_\zeta $ and $\beta _j\in s_\mu $ where $\zeta <\mu $ then $i<j$ , i.e., enumerate first $s_0$ then $s_1$ and so on, such that the resulting enumeration of S is of order-type $\kappa $ . This is possible since each $s_\zeta $ has cardinality less than $\kappa $ . Define

$$ \begin{align*}C=\{\nu<\kappa \mid \forall \xi<\nu (\sup(\gamma\mid \beta_\gamma\in s_\xi)<\nu)\}.\end{align*} $$

Clearly, C is a club. Hence $[id]_U\in j_U(C)$ . Then, by elementarity, for every $\zeta <[id]_U$ , and every $\beta _i\in s^{\prime }_\zeta $ , $i<[id]_U$ .

Let us use the fact that the sequence ${\langle } A_\alpha \mid \alpha <\kappa ^+{\rangle }$ is a strong witness for U being non-Galvin, hence $[id]_U \not \in A^{\prime }_{\beta _\zeta }$ , for every $\kappa \leq \zeta < [id]_U$ . Fix any $\kappa \leq \xi <[id]_U$ , then by elementarity we have in $M_U$ . Therefore there is some $\gamma <\kappa $ such that $\alpha ^{\prime }_\xi =\beta _\gamma $ . Clearly, $\gamma \geq \kappa $ , and by the closure property of $[id]_U$ , we conclude that $\gamma <[id]_U$ . Hence, in $M^*$ , $[id]_U \not \in A^{\prime }_{\beta ^{\prime }_\gamma }=A^{\prime }_{\alpha ^{\prime }_\xi }$ , as wanted.

Theorem 2.6. Assume GCH and let $\kappa $ be measurable in V. Then there is a cofinality preserving forcing extension $V^*$ in which there is a $\kappa $ -complete ultrafilter W over $\kappa $ which concentrates on regulars, extends $Cub_\kappa $ , and has a strong witness for the failure of Galvin’s property.

Proof The forcing is simply adding for each inaccessible $\alpha \leq \kappa $ , $\alpha ^+$ -many Cohen functions to $\alpha $ . Namely, consider the Easton support iteration

such that for $\alpha \leq \kappa $ ,

is trivial unless $\alpha $ is inaccessible, in which case it is a $\mathcal {P}_\alpha $ -name for $\operatorname {\mathrm {Cohen}}(\alpha ,\alpha ^+)$ .

Let $G:=G_\kappa *g_\kappa $ be V-generic for . Denote ${\langle } f_{\kappa ,\alpha }\mid \alpha <\kappa ^+{\rangle }$ be the enumeration of the $\kappa ^+$ Cohen functions added by $g_\kappa $ . The idea is that the sets which are going to be a strong witness for the failure of the Galvin property are ${\langle } A_\alpha \mid \alpha <\kappa ^+{\rangle }$ , where

$$ \begin{align*}A_\alpha=\{\beta<\kappa\mid f_{\kappa,\alpha}(\beta)=1\}.\end{align*} $$

The next step is to construct the measure for this witness by extending ground model embeddings to $V[G]$ . Let $U\in V$ be a normal measure over $\kappa $ and consider the second ultrapower by U and the corresponding commutative diagram

$$ \begin{align*}j_1:=j_{U}:V\rightarrow M_U=:M_1, \ j_2:=j_{U^2}:V\rightarrow M_{U^2}=:M_2\end{align*} $$
$$ \begin{align*}k:M_1\rightarrow M_{2}, \ j_{2}=k\circ j_1,\end{align*} $$

where k is simply the ultrapower embedding defined in $M_U$ using the ultrafilter $j_{1}(U)$ . Denote $\kappa _1=j_1(\kappa )$ and $\kappa _2=j_{2}(\kappa )$ , then $k(\kappa _1)=\kappa _2$ .

By Easton support and elementarity,

where

is the quotient forcing above $\kappa $ , which is forcing equivalent to the continuation of the iteration above $\kappa $ using the same recipe as $\mathcal {P}_\kappa $ .

In $V[G]$ , let us first construct an M-generic filter for . Take $G_{\kappa }*g_{\kappa }$ to be the generic up to $\kappa $ including $\kappa $ . Above $\kappa $ , from the point of view of $V[G]$ , we have $\kappa ^+$ -closure for $\mathcal {P}_{(\kappa ,\kappa _1)}$ . By $GCH$ , and since $j_1$ is an ultrapower by a measure, there are only $\kappa ^+$ -many dense open subsets of this forcing to meet. Therefore we can construct in $V[G]$ by standard construction an $M_1[G]$ -generic filter $G_{(\kappa ,\kappa _1)}$ for $\mathcal {P}_{(\kappa ,\kappa _1)}$ . By $\kappa _1^+-cc$ of $Q_{\kappa _1}$ , we can find $g^{\prime }_{\kappa _1}$ which is $M_1[G*G_{(\kappa ,\kappa _1)}]$ -generic for $Q_{\kappa _1}$ . We need to change the values of $g^{\prime }_{\kappa _1}={\langle } f^{\prime }_{\kappa _1,\alpha }\mid \alpha <\kappa _1^{+}{\rangle }$ to $g_{\kappa _1}={\langle } f_{\kappa _1,\alpha }\mid \alpha <\kappa _1^{+}{\rangle }$ such that for every $\alpha <\kappa ^{+}$ , $f_{\kappa _1,j_1(\alpha )}\restriction \kappa =f_{\kappa ,\alpha }$ . This will ensure that the Silver criterion to lift an elementary embedding holds, namely, $j_1^{\prime \prime }G_{\kappa }*g\subseteq G_{\kappa }*g*G_{(\kappa ,\kappa _1)}*g^{\prime }_{\kappa _1}$ . Also, we would like to tweak the values of $f_{\kappa _1,j_1(\alpha )}(\kappa )$ to ensure that the sets $A_\alpha $ are members of the ultrafilter generated by $\kappa $ . By the definition of $A_\alpha $ , the way to do this is to set $f_{\kappa _1,j_1(\alpha )}(\kappa )=1$ .

Formally, for each condition $p\in \operatorname {\mathrm {Cohen}}(\kappa _1,\kappa _1^+)^{M_1[G_{\kappa }*G*G_{(\kappa ,\kappa _1)}]}$ , define a function $p^*$ with $\operatorname {\mathrm {dom}}(p^*)=\operatorname {\mathrm {dom}}(p)$ and for every ${\langle } \gamma ,\alpha {\rangle }\in \operatorname {\mathrm {dom}}(p^*)$ ,

$$ \begin{align*}p^*({\langle}\gamma,\alpha{\rangle})=\begin{cases} f_{\kappa,\beta}(\gamma), & \gamma<\kappa\wedge j_1(\beta)=\alpha,\\ 1, & \gamma=\kappa\wedge j_1(\beta)=\alpha,\\ p({\langle}\gamma,\alpha{\rangle}), & \text{else}.\end{cases}\end{align*} $$

Let $g_{\kappa _1}:=\{p^*\mid p\in g^{\prime }_{\kappa _1}\}$ . Clearly, the functions ${\langle } f_{\kappa _1,\alpha }\mid \alpha <\kappa _1^+{\rangle }$ derived from $g_{\kappa _1}$ satisfy that $f_{\kappa _1,j_1(\beta )}\restriction \kappa =f_{\kappa ,\beta }$ and $f_{\kappa _1,j_1(\beta )}(\kappa )=1$ for every $\beta <\kappa ^+$ . It remains to show that $g_{\kappa _1}$ is generic:

Lemma 2.7. The filter $g_{\kappa _1}$ is $\operatorname {\mathrm {Cohen}}(\kappa _1,\kappa _1^+)^{M_1[G_{\kappa }*G*G_{(\kappa ,\kappa _1)}]}$ -generic filter over $M_1[G_{\kappa }*g*G_{(\kappa ,\kappa _1)}]$ .

Proof First let us prove that $g_{\kappa _1}\subseteq \operatorname {\mathrm {Cohen}}(\kappa _1,\kappa _1^+)^{M_1[G_{\kappa }*G*G_{(\kappa ,\kappa _1)}]}$ . Indeed, $g^{\prime }_{\kappa _1}\subseteq \operatorname {\mathrm {Cohen}}(\kappa _1,\kappa _1^+)^{M_1[G_{\kappa }*G*G_{(\kappa ,\kappa _1)}]}$ and for any $p\in g^{\prime }_{\kappa _1}$ ,

$$ \begin{align*} M_1[G_{\kappa}*G*G_{(\kappa,\kappa_1)}]\models|p|<\kappa_1, \end{align*} $$

and hence $\operatorname {\mathrm {dom}}(p)_{\leq \kappa }:=\{\alpha \mid \exists {\langle }\gamma ,\alpha {\rangle }\in \operatorname {\mathrm {dom}}(p),\gamma \leq \kappa \}$ is bounded in $\kappa _1^+$ while $j_1^{\prime \prime }\kappa ^+$ is unbounded. It follows that there is $\theta <\kappa ^+$ such that

$$ \begin{align*} \operatorname{\mathrm{dom}}(p)_{\leq\kappa}\cap j_1^{\prime\prime}\kappa^+\subseteq j_1^{\prime\prime}\theta. \end{align*} $$

Hence from the V-perspective, $|\operatorname {\mathrm {dom}}(p)_{\leq \kappa }\cap j_1^{\prime \prime }\kappa ^+|\leq \kappa $ . The difference between p and $p^*$ is only on the coordinates of $\operatorname {\mathrm {dom}}(p)_{\leq \kappa }\cap j_1^{\prime \prime }\kappa ^+$ and by closure of $M_1[G_{\kappa }*g*G_{(\kappa ,\kappa _1)}]$ to $\kappa $ -sequences it follows that

$$ \begin{align*}p^*\in \operatorname{\mathrm{Cohen}}(\kappa_1,\kappa_1^+)^{M_1[G_{\kappa}*G*G_{(\kappa,\kappa_1)}]}, \ g_{\kappa_1}\subseteq\operatorname{\mathrm{Cohen}}(\kappa_1,\kappa_1^+)^{M_1[G_{\kappa}*G*G_{(\kappa,\kappa_1)}]}.\end{align*} $$

To see that $g_{\kappa _1}$ is generic over $M_1[G_{\kappa }*G*G_{(\kappa ,\kappa _1)}]$ , let $D\in M_1[G_{\kappa }*G*G_{(\kappa ,\kappa _1)}]$ be dense open. In $M_1[G_{\kappa }*G*G_{(\kappa ,\kappa _1)}]$ , define $D^*$ to consist of all conditions $p\in \operatorname {\mathrm {Cohen}}(\kappa _1,\kappa _1^+)$ . Such that

$$ \begin{align*}\forall q. \operatorname{\mathrm{dom}}(q)=\operatorname{\mathrm{dom}}(p)\wedge |\{x\mid p(x)\neq q(x)\}|\leq \kappa\rightarrow q\in D\end{align*} $$

then $D^*$ is dense open. To see this, pick any $p\in \operatorname {\mathrm {Cohen}}(\kappa _1,\kappa _1^+)^{M_1[G_{\kappa }*G*G_{(\kappa ,\kappa _1)}]}$ and enumerate by ${\langle } q_r\mid r<\theta {\rangle }$ all the conditions q such that

$$ \begin{align*}\operatorname{\mathrm{dom}}(q)=\operatorname{\mathrm{dom}}(p)\wedge |\{x\mid p(x)\neq q(x)\}|\leq\kappa.\end{align*} $$

Note $\theta <\kappa _1$ since $\kappa _1$ is inaccessible in $ M_U[G_{\kappa }*g*G_{(\kappa ,\kappa _1)}]$ . We define inductively and increasing sequence ${\langle } p_r\mid r<\theta {\rangle }$ , and exploit the $\kappa _1$ -closure of $\operatorname {\mathrm {Cohen}}(\kappa _1,\kappa _1^+)$ to take care of limit stages. Define $p_0=p$ , and suppose that $p_r$ is defined, let $p_{r+1}^{\prime }:=q_r\cup p_r\restriction (\operatorname {\mathrm {dom}}(p_r)\setminus \operatorname {\mathrm {dom}}(p))$ , find $p_{r+1}^{\prime }\leq t_{r+1}\in D$ which exists by density and set

$$ \begin{align*}p_{r+1}=p_r\restriction \operatorname{\mathrm{dom}}(p)\cup t_{r+1}\restriction(\operatorname{\mathrm{dom}}(t_{r+1})\setminus \operatorname{\mathrm{dom}}(p)).\end{align*} $$

Then $p_r\leq p_{r+1}$ . Let

$$ \begin{align*}p^*:=\cup_{r<\theta} p_{r}\end{align*} $$

then $p^{*}$ has the property that for $\kappa $ many changes of $p^*$ from the domain of p stays inside D. Namely any q with $\operatorname {\mathrm {dom}}(q)=\operatorname {\mathrm {dom}}(p^*)$ ,

$$ \begin{align*}q\restriction (\operatorname{\mathrm{dom}}(p^*)\setminus \operatorname{\mathrm{dom}}(p))=p^*\restriction(\operatorname{\mathrm{dom}}(p^*)\setminus \operatorname{\mathrm{dom}}(p))\end{align*} $$

and $|\{x\in \operatorname {\mathrm {dom}}(p)\mid p(x)\neq q(x)\}|\leq \kappa $ , $q\restriction \operatorname {\mathrm {dom}}(p)=q_r$ for some r, therefore $q\geq t_{r+1}\in D$ . Now we define inductively ${\langle } p^{(r)}\mid r<\kappa ^+{\rangle }$ , $p^{(0)}=p$ at limit we take union, and at successor step we take $p^{(r+1)}=(p^{(r)})^*$ . We claim that $p_*:=\cup _{r<\kappa ^+}p^{(r)}\in D^*$ . First note that $\kappa ^+<\kappa _1$ , hence $|p_*|<\kappa _1$ (all the definition is inside $M_U[G_{\kappa }*g_{\kappa }*G_{(\kappa ,\kappa _1)}]$ ). Let q be any condition with $\operatorname {\mathrm {dom}}(q)=\operatorname {\mathrm {dom}}(p^*)$ and denote by

$$ \begin{align*}I=\{x\in \operatorname{\mathrm{dom}}(p_*)\mid q(x)\neq p_*(x)\}\end{align*} $$

and suppose that $|I|\leq \kappa $ . Since $\operatorname {\mathrm {dom}}(p_*)=\cup _{r<\kappa ^+}\operatorname {\mathrm {dom}}(p^{(r)})$ and $\operatorname {\mathrm {dom}}(p^{(r)})$ is $\subseteq $ -increasing, there is $j<\kappa ^+$ such that $I\subseteq \operatorname {\mathrm {dom}}(p^{(j)})$ . The condition $q\restriction I$ is enumerated in the construction of $p^{(j+1)}$ , hence $q\restriction \operatorname {\mathrm {dom}}(p^{(j+1)})\in D$ and since D is open, $q\in D$ . This means that $p_*\in D^*$ .

Finally, by genericity of $g^{\prime }_{\kappa _1}$ , we can find $p\in D^*\cap g^{\prime }_{\kappa _1}$ . By definition, $p^*\in g_{\kappa _1}$ and since $\operatorname {\mathrm {dom}}(p^*)=\operatorname {\mathrm {dom}}(p)$ and $|\{x\mid p(x)\neq p^*(x)\}|\leq \kappa $ it follows that $p^*\in D$ .

Denote by $H=G_\kappa *g_{\kappa }*G_{(\kappa ,\kappa _1)}*g_{\kappa _1}$ , then $j_1^{\prime \prime }G\subseteq H$ . Let

$$ \begin{align*}j_1^*:V[G]\rightarrow M_1[H]\end{align*} $$

be the extended ultrapower and derive the normal ultrafilter over $\kappa $ ,

$$ \begin{align*}U_1:=\{X\subseteq \kappa\mid \kappa\in j^*_1(X)\}\end{align*} $$

then $U\subseteq U_1$ and $j^*_1=j_{U_1}$ . Indeed let $k_1:M_{U_1}\rightarrow M_1[H]$ be the usual factor map $k_1(j_{U_1}(f)(\kappa ))=j^*_1(f)(\kappa )$ . We will prove that $k_1$ is onto and therefore $k_1=id$ . For every $A\in M_1[H]$ , there is a name

such that

. $M_U$ is the ultrapower by U, hence there is $f\in V$ such that

. By elementarity for every $\alpha <\kappa $ , $f(\alpha )$ is a name. In $V[G]$ define $f^*(\alpha )=(f(\alpha ))_G$ , then by elementarity

Denote by $M_1^*=M_1[H]$ and consider $j^*_1(U_1)\in M^*_1$ . Let us now define inside $M^*_1$ an $M_{2}$ -generic filter for

in a similar fashion as H was defined. First we take H to be the generic for

. Note that $M_{2}$ is closed under $\kappa _1$ -sequences with respect to $M_1$ . Therefore, from the $M^*_1$ -point of view,

is $\kappa _1^+$ -closed, and we can construct an $M_{2}[H]$ -generic filter $G_{(\kappa _1,\kappa _2)}*g^{\prime }_{\kappa _2}\in M_1^*$ for it. We change the values of $g^{\prime }_{\kappa _2}$ a bit differently from the way we changed the values of $g^{\prime }_{\kappa _1}$ . If $\alpha <\kappa _1^+$ is of the form $j_1(\beta )$ let $f_{\kappa _2,k(\alpha )}(\kappa _1)=1$ (to guarantee that $A_\alpha $ ’s belong to the ultrafilter generated by $\kappa _1$ ) and if $\alpha \in \kappa _1^+\setminus j_1^{\prime \prime }\kappa ^+$ letFootnote 2 $f_{\kappa _2,k(\alpha )}(\kappa _1)=0$ . Also, we would like that $f_{\kappa _2,\kappa _1}(0)=\kappa $ . Formally, for every $p\in \operatorname {\mathrm {Cohen}}(\kappa _2,\kappa _2^+)^{M_2[H*G_{(\kappa _1,\kappa _2)}]}$ , define $p^*$ to be a function with $\operatorname {\mathrm {dom}}(p)=\operatorname {\mathrm {dom}}(p^*)$ and for every ${\langle }\gamma ,\alpha {\rangle }\in \operatorname {\mathrm {dom}}(p^*)$ ,

$$ \begin{align*}p^*({\langle}\gamma,\alpha{\rangle})=\begin{cases} f_{\kappa_1,\beta}(\gamma),& \gamma<\kappa_1\wedge \alpha=k(\beta), \\ 1, & \gamma=\kappa_1\wedge \alpha=k(j_1(\beta)),\\ 0, & \gamma=\kappa_1\wedge \alpha=k(\beta),\beta\notin j_1^{\prime\prime}\kappa^+,\\ \kappa, & \gamma=0\wedge \alpha=\kappa_1,\\ p({\langle}\gamma,\alpha{\rangle}), & \text{else}.\end{cases}\end{align*} $$

Denote by $g_{\kappa _2}=\{p^*\mid p\in g^{\prime }_{\kappa _2}\}\in V[G]$ the resulting filter. It is important that for each $p\in g^{\prime }_2$ , the set

$$ \begin{align*}X_1:=j_{2}^{\prime\prime}\kappa^+\cap \operatorname{\mathrm{dom}}(f)_{\leq\kappa_1}=\{j_2(\alpha)\mid \exists {\langle}\gamma,j_2(\alpha){\rangle}\in\operatorname{\mathrm{dom}}(f), \gamma\leq\kappa_1\}\end{align*} $$

has size at most $\kappa $ . This ensured that $X_1\in M_1^*$ . Also, $k"\kappa _1^+$ is unbounded in $\kappa _2^+$ and conditions in $\operatorname {\mathrm {Cohen}}(\kappa _2,\kappa _2^+)^{M_2[H*G_{(\kappa _1,\kappa _2)}]}$ have $M_{2}[H*G_{(\kappa _1,\kappa _2)}]$ -cardinality less than $\kappa _2$ , which guarantees that for each $p\in \operatorname {\mathrm {Cohen}}(\kappa _2,\kappa _2^+)$ ,

$$ \begin{align*}X_2:= k"\kappa_1^+\cap \operatorname{\mathrm{dom}}(p)_{\leq\kappa_1}\end{align*} $$

has size at most $\kappa _1$ . Note that $p^*$ is definable in $M_1^*$ from the parameters $p,X_1,X_2\in M_1^*$ , and $p^*$ differs from p at most on $\kappa _1$ -many values. By the closure of $M_{2}[H*G_{(\kappa _1,\kappa _2)}]$ to $\kappa _1$ -sequences from $M_1^*$ ,

$$ \begin{align*}p^*\in M_{2}[H*G_{(\kappa_1,\kappa_2)}]\text{ and }g_{\kappa_2}\subseteq \operatorname{\mathrm{Cohen}}(\kappa_2,\kappa_2^+)^{M_{2}[H*G_{(\kappa_1,\kappa_2)}]}.\end{align*} $$

The genericity argument of Lemma 2.7 extends to the models $M_1$ and $M_{2}[H*G_{(\kappa _1,\kappa _2)}]$ , hence $g_{\kappa _2}$ is $M_{2}[H*G_{(\kappa _1,\kappa _2)}]$ -generic. Denote by $M_2^*=M_{2}[H*G_{(\kappa _1,\kappa _2)}*g_{\kappa _2}]$ . It follows that k can be extended (in $V[G]$ ) to $k^*$ and also $j_{2}$ to $j^*_2=k^*\circ j^*_1:V[G]\rightarrow M^*_2$ . Finally, let

$$ \begin{align*}W:=\{X\in P^{V[G]}(\kappa)\mid \kappa_1\in j^*_2(X)\}\in V[G].\end{align*} $$

Let us prove that W witnesses the theorem:

Claim 2.8. W is a $\kappa $ -complete ultrafilter over $\kappa $ such that:

  1. (1) $j_W=j^*_2$ and $[id]_W=\kappa _1$ .

  2. (2) $Cub_\kappa \subseteq W$ .

  3. (3) $\{\alpha <\kappa \mid cf(\alpha )=\alpha \}\in W$ .

  4. (4) ${\langle } A_\alpha \mid \alpha <\kappa ^+{\rangle }$ is a strong witness for the failure of the Galvin property.

Proof To see (1), let us denote by $j_W:V[G]\rightarrow M_W$ the ultrapower embedding by W and $k_W:M_W\rightarrow M^*_2$ defined by $k_W([f]_W)=j^*_2(f)(\kappa _1)$ the factor map satisfying $k_W\circ j_W=j^*_2$ . Let us argue that $k_W$ is onto and therefore $k_W=id$ and $[id]_W=\kappa _1$ . Indeed, let $A\in M_2^*$ then there is

such that

. Since $j_2=j_{U^2}$ there is $h\in V$ such that

. Note that $\kappa =j^*_2(f_\kappa )_{\kappa _1}(0)$ , hence define in $V[G]$ , $h^*(\alpha )=(h(f_{\kappa ,\alpha }(0),\alpha ))_G$ . We have that

To see $(2)$ , for every club $C\in Cub_\kappa $ , $j^*_2(C)$ is closed and $j^*_1(C)$ is unbounded in $\kappa _1$ . Since $crit(k^*)=\kappa _1$ and $j^*_{2}(C)=k^*(j^*_1(C))$ it follows that $j^*_2(C)\cap \kappa _1=j^*_1(C)$ , hence $j^*_2(C)\cap \kappa _1$ is unbounded in $\kappa _1$ which implies that $\kappa _1\in j^*_2(C)$ .

For $(3)$ , since $M_2^*\models cf(\kappa _1)=\kappa _1$ , it follows that $\{\alpha \mid cf(\alpha )=\alpha \}\in W$ . Finally, for every $\alpha <\kappa ^+$ ,

$$ \begin{align*}j^*_2(A_\alpha)=\{\beta<\kappa_2\mid f_{\kappa_2,j_2(\alpha)}(\beta)=1\}.\end{align*} $$

Since $j_2(\alpha )=k(j_1(\alpha ))$ , by the definition of $g_{\kappa _2}$ , $f_{\kappa _2,j_2(\alpha )}(\kappa _1)=1$ , thus $\kappa _1\in j^*_2(A_\alpha )$ , and by definition of W, $A_\alpha \in W$ .

For (3), let $\{A_{\alpha _i}\mid i<\kappa \}$ be any subfamily of length $\kappa $ and $\kappa \leq \eta <[id]_W=\kappa _1$ . Denote

$$ \begin{align*}j_2^*({\langle} A_{\alpha_i}\mid i<\kappa{\rangle})={\langle} A^{(2)}_{\alpha^{(2)}_i}\mid i<\kappa_2{\rangle}, \ j_1^*({\langle} A_{\alpha_i}\mid i<\kappa{\rangle})={\langle} A^{(1)}_{\alpha^{(1)}_i}\mid i<\kappa_1{\rangle}.\end{align*} $$

Since $\kappa \leq \eta <\kappa _1$ , then $\eta \notin j_1^{\prime \prime }\kappa ^+$ and thus $\alpha ^{(1)}_\eta \notin j_1^{\prime \prime }\kappa ^+$ . Also, $k(\alpha ^{(1)}_\eta )=\alpha ^{(2)}_{k(\eta )}=\alpha ^{(2)}_\eta $ . Hence by definition, $f_{\kappa _2,\alpha ^{(2)}_\eta }(\kappa _1)=0$ , hence $\kappa _1\notin A^{\prime }_{\alpha ^{(2)}_\eta }.$

2.2 Adding $\kappa ^+$ -Cohen subsets to $\kappa $ by Prikry forcing

In this section we will construct a model in which there is a $\kappa $ -complete ultrafilter W such that forcing with $\operatorname {\mathrm {Prikry}}(W)$ adds a generic for $\operatorname {\mathrm {Cohen}}(\kappa ,\kappa ^+)$ . Let us first observe that such an ultrafilter must fail to satisfy the Galvin property:

Proposition 2.9. If $Gal(U,\kappa ,\kappa ^+)$ holds then $\operatorname {\mathrm {Prikry}}(U)$ does not add a V-generic filter for $\operatorname {\mathrm {Cohen}}(\kappa ,\kappa ^+)$ .

Proof Suppose that $Gal(U,\kappa ,\kappa ^+)$ holds and let $G\subseteq \operatorname {\mathrm {Prikry}}(U)$ be V-generic. By [Reference Gitik18, Proposition 1.3] every set $A\in V[G]$ of size $\kappa ^+$ contains a set $B\in V$ of cardinality $\kappa $ . Toward a contradiction suppose that $H\in V[G]$ is a V-generic filter for $\operatorname {\mathrm {Cohen}}(\kappa ,\kappa ^+)$ . Code $H:\kappa \times \kappa ^+\rightarrow 2$ as $X\subseteq \kappa ^+$ , just pick a bijection $\phi $ from $\kappa ^+$ to $\kappa ^+\times \kappa $ , and let $X=\{\alpha <\kappa ^+\mid H(\phi (\alpha ))=1\}$ . The set X does not contain an old subset of cardinality $\kappa $ ; this is a contradiction. To see this, let $Y\in V$ such that $|Y|=\kappa $ , proceed with a density argument: any condition $p\in \operatorname {\mathrm {Cohen}}(\kappa ,\kappa ^+)$ has size $<\kappa $ and therefore can be extended to a condition $p'$ such that for some $y\in Y$ , $\phi (y)\in \operatorname {\mathrm {dom}}(p')$ and $p'(\phi (y))=0$ .

Hence the failure of the Galvin property is necessary.

Theorem 2.10. Assume $GCH$ and that $\kappa $ is a measurable cardinal in V. Then there is a cofinality preserving forcing extension $V^*$ in which $GCH$ still holds, and there is a $\kappa $ -complete ultrafilter $U^*\in V^*$ over $\kappa $ such that forcing with Prikry forcing $Pikry(U^*)$ introduces a $V^*$ -generic filter for $Cohen^{V^*}(\kappa ,\kappa ^+)$ .

Proof The model $V^*$ is obtained by iterating with Easton support the lottery sum of Cohen forcings for adding $\alpha ^+$ -Cohen functions ${\langle } f_{\alpha \gamma }\mid \gamma <\alpha ^+{\rangle }$ over $\alpha $ , and Cohen ${}^2$ for adding two blocks of $\alpha ^+$ -Cohen functions

$$ \begin{align*}{\langle} f_{\alpha \gamma}\mid \gamma<\alpha^+{\rangle},{\langle} h_{\alpha\gamma}\mid \gamma<\alpha^+{\rangle}.\end{align*} $$

More specifically, let

denotes the Easton support iteration, such that for each $\alpha <\kappa $ ,

is the trivial forcing unless $\alpha $ is inaccessible in which case

is a ${\mathcal P}_\alpha $ -name for the lottery sum

$$ \begin{align*}\operatorname{\mathrm{LOTT}}(\operatorname{\mathrm{Cohen}}(\alpha,\alpha^+),\operatorname{\mathrm{Cohen}}(\alpha,\alpha^+)\times \operatorname{\mathrm{Cohen}}(\alpha,\alpha^+)).\end{align*} $$

At $\kappa $ itself we let

. Let $G_\kappa *F_\kappa $ be a V-generic subset of

and let $V^*=V[G_\kappa *F_\kappa ]$ . We denote by $F_\alpha :={\langle } f_{\alpha \gamma }\mid \gamma <\alpha ^+{\rangle }$ the generic Cohen function if $\operatorname {\mathrm {Cohen}}(\alpha ,\alpha ^+)$ was forced in $G_\kappa $ and by

$$ \begin{align*}F_\alpha:={\langle} f_{\alpha \gamma}\mid \gamma<\alpha^+{\rangle},\ H_\alpha:={\langle} h_{\alpha,\gamma}\mid \gamma<\alpha^+{\rangle}\end{align*} $$

if $\operatorname {\mathrm {Cohen}}(\alpha ,\alpha ^+)\times \operatorname {\mathrm {Cohen}}(\alpha ,\alpha ^+)$ was.

Let $U\in V$ be a normal ultrafilter, $j_1:=j_U:V\rightarrow M_U$ the corresponding elementary embedding, $\kappa _1=j_1(\kappa )$ , $k:=j_{j_1(U)}:M_U\rightarrow M_{U^2}$ , $j_2=k\circ j_1$ , and $\kappa _2=j_2(\kappa )$ . Let us extend $j_1, k,j_2$ in $V[G_{\kappa }*F_\kappa ]$ :

We first extend $j_{1}: V \to M_{{U}}$ to $j_1^{*}: V[G_\kappa *F_\kappa ]\to M_{U}[G_{\kappa _1}*F_{\kappa _1}]$ . Do this by taking first $G_{\kappa _1}\cap P_{\kappa }=G_\kappa $ , at $\kappa $ we force with the lottery sum so we can choose to force only one block of Cohens and take $F_\kappa $ as a generic. Then defining a master condition sequence, using the closure of the forcing above $\kappa $ in $M_{U}$ exploiting $GCH$ to ensure that there are only $\kappa ^+$ -many dense sets to meet. This defines $G_{\kappa _1}$ . As for $F_{\kappa _1}$ , we first find an $M_U[G_{\kappa _1}]$ -generic $F^{\prime }_{\kappa _1}\times H^{\prime }_{\kappa _1}\in V[G_{\kappa }*F_\kappa ]$ again using $GCH$ , closure of $M_U[G_{\kappa _1}]$ under $\kappa $ -sequences and the closure of the forcing $(\operatorname {\mathrm {Cohen}}(\kappa _1,\kappa _1^+)^2)^{M_U[G_{\kappa _1}]}$ . Let us alter some values of $F^{\prime }_{\kappa _1}$ and $H^{\prime }_{\kappa _1}$ to define $F_{\kappa _1}={\langle } f_{\kappa _1,\gamma }\mid \gamma <\kappa _1^+{\rangle }$ and $H_{\kappa _1}={\langle } h_{\kappa _1,\gamma }\mid \gamma <\kappa _1^+{\rangle }$ such that for every $\alpha <\kappa _1^+$ :

  1. (1) $f_{\kappa _1,j_1(\alpha )}\restriction \kappa =h_{\kappa _1,j_1(\alpha )}\restriction \kappa =f_{\kappa ,\alpha }$ .

  2. (2) $f_{\kappa _1,j_1(\alpha )}(\kappa )=\alpha $ .

Formally, we change every pair of partial functions $p={\langle } p_0,p_1{\rangle } \in F^{\prime }_{\kappa _1}\times H^{\prime }_{\kappa _1}$ to the pair of partial functions $p_*={\langle } p_0^*,p_1^*{\rangle }$ such that $\operatorname {\mathrm {dom}}(p_0^*)=\operatorname {\mathrm {dom}}(p_0)$ , $\operatorname {\mathrm {dom}}(p_1^*)=\operatorname {\mathrm {dom}}(p_1)$ and for every ${\langle } \alpha ,\delta {\rangle }\in \operatorname {\mathrm {dom}}(p_0)$ :

$$ \begin{align*}p^*_0({\langle}\alpha,\delta{\rangle})=\begin{cases} f_{\kappa,\alpha_0}(\delta), &\exists \alpha_0<\kappa^+.\alpha=j_1(\alpha_0)\text{ and }\delta<\kappa,\\ \alpha_0, & \exists \alpha_0<\kappa^+.\alpha=j_1(\alpha_0)\text{ and }\delta=\kappa,\\ p_0({\langle}\alpha,\delta{\rangle}), & \text{else}.\end{cases}\end{align*} $$
$$ \begin{align*}p^*_1({\langle}\alpha,\delta{\rangle})=\begin{cases} f_{\kappa,\alpha_0}(\delta), &\exists \alpha_0<\kappa^+.\alpha=j_1(\alpha_0)\text{ and }\delta<\kappa,\\ p_1({\langle}\alpha,\delta{\rangle}), & \text{else}.\end{cases}\end{align*} $$

Note that for every $p_0,p_1\subseteq \operatorname {\mathrm {Cohen}}(\kappa _1,\kappa _1^+)^{M_U[G_{\kappa _1}]}$ we only change $\kappa $ -many values as $M_U[G_{\kappa _1}]\models |\operatorname {\mathrm {dom}}(p_0)|,|\operatorname {\mathrm {dom}}(p_1)|<\kappa _1$ , hence

$$ \begin{align*}|j_1^{\prime\prime}\kappa^+\cap\{\alpha\mid\exists\delta.{\langle}\alpha,\delta{\rangle}\in \operatorname{\mathrm{dom}}(p_0)\}|\leq\kappa\end{align*} $$

since $j_1(\kappa ^+)=\bigcup j_1^{\prime \prime }\kappa ^+$ , the same holds for $p_1$ . It follows that

$$ \begin{align*}p^*\in (\operatorname{\mathrm{Cohen}}(\kappa_1,\kappa_1^+)^2)^{M_U[G_{\kappa_1}]}.\end{align*} $$

Changing less than $\kappa _1$ -many values of a generic for $\operatorname {\mathrm {Cohen}}(\kappa _1,\kappa _1^+)^2$ does not impact the genericity. Hence $F_{\kappa _1}\times H_{\kappa _1}:=\{p^*\mid p\in F^{\prime }_{\kappa _1}\times H^{\prime }_{\kappa _1}\}\in V[G_{\kappa }*F_\kappa ]$ is still $M_U[G_{\kappa _1}]$ -generic.

Since at $\kappa $ we only force $\operatorname {\mathrm {Cohen}}(\kappa ,\kappa ^+)$ , in order to extend $j_1$ we only need a generic for $\operatorname {\mathrm {Cohen}}(\kappa _1,\kappa _1^+)$ in the $M_U$ -side. We constructed $F_{\kappa _1}$ so that $j_1^{\prime \prime }F_\kappa \subseteq F_{\kappa _1}$ , hence $j_1^{\prime \prime }G_\kappa *F_\kappa \subseteq G_{\kappa _1}*F_{\kappa _1}$ ( $H_{\kappa _1}$ will be used later). Thus in $V[G_{\kappa }*F_{\kappa }]$ , we have extended $j_1\subseteq j_1^*:V[G_{\kappa }*F_{\kappa }]\rightarrow M_U[G_{\kappa _1}*F_{\kappa _1}]$ . Let us note that $j_1^*$ is actually the elementary embedding derived from the normal measure $U\subseteq U^0:=\{X\in P^{V[G_\kappa *F_\kappa ]}(\kappa )\mid \kappa \in j_1^*(X)\}$ :

Clearly the function $k_0:M_{U^0}\rightarrow M_U[G_{\kappa _1}*F_{\kappa _1}]$ defined by $k_0([f]_{U^0})=j^*_1(f)(\kappa )$ is elementary. To see the $k_0=id$ let us prove that $k_0$ is onto. Fix

and let $f\in V$ be such that

and define in $V[G_{\kappa }*F_\kappa ]$ the function $f^*(x)=(f(f_{\kappa ,\kappa }(x)))_{G_{\kappa }*F_{\kappa }}$ . Then

$$ \begin{align*}k_0(j_{U^0}(f^*)(\kappa))=j^*_1(f^*)(\kappa)=(j^*_1(f)(j^*_1(f_{\kappa,\kappa})(\kappa)))_{G_{\kappa_1}*F_{\kappa_1}}=\end{align*} $$

Recall that we have constructed the function $H_{\kappa _1}\in V[G_{\kappa }*F_{\kappa }]$ such that $F_{\kappa _1}\times H_{\kappa _1}$ is $M_U[G_{\kappa _1}]$ -generic for $\operatorname {\mathrm {Cohen}}(\kappa _1,\kappa _1^+)^2$ . Now we wish to extend $k:M_U\rightarrow M_{U^2}$ to $k^*:M_U[G_{\kappa _1}*F_{\kappa _1}]\rightarrow M_{U^2}[G_{\kappa _2}*F_{\kappa _2}]$ in $V[G_{\kappa }*F_{\kappa }]$ . We do this by taking $G_{\kappa _2}\cap \kappa _1= G_{\kappa _1}$ , at $\kappa _1$ we force $\operatorname {\mathrm {Cohen}}(\kappa _1,\kappa _1^+)\times \operatorname {\mathrm {Cohen}}(\kappa _1,\kappa _1^+)$ putting the generic $F_{\kappa _1}\times H_{\kappa _1}$ , then exploiting the closure and $GCH$ to complete to a generic $G_{\kappa _2}*F^{\prime }_{\kappa _2}\in V[G_\kappa *F_\kappa ]$ . Finally, we wish to modify some values of $F^{\prime }_{\kappa _2}$ to a generic $F_{\kappa _2}={\langle } f_{\kappa _2,\gamma }\mid \gamma <\kappa _2^+{\rangle }$ so that for every $\alpha <\kappa _1^+$ :

  1. (1) $f_{\kappa _2,k(\alpha )}\restriction \kappa _1=f_{\kappa _1,\alpha }$ .

  2. (2) For $\alpha \in j_1^{\prime \prime }\kappa ^+$ , $f_{\kappa _2,k(\alpha )}(\kappa _1)=1$ .

  3. (3) For $\alpha \in \kappa _1^+\setminus j_1^{\prime \prime }\kappa ^+$ , $f_{\kappa _2,k(\alpha )}(\kappa _1)=0$ .

  4. (4) $f_{\kappa _2,\kappa _1}(\kappa _1)=\kappa $ .

Again, this is possible since we do not change too many values of $F^{\prime }_{\kappa _2}$ . At this point, let us emphasize that we do not use $H_{\kappa _1}$ in the generic we have in the $M_U$ -side Footnote 3 . The generic $H_{\kappa _1}$ is used in the construction of the generic on the $M_{U^2}$ -side where we can choose (due to the lottery sum) to force at $\kappa _1$ two copies of $\operatorname {\mathrm {Cohen}}(\kappa _1,\kappa _1^+)$ , of course, that at $\kappa _2=j_{2}(\kappa )$ we are still obligated to force one copy of $\operatorname {\mathrm {Cohen}}(\kappa _2\kappa _2^+)$ which contains the point-wise image of $F_{\kappa _1}$ under the factor map k.

Hence we extended in $V[G_{\kappa }*F_\kappa ]$ , $k\subseteq k^*:M_U[G_{\kappa _1}*F_{\kappa _1}]\rightarrow M_{U^2}[G_{\kappa _2}*F_{\kappa _2}]$ .

Let $j_2^*=k^*\circ j_1^*$ , $V^*=V[G_{\kappa }*F_\kappa ]$ , $M_1^*=M_U[G_{\kappa _1}*F_{\kappa _1}]$ and $M_2^*=M_{U^2}[G_{\kappa _2}*F_{\kappa _2}]$ .

In $V^*$ , define

$$ \begin{align*}U^*=\{X\subseteq\kappa\mid \kappa\in j_2^*(U)\},\end{align*} $$
$$ \begin{align*}W=\{X\subseteq \kappa\mid \kappa_1\in j^*_2(X)\},\end{align*} $$

and for every $\alpha <\kappa ^+$ ,

$$ \begin{align*}A_\alpha=\{\nu<\kappa\mid f_{\kappa,\alpha}(\nu)=1\}.\end{align*} $$

Then as in Claim 2.8, we have that W is a $\kappa $ -complete ultrafilter over $\kappa $ such that:

  1. (1) $j_1^*=j_U^*$ , $j^*_2=j_W$ and $[id]_W=\kappa _1$ .

  2. (2) ${\langle } A_\alpha \mid \alpha <\kappa ^+{\rangle }$ is a strong witness for W being non-Galvin.

  3. (3) $Cub_\kappa \subseteq W$ .

  4. (4) $L_0=\{\alpha <\kappa \mid \operatorname {\mathrm {Cohen}}(\alpha ,\alpha ^+)\times \operatorname {\mathrm {Cohen}}(\alpha ,\alpha ^+)\text { was forced in }G_\kappa \}\in W$ .

Also, recall that $j_2:V\rightarrow M_2$ is also the ultrapower by $U\times U$ under the identification(isomorphism):

$$ \begin{align*}j_{U^2}(f)(\kappa,\kappa_1)=j_{2,1}(j_1(\nu\mapsto f(\nu,*))(\kappa))(\kappa_1).\end{align*} $$

Clearly, the projections $\pi _{1},\pi _2:\kappa \times \kappa \rightarrow \kappa $ on the first and second coordinates (resp. Rudin–Keisler) project $U^2$ on U. Also, $W\cap V=U^*\cap V=U$ and $U^*\leq _{R-K} W$ and the projection map is denoted by $\nu \mapsto \pi _{nor}(\nu )$ .Footnote 4

Let us prove that W witnesses the theorem:

Theorem 2.11. Let $H\subseteq \operatorname {\mathrm {Prikry}}(W)$ be a $V^*$ -generic filter. There is $G^*\in V^*[H]$ which is $V^*$ -generic for $Cohen(\kappa ,\kappa ^+)^{V^*}$ .

Proof of Theorem 2.11

Let ${\langle } c_n \mid n<\omega {\rangle } $ be the W-Prikry sequence corresponding to H. Suppose without loss of generality that for every $n<\omega $ , $c_n\in L_0$ , this will hold from a certain point and the proof can be adjusted in a straightforward way. This guarantees that the generic $H_{c_n}={\langle } h_{c_n,\gamma }\mid \gamma <\alpha ^+{\rangle }$ for the second component of the generic we have in $G_\kappa $ for $\operatorname {\mathrm {Cohen}}(c_n,c_n^+)\times \operatorname {\mathrm {Cohen}}(c_n,c_n^+)$ is defined for every $n<\omega $ . The functions $h_{c_n,\gamma }$ will be used below to define the Cohen generic functions.

Define, for every $n<\omega $ , the set

$$ \begin{align*}Z_n=\{\alpha<\kappa^+\mid \{c_m \mid n\leq m<\omega\} \subseteq A_\alpha \text{ and } n \text{ is least possible}\}.\end{align*} $$

For every $\alpha <\kappa ^+$ , let $n_\alpha $ be the unique n such that $\alpha \in Z_n$ . Let $\alpha <\kappa ^+$ , and define $f^*_\alpha :\kappa \to \kappa $ as follows:

Fix a sequence ${\langle } s_\alpha \mid \alpha <\kappa ^+ {\rangle }\in V^*$ of canonical functions in $\prod _{\nu <\kappa }\nu ^+$ :

$$ \begin{align*}f^*_\alpha\restriction c_{n_\alpha}= h_{c_{n_\alpha} s_\alpha(c_{n_\alpha})},\end{align*} $$
$$ \begin{align*}f^*_\alpha\restriction [c_{m-1}, c_m)= h_{ c_m, s_\alpha(c_m)}\restriction [c_{m-1}, c_m), \text{for }m, n_\alpha< m<\omega.\end{align*} $$

Let us argue that $F={\langle } f^*_\alpha \mid \alpha <\kappa ^+{\rangle } $ induces a $\operatorname {\mathrm {Cohen}}(\kappa ,\kappa ^+)^{V^*}$ generic filter over $V^*$ .

Claim 2.12. Let $G^*=\{p\in \operatorname {\mathrm {Cohen}}(\kappa ,\kappa ^+)^{V^*}\mid p\subseteq F\}$ , then $G^*$ is a $V^*$ -generic filter.

Let $\mathcal {A}\in V^*$ be a maximal antichain in the forcing $\operatorname {\mathrm {Cohen}}(\kappa , \kappa ^+)^{V^*}$ . Note that since $\operatorname {\mathrm {Cohen}}(\kappa ,\kappa ^{+})^{V^*}$ is $\kappa $ -closed then

$$ \begin{align*}Cohen(\kappa,\kappa^{+})^{V[G_\kappa]}=Cohen(\kappa,\kappa^{+})^{V^*}\!\!.\end{align*} $$

By $\kappa ^+$ -cc of the forcing $\mathcal {P}_{\kappa +1}$ , there is $Y\subseteq \kappa ^{+}$ , $Y\in V$ such that $|Y|=\kappa $ and $\mathcal {A}\subseteq \operatorname {\mathrm {Cohen}}(\kappa ,Y)^{V^*}$ . Also, since $|\mathcal {A}|=\kappa $ , $\mathcal {A}\in V[G_\kappa *F_\kappa ]$ , there is $Z\subseteq \kappa ^{+}$ such that $|Z|=\kappa $ such that $\mathcal {A}\in V[G_\kappa *F_\kappa \restriction Z]$ . Without loss of generality assume that $Z=Y\in V$ (Otherwise just take the union). Let $V\ni \phi :\kappa \rightarrow Y$ be a bijection.

Claim 2.13. There is an $\in $ -increasing continuous chain ${\langle } N_\beta \mid \beta <\kappa {\rangle }$ of elementary submodels of $H_\chi $ such that:

  1. (1) $|N_\beta |<\kappa $ .

  2. (2) $G_\kappa ,F_\kappa ,\mathcal {A},\phi ,{\langle } s_\alpha \mid \alpha <\kappa ^+{\rangle }\in N_0$ .

  3. (3) $N_\beta \cap \kappa =\gamma _\beta $ is a cardinal $<\kappa $ , $\gamma _{\beta +1}$ is regular.

  4. (4) For every $\rho ,\delta \in \phi "\gamma _\beta .\rho <\delta \rightarrow \forall \gamma _\beta \leq \mu <\kappa , s_\rho (\mu )<s_\delta (\mu )$ .

  5. (5) If $\gamma _\beta $ is regular, then $N_\beta ^{<\gamma _\beta }\subseteq N_\beta $ . In particular $\operatorname {\mathrm {Cohen}}(\gamma _\beta ,\phi "\gamma _\beta )=\operatorname {\mathrm {Cohen}}(\kappa ,Y)\cap N_\beta $ .

Proof of Claim 2.13

Let us construct such a sequence inductively. Note that $(4)$ follows from elementarity and $(2)$ . Requirements $(1)$ $(5)$ are preserved at limit stages due to continuity. At successor stages, suppose we have constructed $N_\beta $ , find an elementary submodel $N^0_{\beta +1}$ such that $N_\beta \subseteq N^0_{\beta +1}, \ {\langle } N_\alpha \mid \alpha <\beta {\rangle }\in N^0_{\beta +1}$ , then we construct an auxiliary $\in $ -increasing and continuous chain of elementary submodels ${\langle } N^\alpha _{\beta +1}\mid \alpha <\kappa {\rangle }$ as follows: $N^0_{\beta +1}$ is already defined. At limits we take the union and at successor let us take care of requirements 3 and 5. Let $\gamma ^{\prime }_\alpha =\sup (N^{\alpha }_{\beta +1}\cap \kappa )<\kappa $ . Let $N^{\alpha +1}_{\beta +1}$ be an elementary submodel such that $N^{\alpha }_\beta ,^{<\gamma ^{\prime }_\alpha },\subseteq N^{\alpha +1}_{\beta +1}$ and $|N^{\alpha +1}_{\beta +1}|<\kappa $ . Note that the sets

$$ \begin{align*}C_1=\{\alpha<\kappa\mid N^{\alpha}_{\beta+1}\cap\kappa=\gamma^{\prime}_\alpha\in\kappa\},\end{align*} $$
$$ \begin{align*}C_2=\{\alpha\in C_1\mid \text{if }\gamma_\alpha\text{ is regular then } N_{\alpha}^{<\gamma_\alpha}\subseteq N_\alpha\}\end{align*} $$

are clubs and also $\bar C=C_1\cap C_2$ is. It follows that $\{\gamma ^{\prime }_\alpha \mid \alpha \in \bar C\}$ is a club and since $\kappa $ is measurable, there is a $\alpha ^*\in \bar C$ limit such that $\gamma ^{\prime }_{\alpha ^*}$ is regular. Let $N_{\beta +1}=N^{\alpha ^*}_{\beta +1}$ , to conclude $2$ since $\gamma _{\beta +1}=\gamma ^{\prime }_{\alpha ^*}$ is regular.

Set

$$ \begin{align*}C=\{ \beta<\kappa \mid \gamma_\beta=\beta \}.\end{align*} $$

This is club in $\kappa $ since the sequence $\gamma _\beta $ is continuous and since the set $\{\beta \mid \gamma _\beta =\beta \}$ is a club.

Claim 2.14. Let

$$ \begin{align*}E:=\{\beta<\kappa\mid\forall\gamma\in\phi"\beta.\exists\ \delta<\beta^+.f_{\kappa,\gamma}\restriction\beta=f_{\beta,\delta}\}.\end{align*} $$

Then $E\in W$ .

Proof of Claim 2.14

By construction, for every $\alpha <\kappa _1^{+}$ , $f_{\kappa _2,k(\alpha )}\restriction \kappa _1=f_{\kappa _1,\alpha }$ and therefore for every $\alpha \in j^*_2(\phi )"\kappa _1$ , there is $\nu <\kappa _1^+$ such that $\alpha =k^*(j_1^*(\phi ))(\nu )=k^*(j_1^*(\phi )(\nu ))$ and $j_1^*(\phi )(\nu )<\kappa _1^{+}$ . Hence $f_{\kappa _2,\alpha }\restriction \kappa _1=f_{\kappa _1,\beta }$ for some $\beta <\kappa _1^{+}$ . Reflecting this we obtain the set $E\in W$ .

To see that $G^*\cap \mathcal {A}\neq \emptyset $ , we will need to catch a piece of $\mathcal {A}$ in the elementary submodels constructed and pick the Prikry points in the club C prepared:

Claim 2.15. For every $\nu _0\in C\cap E$ , there is $d=d^{\nu _0}\in N_{\nu _0}\cap \mathcal {A}$ such that d is extended by ${\langle } h_{\nu _0,s_{\tau }(\nu _0)}\mid \tau \in \phi "\nu _0{\rangle }$ .

Proof of Claim 2.15

Fix any $\nu _0\in C\cap E$ . Consider the transitive collapse of $\pi :N_{\nu _0}\rightarrow N_{\nu _0}^*$ . Then the critical point of $\pi ^{-1}:N_{\nu _0}^*\rightarrow N_{\nu _0}$ is $\nu _0$ and $\pi ^{-1}(\nu _0)=\kappa $ . Denote by $\overline {F}_\kappa =\pi (F_\kappa ), \overline {\phi }=\pi (\phi )$ . Denote $\overline {F}_\kappa ={\langle } \overline {f}_{\kappa ,\gamma }\mid \gamma < \pi (\kappa ^+){\rangle }$ . For every $\gamma \in \overline {\phi }"{\nu _0}$ , there is some $\delta <{\nu _0}$ such that

$$ \begin{align*}\gamma=\pi(\phi)(\delta)=\pi(\phi(\delta))\text{ and }\overline{f}_{\kappa,\gamma}=\pi(f_{\kappa,\phi(\delta)}).\end{align*} $$

Moreover, since ${\nu _0}\in E$ , $f_{\kappa ,\phi (\delta )}\restriction {\nu _0}=f_{{\nu _0},\rho }$ for some $\rho <{\nu _0}^{+}$ and therefore $\overline {f}_{\kappa ,\gamma }=f_{{\nu _0},\rho }$ . Recall that

, hence

. We conclude that for some subset $Z\subseteq {\nu _0}^{+}$ ,

Since ${\nu _0}\in L_0$ , in $V[G_{\kappa }*F_\kappa ]$ we also have $H_{\nu _0}={\langle } h_{\nu _0,\alpha }\mid \alpha <\nu _0^+{\rangle }$ which are mutually Cohen-generic over $V[G_{\nu _0}*F_{\nu _0}\restriction Z]$ .

By construction, $\forall \tau _1<\tau _2\in \phi "\nu _0$ , $s_{\tau _1}(\nu _0)<s_{\tau _2}(\nu _0)$ , hence ${\langle } h_{\nu _0,s_\tau (\nu _0)}\mid \tau \in \phi "\nu _0{\rangle }$ are Cohen functions over $\nu _0$ which are distinct mutually $V[G_{\nu _0}*F_{\nu _0}\restriction Z]$ -generic. Also, $\overline {\mathcal {A}}\subseteq \pi (\operatorname {\mathrm {Cohen}}(\kappa ,Y))= \operatorname {\mathrm {Cohen}}(\nu _0,\pi (\phi )"\nu _0)=\operatorname {\mathrm {Cohen}}(\nu _0,\pi "[\phi "\nu _0])$ is a maximal antichain. Since $|\pi "\phi "\nu _0|=\nu _0=|\phi "\nu _0|$ , we can change the enumeration of the functions ${\langle } h_{\nu _0,s_{\tau }(\nu _0)}\mid \tau \in \phi "\nu _0{\rangle }$ to $h^{\prime }_{\pi (\tau )}=h_{\nu _0,s_{\tau }(\nu _0)}$ so that ${\langle } h^{\prime }_{\rho }\mid \rho \in \pi "\phi "\nu _0{\rangle }$ is generic for $\operatorname {\mathrm {Cohen}}(\nu _0,\pi "\phi _0)$ . Thus pick $d_0 \in \overline {\mathcal {A}}$ such that $d_0$ is extended by ${\langle } h^{\prime }_{\rho }\mid \rho \in \pi "\phi "\nu _0{\rangle }$ . It follows that

$$ \begin{align*}d:=\pi^{-1}(d_0)\in \mathcal{A}\cap N_{\nu_0}\end{align*} $$

is a condition with $\operatorname {\mathrm {dom}}(d)=\pi ^{-1}(\operatorname {\mathrm {dom}}(d_0))$ . Since the critical point of $\pi $ is $\nu _0$ , for every ${\langle }\alpha ,\beta {\rangle }\in \operatorname {\mathrm {dom}}(d_0)$ , $\pi ^{-1}({\langle }\alpha ,\beta {\rangle }))={\langle }\alpha ,\pi ^{-1}(\beta ){\rangle }$ , hence

$$ \begin{align*}d({\langle}\alpha,\pi^{-1}(\beta){\rangle})=\pi^{-1}(d_0(\alpha,\beta))=d_0(\alpha,\beta).\end{align*} $$

In particular for every ${\langle }\gamma ,\alpha {\rangle }\in \operatorname {\mathrm {dom}}(d)$ ,

$$ \begin{align*}d(\gamma,\alpha)=d_0(\gamma,\pi(\alpha))=h^{\prime}_{\pi(\alpha)}(\gamma)=h_{\nu_0,s_{\alpha}(\nu_0)}(\gamma).\end{align*} $$

Thus d is extended by ${\langle } h_{\nu _0,s_\tau (\nu _0)}\mid \tau \in \phi "\nu _0 {\rangle }$ .

It suffices to show that any condition in $\operatorname {\mathrm {Prikry}}(W)$ has an extension which forces that $G^*$ meets a member of $\mathcal {A}$ .

Let $p={\langle } {\langle }{\rangle }, B{\rangle }$ be a condition (we assume for simplicity that its finite sequence is empty) and shrink B to $B\cap C\cap E$ . For any $\nu _0\in B\cap C\cap E$ , we split $\phi "\nu _0$ into two sets:

$$ \begin{align*}X^{\nu_0}_0:=\{\tau\in\phi"\nu_0\mid \nu_0\in A_\tau\}\text{ and }X^{\nu_0}_1=\phi"\nu_0\setminus X^{\nu_0}_0.\end{align*} $$

The condition $p_0={\langle } {\langle } \nu _0{\rangle }, B\cap C\cap E\cap X\cap (\bigcap _{\tau \in \phi "\nu _0}A_\tau ){\rangle }$ forces the following:

  1. (1) The Prikry sequence is included in each $A_\tau $ , $\tau \in X^{\nu _0}_0$ , i.e., $n_\tau =0$ .

  2. (2) $n_\tau =1$ , for every $\tau \in X^{\nu _0}_1$ .

In particular, this condition forces some information about the Cohen functions. Namely that:

  1. (1) For $\tau \in X_0^{\nu _0}$ , $f^*_{\tau }\restriction \nu _0=h_{\nu _0,s_\tau (\nu _0)}.$

  2. (2) For $\tau \in X_1^{\nu _0}$ , .

We would like to find a condition in $\mathcal {A}$ which is below these decided parts of the Cohen. By the previous proposition, there is $d\in N_{\nu _0}\cap \operatorname {\mathrm {Cohen}}(\kappa ,Y)=\operatorname {\mathrm {Cohen}}(\nu _0,\phi "\nu _0)$ , which is extended by ${\langle } h_{\nu _0,s_\tau (\nu _0)}\mid \tau \in \phi "\nu _0{\rangle }$ . However, by $(1)$ and $(2)$ we can only ensure that the generic $f^*_\tau $ to extend $d\restriction \nu _0\times X^{\nu _0}_0$ in $X^{\nu _0}_0$ . We are left to extend $d\restriction \nu _0\times X^{\nu _0}_1$ . Let us show that for many $\nu _0$ , $X^{\nu }_0$ is a relatively large subset of $\phi "\nu _0$ :

Claim 2.16. Let

$$ \begin{align*}R=\{\nu<\kappa\mid \forall\alpha\in\phi"\pi_{nor}(\nu), \nu\in A_\alpha\}.\end{align*} $$

Then $R\in W$ .

Proof Clearly, for every $\alpha \in j^*_2(\phi )"\kappa $ , $\alpha =j^*_2(\phi (\gamma ))$ , and $f_{\kappa _2,\alpha }(\kappa _1)=1$ , reflecting this, we can find a W-large set of $\nu $ ’s such that for every $\alpha \in \phi "\pi _{nor}(\nu )$ , $f_{\kappa ,\alpha }(\nu )=1$ . And by definition of $A_\alpha $ , $\nu \in A_\alpha $ .

Denote $B_0:=B\cap C\cap E\cap R$ . In order to extend $d\restriction \nu _0\times X_1$ , we will need to pick $\nu _0$ high enough in $B_0$ , but also the next point $\nu _1\in B_0\setminus \nu _0+1$ in the Prikry sequence such that it will belong to all $A_\tau $ with $\tau \in X_1$ and in addition the relevant Cohen functions over $\nu _1$ extend $d\restriction \nu _0\times X_1$ .

Let us look at $B_0$ more carefully. Let

be its name in V. We fix a condition $m_0\in G_\kappa *F_\kappa $ which forces that if

then the properties of Claims 2.15 and 2.16 hold, namely there is

which is extended by

, and

. Recall that by the construction of $G_{\kappa _2}$ , we have $m_0\in G_{\kappa _2}*F_{\kappa _2}$ . Let $ m_0\leq t\in G_{\kappa _2}*F_{\kappa _2}$ be a condition such that

By the construction of $G_{\kappa _2}*F_{\kappa _2}$ , t has the form:

$$ \begin{align*}t={\langle} t_{<\kappa}, t_{\kappa},t_{(\kappa,\kappa_1)},\underset{t_{\kappa_1}}{\underbrace{{\langle} t_{\kappa_1}^0,t_{\kappa_1}^1{\rangle}}},t_{(\kappa_1,\kappa_2)},t_{\kappa_2}{\rangle}.\end{align*} $$

Since $f_{\kappa _2,j_2(\alpha )}(\kappa _1)=1$ for every $\alpha <\kappa ^+$ , this will hold for every $\alpha \in \phi "\kappa $ as well. Also, recall that $Y\in V$ , hence $\phi \in V$ . Thus $j_2(\phi )\in M_2$ and $j_2(\phi )"\kappa \in M_2$ . Also, for $(t_{\kappa _2})_{G_{\kappa _2}}\in M_2[G_{\kappa _2}]$ ,

$$ \begin{align*}j_2^{\prime\prime}\kappa^+\cap \operatorname{\mathrm{Supp}}((t_{\kappa_2})_{G_{\kappa_2}})\in M_2[G_{\kappa_2}]\end{align*} $$

and $(t_{\kappa _2})_{G_{\kappa _2}}\restriction \kappa \times \{j_2(\alpha )\}\subseteq f_{\kappa ,\alpha }$ . We also fix $\theta <\kappa ^+$ such that $\operatorname {\mathrm {Supp}}((t_{\kappa _2})_{G_{\kappa _2}})\subseteq j_2(\theta )$ , there is such $\theta $ since $j_2^{\prime \prime }\kappa ^+$ is unbounded in $j_2(\kappa ^+)$ . Therefore, we can extend if necessary t such that

$$ \begin{align*}(2a)\ \ t_{<\kappa_2}\Vdash (\kappa\cup\{\kappa_1\})\times j_2(\phi)"\kappa \subseteq \operatorname{\mathrm{dom}}(t_{\kappa_2})\wedge (0,\kappa_1)\in\operatorname{\mathrm{dom}}(t_{\kappa_2})\wedge \operatorname{\mathrm{Supp}}(t_{\kappa_2})\subseteq j_2(\theta),\end{align*} $$
$$ \begin{align*}(2b) \ \ t_{<\kappa_2}\Vdash \ t_{\kappa_2}(\kappa_1,\alpha)=1,\text{ for every }\alpha\in j_2(\phi)"\kappa\text{ and } t_{\kappa_2,\kappa_1}(0)=\kappa,\end{align*} $$

Next consider $t_{\kappa _1}={\langle } t_{\kappa _1}^0,t_{\kappa _1}^1{\rangle }$ ; it is a ${\mathcal P}_{\kappa _1}$ -name for a condition in $F_{\kappa _1}\times H_{\kappa _1}$ . By the construction of the generic $F_{\kappa _1}\times H_{\kappa _1}$ , for every $\alpha <\kappa ^+$ , we made sure that $h_{\kappa _1,j_1(\alpha )}\restriction \kappa =f_{\kappa ,\alpha }$ . Also, $j_1(\phi )"\kappa \in M_2$ . Let

$$ \begin{align*}\mu_1=(j_1\restriction \phi"\kappa)^{-1}\in M_1.\end{align*} $$

Note that for every $\beta <\kappa ^+$ , $j_1(s_\beta )=s_{j_1(\beta )}:\kappa _1\rightarrow \kappa _1$ is the canonical function for $j_1(\beta )$ defined in $M_U$ , hence $j_2(s_\beta )(\kappa _1)=k(s_{j_1(\beta )})(\kappa _1)=j_1(\beta )$ . Hence

$$ \begin{align*}\operatorname{\mathrm{dom}}(\mu_1)=j_1(\phi)"\kappa=\{s_\gamma(\kappa_1)\mid \gamma\in j_2(\phi)"\kappa\}, \ rg(\mu_1)=\phi"\kappa\subseteq\kappa^+.\end{align*} $$

Extend if necessary $t_{<\kappa _1}$ , and assume that

As for the lower part, due to the Easton support, we have

$$ \begin{align*}(4) \ \ t_{<\kappa}\in V_\kappa.\end{align*} $$

Fix functions $r,\Gamma _1$ which represents $t,\mu $ resp. in the ultrapower $M_{U^2}$ , namely $j_2(r)(\kappa ,\kappa _1)=t, \ j_2(\Gamma _1)(\kappa ,\kappa _1)=\mu $ . Without loss of generality, suppose that for every $(\nu ',\nu )$ , it takes the form

$$ \begin{align*}r(\nu',\nu)={\langle} r_{<\nu'} r_{\nu'},r_{(\nu',\nu)},{\langle} r_{\nu}^0,r_{\nu}^1{\rangle},r_{(\nu,\kappa)},r_\kappa{\rangle}.\end{align*} $$

Reflecting some of the properties of t we obtain a set $B'\in U^2$ such that for every $(\nu ',\nu )\in B'$ :

  1. $(1)_{(\nu ',\nu )}$ .

  2. $(2a)_{(\nu ',\nu )}$ $ r_{<\kappa }\Vdash (\nu '\cup \{\nu \})\times \phi "\nu '\hspace{-1pt} \subseteq\hspace{-1pt} \operatorname {\mathrm {dom}}(r_{\kappa })\wedge {\langle }0,\nu {\rangle }\in \operatorname {\mathrm {dom}}(r_{\kappa })\wedge \operatorname {\mathrm {Supp}}(r_{\kappa }) \subseteq \theta .$

  3. $(2b)_{(\nu ',\nu )}$ $r_{<\kappa }\Vdash \forall \alpha \in \phi "\nu '.r_{\kappa ,\alpha }(\nu )=1$ and $r_{\kappa ,\nu }(0)=\nu '$ .

  4. $(3)_{(\nu ',\nu )}$ $ r_{<\nu }\Vdash \nu '\times \operatorname {\mathrm {dom}}(\Gamma _1(\nu ',\nu ))\hspace{-1pt}\subseteq\hspace{-1pt} \operatorname {\mathrm {dom}}(r^1_\nu )$ and for every .

  5. $(4)_{(\nu ',\nu )}$ $r_{<\nu '}=t_{<\kappa }\in V_{\nu '}$ .

Let

$$ \begin{align*}B"=\{\nu \mid\exists(\nu',\nu)\in B'. r(\nu',\nu)\in G_\kappa*F_\kappa\}.\end{align*} $$

Since $B'\in U^2$ we have that $(\kappa ,\kappa _1)\in j_2(B')$ and since $j_2(r)(\kappa ,\kappa _1)=t\in j^*_2(G_\kappa *F_\kappa )=G_{\kappa _2}*F_{\kappa _2}$ , we conclude that $B"\in W$ . Also, $B"\subseteq B_0$ by clause $(1)$ .

We proceed by a density argument, recalling that by the definition of $G_2$ , we have that ${\langle } t_{<\kappa },t_\kappa {\rangle }\in G_\kappa *F_\kappa $ .

Claim 2.17. Let D be the set of all conditions $q\in \mathcal {P}_{\kappa +1}$ , such that exists $(\nu ^{\prime }_0,\nu _0),(\nu ^{\prime }_1,\nu _1)\in B', \ \nu ^{\prime }_1>\nu _0$ and a $\mathcal {P}_{\nu _0}$ -name such that:

  1. (a) $r(\nu ^{\prime }_0,\nu _0),r(\nu ^{\prime }_1,\nu _1)\leq q$ .

  2. (b) .

  3. (c)

Then D is dense (open) above ${\langle } t_{<\kappa },t_\kappa {\rangle }$ and thus $D\cap G_\kappa *F_\kappa \neq \emptyset .$

Proof Work in V, and let ${\langle } t_{<\kappa },t_\kappa {\rangle }\leq p:={\langle } p_{<\kappa },p_\kappa {\rangle } \in {\mathcal P}_{\kappa +1}$ . We will define two extensions $p\leq q\leq q^*$ which corresponds to the choice of $(\nu ^{\prime }_0,\nu _0),(\nu ^{\prime }_1,\nu _1)$ such that $q^*\in D$ . By definition of $\mathcal {P}_{\kappa +1}$ , $p_{<\kappa }\Vdash p_\kappa \in \operatorname {\mathrm {Cohen}}(\kappa ,\kappa ^+)$ , by $\kappa -$ cc of $\mathcal {P}_\kappa $ , for some $Z\subseteq \kappa ^+,\ Z\in V$ , $|Z|<\kappa $ and some $\gamma <\kappa $ , $p_{<\kappa }\Vdash \operatorname {\mathrm {dom}}(p_\kappa )\subseteq \gamma \times Z$ . Applying $j_2$ , we have that

$$ \begin{align*}j_2(p_{<\kappa})=p_{<\kappa}\Vdash\operatorname{\mathrm{dom}}(j_2(p_\kappa))\subseteq j_2(\gamma\times Z)= \gamma\times j_2^{\prime\prime}Z\text{ and } j_2(p_{\kappa})_{j_2(\alpha)}=p_{\kappa,\alpha}\geq t_{\kappa,\alpha}.\end{align*} $$

Combining with $(2c)$ , we have both

$$ \begin{align*}p_{<\kappa}\Vdash Z\supseteq \operatorname{\mathrm{Supp}}(t_\kappa)\wedge \forall \beta\in Z.j_2(p_{\kappa})_{j_2(\beta)}\geq t_{\kappa,\beta},\end{align*} $$

To reflect this, denote $\mu =(j_2\restriction (Z\cup \theta ))^{-1}\in M_2$ , then

$$ \begin{align*}\operatorname{\mathrm{dom}}(\mu)=j_2(Z)\cup j_2^{\prime\prime}\theta, \ \operatorname{\mathrm{rng}}(\mu)=Z\cup\theta, \ \mu\text{ is 1--1},\end{align*} $$

and we can reformulate

$$ \begin{align*}p_{<\kappa}\Vdash \mu"j_2(Z)\supseteq \operatorname{\mathrm{Supp}}(t_\kappa)\wedge \forall \beta\in j_2(Z).j_2(p_{\kappa})_{\beta}\geq t_{\kappa,\mu(\beta)},\end{align*} $$

Also, since we can find $\delta <\kappa $ such that $t_{<\kappa }\Vdash \phi "(\delta ,\kappa )\cap Z=\emptyset .$ There exists such $\delta $ since $|Z|<\kappa $ , $t_{<\kappa }\Vdash |\operatorname {\mathrm {Supp}}(t_{\kappa })|<\kappa $ and by $\kappa $ -cc of $\mathcal {P}_\kappa $ . Recall that by the definition of $\mu _1$ , $\phi "(\delta ,\kappa )=\mu _1^{\prime \prime }\{s_\gamma (\kappa _1)\mid \gamma \in j_2(\phi )"(\delta ,\kappa )\}$ and that $\mu "\operatorname {\mathrm {Supp}}(j_2(p_\kappa ))=Z$ . Therefore in $M_2$ we will have that

$$ \begin{align*}p_{<\kappa}\Vdash [\mu_1^{\prime\prime}\{s_\gamma(\kappa_1)\mid \gamma\in j_2(\phi)"(\delta,\kappa)\}]\cap[ \mu"\operatorname{\mathrm{Supp}}(j_2(p_\kappa))]=\emptyset.\end{align*} $$

Let $\Gamma $ be such that $j_2(\Gamma )(\kappa ,\kappa _1)=\mu $ , and there is a set $\overline {B}_0\subseteq B'$ , $\overline {B}_0\in U^2$ such that for every $(\nu ',\nu )\in \overline {B}_0$ ,

$$ \begin{align*}(i) \ \ p_{<\kappa}\Vdash \Gamma(\nu',\nu)"Z\supseteq \operatorname{\mathrm{Supp}}(r_{\nu'})\wedge \forall\beta\in Z. p_{\kappa,\beta}\geq r_{\nu',\Gamma(\nu',\nu)(\beta)},\end{align*} $$
$$ \begin{align*}(iii) \ \ p_{<\kappa}\Vdash\Gamma_1(\nu',\nu)"\{s_\gamma(\nu)\mid \gamma\in \phi"(\delta,\nu')\}\cap [\Gamma(\nu',\nu)"\operatorname{\mathrm{Supp}}(p_\kappa)]=\emptyset.\end{align*} $$

Let us move to the choice of $(\nu ^{\prime }_0,\nu _0),(\nu ^{\prime }_1,\nu _1)$ . In $V[G_{\kappa }*F_{\kappa }]$ , there exists $(\nu ^0_0,\nu _0),(\nu ^0_1,\nu _1)\in \overline {B}_0$ such that $r(\nu ^0_0,\nu _0),r(\nu ^0_1,\nu _1)\in G_{\kappa }*F_{\kappa }$ (hence they are compatible) such that $\nu ^0_0>\delta ,\gamma ,\sup (\operatorname {\mathrm {Supp}}(p_{<\kappa }))$ and $\nu ^0_1>\nu _0,\operatorname {\mathrm {Supp}}(r_{<\kappa }(\nu ^0_0,\nu _0))$ . In particular, in V we can find $(\nu ^{\prime }_0,\nu _0),(\nu ^{\prime }_1,\nu _1)\in \overline {B}_0$ such that $r(\nu _0^{\prime },\nu _0),r(\nu _1^{\prime },\nu _1)$ are compatible, $\nu ^{\prime }_0>\delta ,\gamma ,\sup (\operatorname {\mathrm {Supp}}(p_{<\kappa }))$ , and $\nu ^{\prime }_1>\nu _0,\sup (\operatorname {\mathrm {Supp}}(r_{<\kappa }(\nu _0^{\prime },\nu _0)))$ . Denote

$$ \begin{align*}r^0:=r(\nu^{\prime}_0,\nu_0)={\langle} r^0_{<\nu^{\prime}_0},r^0_{\nu^{\prime}_0},r^0_{(\nu^{\prime}_0,\kappa)},r^0_\kappa{\rangle},\end{align*} $$
$$ \begin{align*}r^1:=r(\nu^{\prime}_1,\nu_1)=(r^1_{<\nu^{\prime}_1},r_{\nu^{\prime}_1},r_{(\nu^{\prime}_1,\nu_1)},{\langle} r^{0,1}_{\nu_1},r^{1,1}_{\nu_1}{\rangle},r^1_{(\nu_1,\kappa)},r^1_\kappa{\rangle}.\end{align*} $$

Let us define the first extension q, and it has the form:

$$ \begin{align*}q=p_{<\kappa}{}^{\smallfrown}q_{\nu^{\prime}_0}{}^{\smallfrown}r^0_{(\nu^{\prime}_0,\kappa)}{}^{\smallfrown}q_\kappa.\end{align*} $$

First, $q_{\nu ^{\prime }_0}$ is a $\mathcal {P}_{\nu ^{\prime }_0}$ -name for a condition with $\operatorname {\mathrm {Supp}}(q_{\nu ^{\prime }_0})= \Gamma (\nu ^{\prime }_0,\nu _0)"Z$ , by $(i) \ \operatorname {\mathrm {Supp}}(q_{\nu ^{\prime }_0})\supseteq \operatorname {\mathrm {Supp}}(r^0_{\nu ^{\prime }_0})$ . Set $q_{\nu ^{\prime }_0,\Gamma (\nu ^{\prime }_0,\nu _0)(\beta )}= p_{\kappa ,\beta }$ . As for $q_\kappa $ , we set it to be a $\mathcal {P}_\kappa $ -name for $r^0_\kappa \cup p_\kappa $ .

Once we will prove that $p_{<\kappa },r^0_{<\kappa }\leq q_{<\kappa }$ , from $(i)$ and $(ii)$ it will follow that $q_{<\kappa }$ forces $q_\kappa $ to be a partial function. Indeed, for every $\beta \in \operatorname {\mathrm {Supp}}(r^0_\kappa )\cap Z$ , $q_{<\kappa }$ will force

Clearly $p\leq q$ . To see that $r^0\leq q$ , up to $\nu ^{\prime }_0$ , we have that by $(4)_{(\nu ^{\prime }_0,\nu _0)}$ that

$$ \begin{align*}q_{<\nu^{\prime}_0}=p_{<\kappa}\geq t_{<\kappa}=r^0_{<\nu^{\prime}_0}.\end{align*} $$

At $\nu ^{\prime }_0$ , if $\alpha = \Gamma (\nu ^{\prime }_0,\nu _0)(\beta )$ , then $(i)$ insures that $q_{\nu ^{\prime }_0,\alpha }=p_{\kappa ,\beta }\geq r^0_{\nu ',\alpha }$ . Since in the interval $(\nu ^{\prime }_0,\kappa )$ , q and $r^0$ are the same, it follows that $q_{<\kappa }\geq r^0_{<\kappa }$ and at $\kappa $ it is clear that $q_{<\kappa }\Vdash r^0_\kappa \leq q_\kappa $ .

Next let us move to the choice of . Since $r^0\leq q$ and , use the maximality principal to find a $\mathcal {P}_{\nu _0}$ -name, such that q forces $(b)$ .Footnote 5

Define the final condition $q\leq q^*$ ,

$$ \begin{align*}q^*=q_{<\kappa}{}^{\smallfrown}q^*_{\nu^{\prime}_1}{}^{\smallfrown}r^1_{(\nu_1^{\prime},\kappa)}{}^{\smallfrown}q^*_\kappa.\end{align*} $$

The crucial point here is that by $(2b)_{(\nu ^{\prime }_1,\nu _1)}$ ,

and since

we have that $r^0\Vdash X_1^{\nu _0}\subseteq \phi "(\nu ^{\prime }_0,\nu _0)\subseteq \phi "(\nu ^{\prime }_0,\nu _1^{\prime })$ . By $(iii)$ we have that $q_{<\kappa }\Vdash [\Gamma _1(\nu ^{\prime }_1,\nu _1)"\{s_\gamma (\nu _1)\mid \gamma \in X^{\nu _0}_1\}]\cap [\Gamma (\nu _1^{\prime },\nu _1)"Z]=\emptyset $ . This will permit to code $d^{\nu _0}$ , let

$$ \begin{align*}\operatorname{\mathrm{Supp}}(q^*_{\nu^{\prime}_1})=[\Gamma_1(\nu^{\prime}_1,\nu_1)"\{s_\gamma(\nu_1)\mid \gamma\in X^{\nu_0}_1\}]\uplus [\Gamma(\nu_1^{\prime},\nu_1)"Z]\end{align*} $$

and

and $q^*_\kappa =q_\kappa \cup r^1_\kappa $ . Note that if $\tau \in \operatorname {\mathrm {Supp}}(q_{\kappa })\cap \operatorname {\mathrm {Supp}}(r^1_{\kappa })$ then either $\tau \in \operatorname {\mathrm {Supp}}(r^0_{\kappa })\cap \operatorname {\mathrm {Supp}}(r^1_{\kappa })$ , and $r^0_{\kappa },r^1_{\kappa }$ are forced to be compatible by $q_{<\kappa }$ and if $\tau \in Z\cap \operatorname {\mathrm {Supp}}(r^1_{\kappa })$ then the same argument as before works. We conclude that $r^0\leq q\leq q^*$ , $r^1\leq q^*$ , namely $(a)$ . Finally, for every $\tau \in X^{\nu _0}_1$ , $s_\tau (\nu _1)\in \operatorname {\mathrm {dom}}(\Gamma _1(\nu ^{\prime }_1,\nu _1))$ and by $(3)_{(\nu ^{\prime }_1,\nu _1)}$ we have that $q^*$ forces that

Then $p\leq q^*$ and $q^*\in D.$

By density, we can find such a condition $p^*\in G_\kappa *F_\kappa \cap D$ and points $(\nu ^{\prime }_0,\nu _0),(\nu _1^{\prime },\nu _1)\in B'$ witnessing $p^*\in D$ . It follows that $r(\nu ^{\prime }_0,\nu _0),r(\nu ^{\prime }_1,\nu _1)\in G_\kappa *F_\kappa $ , and by $(1)_{(\nu ^{\prime }_0,\nu _0)},(1)_{(\nu ^{\prime }_1,\nu _1)}$ , $\nu _0,\nu _1\in B_0$ . Extend ${\langle } {\langle }{\rangle },B{\rangle }$ by $p^*={\langle } \nu _0,\nu _1,B_0\cap (\cap _{\tau \in \phi "\nu _0}A_\tau {\rangle }\setminus \nu _1+1{\rangle }$ . By $(2b)_{(\nu ^{\prime }_1,\nu _1)}$ , for every $\tau \in \phi "\nu _0\subseteq \phi "\nu ^{\prime }_1$ , $f_{\kappa ,\tau }(\nu _1)=r_{\kappa ,\tau }(\nu _1)=1$ , hence $\nu _1\in \cap _{\tau \in \phi "\nu _0}A_\tau $ and $p^*\Vdash n_\tau =\begin {cases} 0, &\tau \in X_0,\\ 1, & \tau \in X_1.\end {cases}$ In other words, since $\nu _0\in B_0$ ,

Let

, it follows that

extends $d_\tau $ , and by $(c)$ of the definition of D,

extends $d_\tau $ . Thus

. This concludes the genericity proof.

3 The results where $2^{\kappa }=\kappa ^{++}$

3.1 Strong non-Galvin witnesses of length $2^{\kappa }=\kappa ^{++}$

In this section we produce a model with a non-Galvin ultrafilter with a strong witnessing sequence of length $2^{\kappa }=\kappa ^{++}$ . This will of course require to violate GCH on a measurable cardinal and in turn to start with a stronger large cardinal assumption (see [Reference Gitik15, Reference Mitchell32]). We will follow a similar construction to the one given in the case of $\kappa ^+$ addressed in previous sections. Indeed, instead of iterating $\operatorname {\mathrm {Cohen}}(\alpha ,\alpha ^+)$ we will iterate $\operatorname {\mathrm {Cohen}}(\alpha ,\alpha ^{++})$ aiming to force $\operatorname {\mathrm {Cohen}}(\kappa ,\kappa ^{++})$ , from which we will be able to define a non-Galvin ultrafilter and a strong witness of length $\kappa ^{++}$ in a similar fashion to the one we have on $\kappa ^+$ , distinguishing between $\alpha $ ’s which are in the image of the second iteration and those which are in the image of the factor map. The difficulty is, as always, to extend a ground model embedding. By the large cardinal lower bound, we can no longer work with an ultrapower by an ultrafilter. The usual embedding to lift in the context of violation of GCH at measurables is a $(\kappa ,\kappa ^{++})$ -extender ultrapower embedding, which we will use here. This makes the lifting argument more involved and the existence of generic filters for the iteration requires variations of Woodin’s surgery method (see [Reference Cummings, Foreman and Kanamori12, Section 25]).

Theorem 3.1. Assume GCH and that there is a $(\kappa ,\kappa ^{++})$ -extender over $\kappa $ in V. Then there is a cofinality preserving forcing extension $V^*$ such that $V^*\models 2^{\kappa }=\kappa ^{++}$ , and in $V^*$ there is a $\kappa $ -complete ultrafilter W over $\kappa $ which concentrates on regulars, extends $Cub_\kappa $ , and has a strong witness of length $\kappa ^{++}$ for the failure of Galvin’s property.

Proof Let E be a $(\kappa ,\kappa ^{++})$ -extender. Let $j_1=j_E:V\to M_E=:M_1$ be its ultrapower embedding with $crit(j_E)=\kappa $ and ${}^{\kappa } M_E\subseteq M_E$ . Denote by $E_\alpha $ the ultrafilter

$$ \begin{align*}E_\alpha:=\{X\subseteq \kappa\mid \alpha\in j_E(X)\}.\end{align*} $$

Denote $U:=E_\kappa $ the normal ultrafilter and let $k:M_U \to M_E$ be the factor map defined by setting $k(j_U(f)(\kappa ))=j_E(f)(\kappa )$ such that $j_E=k\circ j_U$ . Define an Easton support iteration as follows:

is trivial unless $\beta $ is inaccessible, in which case $Q_{\beta }=\operatorname {\mathrm {Cohen}}(\beta , \beta ^{++})$ .

Let $G_{\kappa +1}:=G_\kappa *g_\kappa $ be a V-generic subset of . Keeping similar notations to those from previous sections, let ${\langle } f_{\kappa ,\alpha } \mid \alpha <\kappa ^{++}{\rangle } $ be the Cohen generic functions from $\kappa $ to $2$ introduced by $g_\kappa $ .

Now we apply Woodin’s argument (see [Reference Cummings, Foreman and Kanamori12, Section 25], and [Reference Ben-Shalom10] for constructing generics without additional forcing) to see that there will be $G_{j_E(\kappa )+1}*H^*\subseteq j_E({\mathcal P}_{\kappa +1})*\mathbb {S}_0$ in $V^*_1:=V[G_{\kappa +1}][H]$ , where $H\subseteq \mathbb {S}_0$ is a $V[G_{\kappa +1}]$ -generic filter, where $\mathbb {S}_0$ is some $\kappa ^+$ -distributive in $V[G_{\kappa +1}]$ (in the case of Ben-Shalom, there is no need for $H^*$ and we can work directly in $V[G_{\kappa +1}]$ ) generic over $M_E$ and an elementary embedding

$$ \begin{align*}j^*_1: V^*_1\to M_E[G_{j_E(\kappa)+1}*f^*]\end{align*} $$

which extends $j_1$ . Recall that the generic filter constructed for $j_1(Q_\kappa )$ is obtained by a surgery argument, making small changes on an $M_1[G_{j_1(\kappa )}]$ -generic filter f to be compatible with $j^{\prime \prime }_1g_\kappa $ . For our purposes, we need some additional changes to be made; for every $p\in f$ we change p to $p^*$ such that $\operatorname {\mathrm {dom}}(p^*)=\operatorname {\mathrm {dom}}(p)$ and

$$ \begin{align*}p^*({\langle}\gamma,\alpha{\rangle})=\begin{cases}f_\beta(\gamma), & \gamma<\kappa \wedge \alpha=j_1(\beta),\\ \beta, & \gamma=\kappa\wedge \alpha=j_1(\beta),\\ p({\langle}\gamma,\alpha{\rangle}), & \text{else}.\end{cases}\end{align*} $$

To see that p was only changed at $\kappa $ -many places, find $a\in [\kappa ^{++}]^{<\omega }$ such that $j_E(P)(a)=p$ , where $P:\kappa ^{|a|}\rightarrow Q_\kappa $ . By elementarity, for every ${\langle } \alpha ,j_1(\beta ){\rangle }\in \kappa \times j_1^{\prime \prime }\kappa ^{++}\cap \operatorname {\mathrm {dom}}(p)$ , there is $x\in [\kappa ]^{|a|}$ such that ${\langle }\alpha ,\beta {\rangle }\in \operatorname {\mathrm {dom}}(P(x))$ . It follows that $|\kappa \times j_1^{\prime \prime }\kappa ^{++}\cap \operatorname {\mathrm {dom}}(p)|\leq \kappa $ . Moreover, $|\{\kappa \}\times j^{\prime \prime }_1\kappa ^{++}\cap \operatorname {\mathrm {dom}}(p)|\leq \kappa $ , since otherwise there would be some $\alpha <\kappa ^{++}$ such that

$$ \begin{align*}{\mathrm{cf}}(\alpha)=\kappa^+\text{ and }\sup\{j_E(\beta)\mid {\langle}\kappa,j_E(\beta){\rangle}\in\operatorname{\mathrm{dom}}(p)\}=j_E(\alpha).\end{align*} $$

But $|\operatorname {\mathrm {dom}}(p)|^{M_1}<j_1(\kappa )$ and ${\mathrm {cf}}^{M_1}(j_1(\alpha ))=j_1(\kappa )^+$ which is a contradiction. Hence $p^*\in M_1[G_{j_1(\kappa )}]$ since we have only changed p at $\kappa $ -many values and ${}^{\kappa } M_1[G_{j_1(\kappa )}]\subseteq M_1[G_{j_1(\kappa )}]$ .

The argument that such changes do not affect the genericity is the same as in [Reference Cummings, Foreman and Kanamori12]. So we additionally obtain that $f_{\kappa _1,j_1(\beta )}(\kappa )=\beta ,$ for every $\beta <\kappa ^{++}$ .

We also claim that $j^*_1$ is actually the ultrapower embedding by the normal ultrafilter

$$ \begin{align*}U^*=\{X\subseteq \kappa\mid \kappa\in j^*_1(X)\}\end{align*} $$

extending U. To see this, consider $k^*:M_{U^*}\rightarrow M_1[G_{j_1(\kappa )+1}*H^*]$ defined by $k^*([f]_{U^*})=j^*_1(f)(\kappa )$ , which is clearly elementary. To see that $k^*=id$ , let us prove that $k^*$ is onto. Fix

and let $f\in V$ , $a=\{\alpha _1,...,\alpha _r\}\in [\kappa ^{++}]^{<\omega }$ be such that

. Define in $V[G_{\kappa +1}]$ the function $f^*(x)=(f(\{f_{\alpha _1}(x),...,f_{\alpha _r}(x)\}))_{G_{\kappa +1}*H}$ . Then

$$ \begin{align*}k^*(j_{U^*}(f^*)(\kappa))=j^*_1(f^*)(\kappa)=(j_1(f)(\{j_1^*(f_{\alpha_1})(\kappa),...,j^*_1(f_{\alpha_2})(\kappa)\}))_{G_{j_1(\kappa)+1}*H^*}\end{align*} $$

We would like now to construct a $\kappa $ -complete ultrafilter $W\in V[G_{\kappa +1}]$ over $\kappa $ which includes $Cub_\kappa $ and the family ${\langle } A_\alpha \mid \alpha <\kappa ^{++}{\rangle }$ which is a strong witness that W fails to satisfy the Galvin Property. Set

$$ \begin{align*}A_\alpha:=\{\nu<\kappa\mid f_\alpha(\nu) \text{ is odd}\},\end{align*} $$

for every $\alpha <\kappa ^{++}$ .

Consider the second ultrapower (of V) by E, i.e., $\mathrm {Ult}(M_E, j_E(E))$ . In order to simplify the notation let us denote $M_E$ by $M_1$ and $\mathrm {Ult}(M_1, j_1(E))$ by $M_2$ and $j_{2,1}:=j_{j_1(E)}:M_1\rightarrow M_2$ . Also, let $\kappa _1=j_1(\kappa ), E_1=j_1(E),$ and $\kappa _2=j_{2,1}(\kappa _1)$ . Let $j_2:V\to M_2$ be the composition of $j_1$ with $j_{2,1}$ .

Work in $M_1[G_{\kappa _1+1}*H^*]$ , and apply there the Woodin argument to $E_1$ . There will be $G_{\kappa _2+1}*H^{**}\subseteq j_2(P_{\kappa }*Q_\kappa *\mathbb {S}_0)$ (in $M_1[G_{\kappa _1+1}*H^*]$ ) generic over $M_2$ and an elementary embedding

$$ \begin{align*}j_{2,1}^*: M_1[G_{\kappa_1+1}*H^*]\to M_2[G_{\kappa_2+1}*H^{**}]\end{align*} $$

which extends $j_{E_1}$ . Additionally, for every $\alpha <(\kappa _1^{++})^{M_1}$ let us arrange the following:

  1. (1) $f_{\kappa _2,j_{2,1}(\alpha )}(\kappa _1)$ is odd, if $\alpha \in j_E^{\prime \prime }\kappa ^{++}$ .

  2. (2) $f_{\kappa _2, j_{2,1}(\alpha )}(\kappa _1)$ is an even, if $\alpha \in (\kappa _1^{++})^{M_1}\setminus j_E^{\prime \prime }\kappa ^{++}$ .

  3. (3) $f_{\kappa _2,\kappa _1}(\kappa _1)=\kappa $ .

The point being that this requires only small changes of conditions in $(\operatorname {\mathrm {Cohen}}(\kappa _2, \kappa _2^{++}))^{M_2}$ , and so preserves the genericity.

Namely, given $p \in (\operatorname {\mathrm {Cohen}}(\kappa _2, (\kappa _2)^{++}))^{M_2}$ , define $p^*$ such that $\operatorname {\mathrm {dom}}(p^*)=\operatorname {\mathrm {dom}}(p)$ and

$$ \begin{align*}p^*({\langle}\gamma,\alpha{\rangle})=\begin{cases} f_{\kappa_1,\beta}(\gamma), & \gamma<\kappa_1\wedge\exists \beta<\kappa_1^{++} \alpha=j_{2,1}(\beta),\\ \beta\cdot 2+1, & \gamma=\kappa_1\wedge \exists\beta\in j_1^{\prime\prime}\kappa^{++}.\alpha=j_{2,1}(\beta),\\ \beta\cdot 2, & \gamma=\kappa_1\wedge \exists\beta\in\kappa_1^{++}\setminus j_1^{\prime\prime}\kappa^{++}.j_{2,1}(\beta)=\alpha,\\ \kappa, & \gamma=\alpha=\kappa_1,\\ p({\langle}\gamma,\alpha{\rangle}), & \text{otherwise}.\end{cases}\end{align*} $$

In $V[G_{\kappa +1}*H]$ , $|\operatorname {\mathrm {Supp}}(p)\cap j_2^{*\prime \prime }\kappa ^{++}|\leq \kappa $ and $M_1[G_{\kappa _1+1}*H^*]$ is closed under $\kappa $ -sequences, hence $p^*\in M_1$ . The argument we have seen before applied in $M_1[G_{\kappa _1+1}*H^{*}]$ shows that

$$ \begin{align*}M_1[G_{\kappa_1+1}^*]\models |\operatorname{\mathrm{dom}}(p) \cap (\kappa_1+1)\times j^{\prime\prime}_{2,1}{}(\kappa_1^{++})^{M_1[G_{\kappa_1+1}]}|\leq \kappa_1.\end{align*} $$

This implies that $p^*\in M_2[G_{\kappa _2+1}*H^{**}]$ since $M_2[G_{\kappa _2+1}*H^{**}]$ is closed under $\kappa _1$ -sequences from $M_1[G_{\kappa _1+1}*H^*]$ . Then the embedding $j_2:V\to M_2$ extends to

$$ \begin{align*}j^{*}_2: V[G_{\kappa+1}*H^*]\to M_2[G_{\kappa_2+1}*H^{**}].\end{align*} $$

Define now

$$ \begin{align*}W=\{X\subseteq \kappa \mid \kappa_1 \in j_2^{*}(X)\}.\end{align*} $$

Claim 3.2.

  1. (1) $j_W=j^*_2$ , $[id]_W=\kappa _1$ , $U^*\leq _{R-K}W$ .

  2. (2) $Cub_\kappa \subseteq W$ , $\{\alpha <\kappa \mid {\mathrm {cf}}(\alpha )=\alpha )\}\in W$ .

  3. (3) The sequence ${\langle } A_\alpha \mid \alpha <\kappa ^{++}{\rangle }$ is a strong witness for $\neg \text {Gal}(W,\kappa ,\kappa ^{++})$ , where

    $$ \begin{align*}A_\alpha:=\{\nu<\kappa\mid f_{\kappa,\alpha}(\nu)\text{ is odd}\}.\end{align*} $$

Proof Indeed $Cub_\kappa \subseteq W$ and $\{\alpha <\kappa \mid {\mathrm {cf}}(\alpha )=\alpha \}\in W$ , is the same as in Claim 2.8 from the last section. To see $(1)$ , we let $k_W:M_W\rightarrow M_2[j_2^*(G)]$ be the usual factor map $k_W([f]_W)=j^*_2(f)(\kappa _1)$ and we prove that $k_W=id$ by proving that $k_W$ is onto. Let $A\in M_2[G_{\kappa _2+1}*H^{**}]$ , then

where

is a $\mathcal {P}_{\kappa _2+1}*j_2(\mathbb {S}_0)$ -name. Since $j_{2,1}$ is a $(\kappa _1,\kappa _1^{++})$ -extender ultrapower, there is $f\in M_1$ and $a\in [\kappa _1^{++}]^{<\omega }$ such that

. Suppose that $a=\{\alpha _1,...,\alpha _n\}$ is an increasing enumeration. Then by construction, $f_{\kappa _2,j_{2,1}(\alpha _i)}(\kappa _1)\in \{\alpha _i\cdot 2,\alpha _i\cdot 2+1\}$ . In particular we derive $\alpha _i$ from $f_{\kappa _2,j_{2,1}(\alpha _i)}(\kappa _1)$ Footnote 6 . Define $g_{\alpha _i}:\kappa _1\rightarrow \kappa _1\in M_1[G_{\kappa _1+1}*H^*]$ by $g_{\alpha _i}(\alpha )=\lfloor \frac {f_{\kappa _1,\alpha _i}(\alpha )}{2}\rfloor $ , then $j^*_{2,1}(g_{\alpha _i})(\kappa _1)=\lfloor \frac {f_{\kappa _2,j_{2,1}(\alpha _i)}(\kappa _1)}{2}\rfloor =\alpha _i$ . Finally, let $g(\alpha )=f(g_{\alpha _1}(\alpha ),...,g_{\alpha _n}(\alpha ))$ . Then,

We already know that $M_1[G_{\kappa _1+1}*H^*]$ is the ultrapower by $U^*$ , hence $g=j^*_1(h)(\kappa )$ for some $h\in V[G_{\kappa +1}*H]$ and in turn

. Finally, we made sure that $\kappa $ is expressible by $\kappa _1$ , so we define in $V[G_{\kappa +1}*H] \ f^*:\kappa \rightarrow \kappa $ by

$$ \begin{align*}f^*(\alpha)=(h(f_{\kappa,\alpha}(\alpha),\alpha))_G.\end{align*} $$

It follows that

$$ \begin{align*}k_W([f^*]_W)=j_2(f^*)(\kappa_1)=(j_2^*(h)(f_{\kappa_2,\kappa_1}(\kappa_1),\kappa_1))_{G_{\kappa_2+1}*H^{**}}\end{align*} $$

this concludes $(1)$ . $(2)$ and $(3)$ are completely analogous to Claim 2.8.

3.2 Adding $\kappa ^{++}$ -Cohens using Prikry forcing

The construction of the previous section can be modified to obtain a model in which there is a $\kappa $ -complete ultrafilter $U^*$ over $\kappa $ such that $\operatorname {\mathrm {Prikry}}(U^*)$ adds a generic filter for $\operatorname {\mathrm {Cohen}}(\kappa ,\kappa ^{++})$ . This will require the violation of $SCH$ and in turn larger cardinals [Reference Gitik16, Reference Mitchell33].

Theorem 3.3. Assume $GCH$ and that E is a $(\kappa ,\kappa ^{++})$ -extender in V. Then there is a cofinality preserving forcing extension $V^*$ in which $2^{\kappa }=\kappa ^{++}$ and a non-Galvin ultrafilter $W\in V^*$ such that forcing with $\operatorname {\mathrm {Prikry}}(W)$ introduces a $V^*$ -generic filter for $Cohen^{V^*}(\kappa ,\kappa ^{++})$ -generic filter.

Proof Let $j_1:V\to M_E=:M_1$ be the ultrapower embedding of E with $crit(j_1)=\kappa $ and ${}^{\kappa } M_1\subseteq M_1$ and $\kappa _1=j_1(\kappa )$ . Denote by $E_\alpha $ the ultrafilter $\{X\subseteq \kappa \mid \alpha \in j_E(X)\}$ . As before, denote $E_\kappa $ by U and let $k:M_U \to M_E$ be defined by setting $k(j_U(f)(\kappa ))=j_E(f)(\kappa )$ . Define an Easton support iteration as follows:

is trivial unless $\beta $ is inaccessible. If $\beta <\kappa $ is inaccessible, then

Over $\kappa $ , we let

.

Let $G_{\kappa +1}=G_\kappa *F_\kappa $ be a V-generic filter of ${\mathcal P}_{\kappa +1}$ . We denote by $F_\alpha :={\langle } f_{\alpha , \gamma }\mid \gamma <\alpha ^{++}{\rangle }$ the generic Cohen function if $\operatorname {\mathrm {Cohen}}(\alpha ,\alpha ^{++})$ was forced in $G_\kappa $ and by

$$ \begin{align*}F_\alpha:={\langle} f_{\alpha, \gamma}\mid \gamma<\alpha^{++}{\rangle},\ H_\alpha:={\langle} h_{\alpha,\gamma}\mid \gamma<\alpha^{++}{\rangle}\end{align*} $$

if $\operatorname {\mathrm {Cohen}}(\alpha ,\alpha ^{++})\times \operatorname {\mathrm {Cohen}}(\alpha ,\alpha ^{++})$ was. The elementary embedding $j_1$ extends to $j^*_1:V[G_{\kappa +1}]\to M_1[G_{\kappa _1+1}]$ such that at $\kappa $ we forced one block of Cohen’s, $\operatorname {\mathrm {Cohen}}(\kappa ,\kappa ^{++})$ , and for every $\alpha <\kappa ^{++}$ ,

$$ \begin{align*}f_{\kappa_1,j_1(\alpha)}(\kappa)=\alpha.\end{align*} $$

Indeed, in the Woodin and Ben-Shalom argument we first build the generic $G_{\kappa _1}$ up to $\kappa _1$ not including $\kappa _1$ in the same standard fashion as in [Reference Cummings, Foreman and Kanamori12]. The original construction of Woodin or Ben-Shalom of the Cohen generic $F_{\kappa _1}$ which is $M_1[G_{\kappa _1}]$ -generic for $\operatorname {\mathrm {Cohen}}(\kappa _1,\kappa _1^{++})^{M_1[G_{\kappa _1}]}$ applies in our case, as it only uses the fact that $M_1[G_{\kappa _1}]$ is closed under $\kappa $ -sequences and properties of $\operatorname {\mathrm {Cohen}}(\kappa _1,\kappa _1^{++})$ . Since

$$ \begin{align*}\operatorname{\mathrm{Cohen}}(\kappa_1,\kappa_1^{++})\simeq \operatorname{\mathrm{Cohen}}(\kappa_1,\kappa_1^{++})\times \operatorname{\mathrm{Cohen}}(\kappa_1,\kappa_1^{++}),\end{align*} $$

we can split the generic $F_{\kappa _1}$ and assume it is of the form $F_{\kappa _1}\times H_{\kappa _1}$ , which is $M_1[G_{\kappa _1}]$ -generic for $\operatorname {\mathrm {Cohen}}(\kappa _1,\kappa _1^{++})\times \operatorname {\mathrm {Cohen}}(\kappa _1,\kappa _1^{++})$ . Work inside $V[G_{\kappa }*F_\kappa ]$ , and modify the values of $F_{\kappa _1}$ and $H_{\kappa _1}$ , as in the previous section so that for every $\alpha <\kappa ^{++}$ ,

$$ \begin{align*}f_{\kappa_1,j_1(\alpha)}\restriction \kappa= h_{\kappa_1,j_1(\alpha)\cdot2+1}\restriction \kappa=f_{\kappa,\alpha}\end{align*} $$

and for every $\alpha <\kappa ^{++}$ , $f_{\kappa _1,j_1(\alpha )}(\kappa )=\alpha $ .

Lift $j_1$ to the embedding $j_1\subseteq j_1^*:V[G_{\kappa +1}]\rightarrow M_E[G_{\kappa _1}*F_{\kappa _1}]$ . Note that $H_{\kappa _1}$ will be used only later. Set

$$ \begin{align*}U^*=\{X\subseteq\kappa\mid \kappa\in j^*_1(X)\},\end{align*} $$

then $U\subseteq U^*$ and $j^*_1$ is actually the ultrapower embedding by $U^*$ . Continuing as before, consider the second ultrapower (of V) by E. Denote $M_E$ by $M_1$ and $\mathrm {Ult}(M_E, j_E(E))$ by $M_2$ , $j_{2,1}=j_{j_1(E)}:M_1\rightarrow M_2$ the ultrapower embedding. Also, let $ E_1=j_1(E)$ and $\kappa _2=j_{2,1}(\kappa _1)$ . Let $j_2:V\to M_2$ be the composition of $j_1$ with $j_{2,1}$ . The extension of $j_{2,1}$ will be such that at $\kappa _1$ we force with $\operatorname {\mathrm {Cohen}}(\kappa _1,\kappa _1^{++})\times \operatorname {\mathrm {Cohen}}(\kappa _1,\kappa _1^{++})$ part of the Lottery sum. To realize this, we define in $M_1[G_{\kappa _1}*(F_{\kappa _1}\times H_{\kappa _1})]$ we take the generic $G_{\kappa _1}$ up to $\kappa _1$ . At $\kappa _1$ we take $F_{\kappa _1}\times H_{\kappa _1}$ , then in $M_1[G_{\kappa _1}*(F_{\kappa _1}\times H_{\kappa _1})]$ we construct as in Woodin and Ben-shalom argument in $V[G_{\kappa }*F_\kappa ]$ an $M_2[G_{\kappa _1}*(F_{\kappa _1}\times H_{\kappa _1})]$ -generic $G_{(\kappa _1,\kappa _2)}*F_{\kappa _2}$ such that $j_{2,1}^{\prime \prime }G_{\kappa _1}*F_{\kappa _1}\subseteq G_{\kappa _2}*F_{\kappa _2}$ . Denote by ${\langle } f_{\kappa _2,\alpha }\mid \alpha <(\kappa _2^{++})^{M_2}{\rangle }$ the Cohen function induced by $F_{\kappa _2}$ . We also secure that for every $\alpha <(\kappa _1^{++})^{M_1}$ :

  1. (1) $f_{\kappa _2 k(\alpha )}(\kappa _1)=\alpha \cdot 2+1$ , if $\alpha \in j_E^{\prime \prime }\kappa ^{++}$ .

  2. (2) $f_{\kappa _2 k(\alpha )}(\kappa _1)=\alpha \cdot 2$ , if $\alpha \in (\kappa _1^{++})^{M_1}\setminus j_E^{\prime \prime }\kappa ^{++}$ .

  3. (3) $f_{\kappa _2,\kappa _1}(\kappa _1)=\kappa $ .

Formally, given $p \in (\operatorname {\mathrm {Cohen}}(\kappa _2, (\kappa _2)^{++}))^{M_2[G_{\kappa _2}]}$ , define $p^*$ such that $\operatorname {\mathrm {dom}}(p^*)=\operatorname {\mathrm {dom}}(p)$ and

$$ \begin{align*}p^*({\langle}\gamma,\beta{\rangle})=\begin{cases} f_{\kappa_1,\alpha}(\gamma), & \gamma<\kappa_1\wedge \beta=k(\alpha),\\ \alpha\cdot 2+1, & \gamma=\kappa_1\wedge \beta=k(\alpha)\wedge \alpha\in j^{\prime\prime}_E\kappa^{++},\\ \alpha\cdot 2, & \gamma=\kappa_1\wedge \beta=k(\alpha)\wedge \alpha\in (\kappa_1^{++})^{M_1}\setminus j_E^{\prime\prime}\kappa^{++},\\ \kappa, & \alpha=\gamma=\kappa_1,\\ p({\langle}\gamma,\alpha{\rangle}), & \text{otherwise}.\end{cases}\end{align*} $$

In $V[G_{\kappa +1}]$ , $|\operatorname {\mathrm {dom}}(p)\cap j^{\prime \prime }_{E^2}\kappa ^{++}|\leq \kappa $ and $M_1[G_{\kappa _1+1}]$ is closed under $\kappa $ -sequences, hence $p^*\in M_1[G_{\kappa _1+1}]$ . The argument we have seen before applied in $M_1[G_{\kappa _1+1}]$ , thus

$$ \begin{align*}M_1[G_{\kappa_1+1}]\models |\operatorname{\mathrm{dom}}(p) \cap (\kappa_1+1)\times j^{\prime\prime}_{1 2}{}(\kappa_1^{++})^{M_1[G_{\kappa_1+1}]}|\leq \kappa_1.\end{align*} $$

This implies that $p^*\in M_2[G_{\kappa _2+1}]$ since $M_2[G_{\kappa _2+1}]$ is closed under $\kappa _1$ -sequences from $M_1[G_{\kappa _1+1}]$ .

Extend in $V[G_{\kappa }*F_\kappa ]$ , $j_{2,1}\subseteq j^*_2:M_1[G_{\kappa _1}*F_{\kappa _1}\rightarrow M_2[G_{\kappa _2}*F_{\kappa _2}]$ and let $j^*_2:V[G_{\kappa }*F_\kappa ]\rightarrow M_2[G_{\kappa _2}*F_{\kappa _2}]$ be the composition $j^*_{2,1}\circ j_1^*$ . Note that $j_{2,1}^*$ is definable only in $V[G_{\kappa }*(F_{\kappa }]$ . Denote by $V[G_{\kappa }*F_\kappa ]=V^*$ , define

$$ \begin{align*}W=\{X\subseteq\kappa\mid \kappa_1\in j^*_2(X)\}\in V^*\text{ and }A_\alpha=\{\beta<\kappa\mid f_\alpha(\beta)\text{ is odd}\}.\end{align*} $$

Claim 3.4. W is a $\kappa $ -complete ultrafilter over $\kappa $ such that:

  1. (1) $j_W=j^*_2$ , $[id]_W=\kappa _1$ , $U^*\leq _{R-K}W$ .

  2. (2) $Cub_\kappa \subseteq W$ , $\{\alpha <\kappa \mid {\mathrm {cf}}(\alpha )=\alpha )\}\in W$ .

  3. (3) $L_0:=\{\beta <\kappa \mid \operatorname {\mathrm {Cohen}}(\beta ,\beta ^{++})\times \operatorname {\mathrm {Cohen}}(\beta ,\beta ^{++})\text { was forced in }G_{\kappa +1}\}\in W.$

  4. (4) For every $\alpha <\kappa ^{++}$ , $L_{1,\alpha }:=\{\nu <\kappa \mid f_{\kappa ,\alpha }(\nu )<\nu ^{++}\}\in W$ .

  5. (5) The sequence ${\langle } A_\alpha \mid \alpha <\kappa ^{++}{\rangle }$ is a strong witness for $\neg \text {Gal}(W,\kappa ,\kappa ^{++})$ . Moreover, the sequence ${\langle } A_\alpha \cap L_{1,\alpha }\mid \alpha <\kappa ^{++}{\rangle }$ is a witness for $\neg \text {Gal}(W,\kappa ,\kappa ^{++})$ .

Proof $(1),(2),$ and the first part of $(5)$ are the same argument as in Claim 3.2. As for $(3)$ , note that we have constructed the generic $G_{\kappa _2+1}=j^*_2(G_{\kappa +1})$ so that on $\kappa _1$ we have forced $\operatorname {\mathrm {Cohen}}(\kappa _1,\kappa _1^{++})\times \operatorname {\mathrm {Cohen}}(\kappa _1,\kappa _1^{++})$ . To see $(4)$ , for every $\alpha <\kappa ^{++}$ ,

$$ \begin{align*}j_2^*(f_{\kappa,\alpha})(\kappa_1)=f_{\kappa_2,j_{2,1}(j_1(\alpha))}(\kappa_1)=j_1(\alpha)\cdot 2+1<\kappa_1^{++}.\end{align*} $$

Hence by elementarity, $\kappa _1\in j^*_2(L_{1,\alpha })$ . Finally, the moreover part of $(5)$ , toward a contradiction if there would be a set $I\in [\kappa ^{++}]^{\kappa }$ such that $\cap _{i\in I}A_\alpha \cap L_{1,\alpha }\in W$ then clearly $\cap _{i\in I}A_\alpha \in W$ , contradicting the first part of $(5)$ that $A_\alpha $ ’s form a witness for $\neg \text {Gal}(W,\kappa ,\kappa ^{++})$ .

Denoted by $\nu \mapsto \pi _{nor}(\nu )$ the Rudin–Keisler projection from W to $U^*$ , and let us prove that W witnesses the theorem:

Proposition 3.5. Let $H\subseteq \operatorname {\mathrm {Prikry}}(W)$ be a $V^*$ -generic filter. There is $G^*\in V^*[H]$ which is $V^*$ -generic for $Cohen(\kappa ,\kappa ^{++})^{V^*}$ .

Proof of Proposition 3.5

Let ${\langle } c_n \mid n<\omega {\rangle } $ be the W-Prikry sequence corresponding to H. Suppose without loss of generality that for every $n<\omega $ , $c_n\in L_0$ .

Define, for every $n<\omega $ , the set

$$ \begin{align*}Z_n=\{\alpha<\kappa^{++}\mid \{c_m \mid n\leq m<\omega\} \subseteq A_\alpha\cap L_{1,\alpha} \text{ and } n \text{ is least possible} \}.\end{align*} $$

For every $\alpha <\kappa ^{++}$ , let $n_\alpha $ be the unique n such that $\alpha \in Z_n$ . Let $\alpha <\kappa ^+$ , and define $f^*_\alpha :\kappa \to \kappa $ as follows:

Denote by

$$ \begin{align*}{\langle} f_{c_n,\alpha}\mid\alpha<c_n^{++}{\rangle}, \ {\langle} h_{c_n,\alpha}\mid\alpha<c_n^{++}{\rangle}\end{align*} $$

the generic $c_n$ -Cohen functions forced by G and define the function $f^*_\alpha :\kappa \rightarrow \kappa $ by

$$ \begin{align*}f^*_\alpha=h_{c_{n_\alpha},f_{\kappa,\alpha}(c_{n_\alpha})}\cup\left(\bigcup_{n_\alpha<n<\omega}h_{c_n,f_{\kappa,\alpha}(c_n)}\restriction[c_{n-1},c_n)\right).\end{align*} $$

Note that the Cohen functions on $\kappa $ play the role of the canonical functions from the previous section. Let us prove that $F={\langle } f^*_\alpha \mid \alpha <\kappa ^{++}{\rangle }$ are Cohen generic functions over $V^*$ .

Claim 3.6. Let $G^*=\{p\in \operatorname {\mathrm {Cohen}}(\kappa ,\kappa ^{++})^{V^*}\mid p\subseteq F\}$ , then $G^*$ is a $V^*$ -generic filter.

Let $\mathcal {A}\in V^*$ be a maximal antichain in the forcing $\operatorname {\mathrm {Cohen}}(\kappa , \kappa ^{++})^{V^*}$ . Note that since $\operatorname {\mathrm {Cohen}}(\kappa ,\kappa ^{++})^{V^*}$ is $\kappa $ -closed then

$$ \begin{align*}Cohen(\kappa,\kappa^{++})^{V[G_\kappa]}=Cohen(\kappa,\kappa^{++})^{V^*}.\end{align*} $$

By $\kappa ^+$ -cc of the forcing, there is $Y'\subseteq \kappa ^{++}$ , $Y'\in V$ such that $|Y'|=\kappa $ and $\mathcal {A}\subseteq \operatorname {\mathrm {Cohen}}(\kappa ,Y')^{V^*}$ . Also, since $|\mathcal {A}|=\kappa $ , $\mathcal {A}\in V[G_\kappa *F_\kappa ]$ , there is $Z\subseteq \kappa ^{++}$ such that $|Z|=\kappa $ such that $\mathcal {A}\in V[G_\kappa *F_\kappa \restriction Z]$ . Without loss of generality assume that $Z=Y\in V$ . Let $V\ni \phi :\kappa \rightarrow Y$ be a bijection.

As in Claim 2.13, we can construct an $\in $ -increasing continuous chain ${\langle } N_\beta \mid \beta <\kappa {\rangle }\in V^*$ of elementary submodels of $H_\chi $ such that:

  1. (1) $|N_\beta |<\kappa $ .

  2. (2) $G_{\kappa +1},\mathcal {A},\phi ,Y\in N_0$ .

  3. (3) $N_\beta \cap \kappa =\gamma _\beta $ is a cardinal $<\kappa $ , and $\gamma _{\beta +1}$ is regular.

  4. (4) If $\gamma _\beta $ is regular, then $\operatorname {\mathrm {Cohen}}(\gamma _\beta ,\phi "\gamma _\beta )=\operatorname {\mathrm {Cohen}}(\kappa ,Y)\cap N_\beta $ .

Set

$$ \begin{align*}C=\{ \beta<\kappa \mid \gamma_\beta=\beta \}.\end{align*} $$

This is club in $\kappa $ since the sequence $\gamma _\beta $ is continuous and since the set $\{\beta \mid \gamma _\beta =\beta \}$ is a club.

Recall that by construction $j^*_2({\langle } f_{\kappa ,\alpha }\mid \alpha <\kappa ^{++}{\rangle })={\langle } f_{\kappa _2,\alpha }\mid \alpha <\kappa _2^{++}{\rangle }$ . Also, for every $\nu \in j_2(\phi )"\kappa _1$ there is $\gamma <\kappa _1$ such that $\nu =j_2(\phi )(\gamma )$ , and since $crit(j_{2,1})=\kappa _1$ , $\nu =j_{2,1}(j_1(\phi )(\gamma ))$ . Since $j_1(\phi ):\kappa _1\rightarrow \kappa _1^{++}$ we conclude that $\nu =j_{2,1}(\alpha )$ for some $\alpha <(\kappa _1^{++})^{M_1}$ which implies that

$$ \begin{align*}f_{\kappa_2,\nu}(\kappa_1)\in\{\alpha\cdot 2,\alpha\cdot 2+1\}.\end{align*} $$

Since $\phi $ is a bijection, for every distinct $\nu _1,\nu _2\in j_2(\phi )"\kappa _1$ , $f_{\kappa _2,\nu _1}(\kappa _1)\neq f_{\kappa _2,\nu _2}(\kappa _1)$ . Reflecting this, we obtain that the set

$$ \begin{align*}E:=\{\nu<\kappa\mid \forall\nu_1,\nu_2\in \phi"\nu.\nu_1\neq\nu_2\rightarrow f_{\kappa,\nu_1}(\nu)\neq f_{\kappa,\nu_2}(\nu)\}\in W.\end{align*} $$

Also, by construction, for every $\alpha <\kappa _1^{++}$ , $f_{\kappa _2,j_{2,1}(\alpha )}\restriction \kappa _1=f_{\kappa _1,\alpha }$ and therefore for every $\alpha \in j_2(\phi )"\kappa _1$ , there is $\nu <\kappa _1^{++}$ such that

$$ \begin{align*}\alpha=j_{2,1}(j_1(\phi))(\nu)=j_{2,1}(j_1(\phi)(\nu))\end{align*} $$

and $j_1(\phi )(\nu )<\kappa _1^{++}$ . Hence $f_{\kappa _2,\alpha }\restriction \kappa _1=f_{\kappa _1,\beta }$ for some $\beta <\kappa _1^{++}$ . Reflecting this we obtain that the set

$$ \begin{align*}F:=\{\beta<\kappa\mid \forall \gamma\in \phi"\beta.\exists\delta<\beta^{++}.f_{\kappa,\gamma}\restriction\beta=f_{\beta,\delta}\} \in W.\end{align*} $$

Now the argument of Claim 2.15 applies since for every $\nu _0\in C\cap E\cap F$ , $\forall \tau _1<\tau _2\in \phi "\nu _0$ , $f_{\kappa ,\tau _1}(\nu _0)\neq f_{\kappa ,\tau _2}(\nu _0)$ , hence ${\langle } h_{\nu _0,f_{\kappa ,\tau }(\nu _0)}\mid \tau \in \phi "\nu _0{\rangle }$ are distinct mutually $V[G_{\nu _0}*F_{\nu _0}]$ -generic Cohen functions over $\nu _0$ . Thus, we can find $d\in \mathcal {A}\cap \operatorname {\mathrm {Cohen}}(\nu _0,\nu _0^{++})$ such that d is extended by ${\langle } h_{\nu _0,f_{\kappa ,\alpha }(\nu _0)}\mid \alpha \in \phi "\nu _0{\rangle }$ . Finally we note that

$$ \begin{align*}R:=\{\nu<\kappa\mid \forall \alpha\in\phi"\pi_{nor}(\nu). f_{\kappa,\alpha}(\nu)\text{ is odd}\}\in W.\end{align*} $$

Let $p={\langle } {\langle }{\rangle }, B{\rangle }$ be a condition, shrink B to $B_0:=B\cap C\cap E\cap F\cap R\in W$ , and pick now any $\nu _0\in B_0$ . Split $\phi "\nu _0$ into two sets:

$$ \begin{align*}X^{\nu_0}_0:=\{\tau\in\phi"\nu_0\mid \nu_0\in A_\tau\}\text{ and }X^{\nu_0}_1=\phi"\nu_0\setminus X^{\nu_0}_0.\end{align*} $$

Since $\nu _0\in R$ we have that $X_1\subseteq \phi "(\pi _{nor}(\nu _0),\nu _0)$ . The condition $p_0={\langle } {\langle } \nu _0{\rangle }, B_0\cap (\bigcap _{\tau \in \phi "\nu _0}A_\tau ){\rangle }$ forces the following:

  1. (1) The Prikry sequence is included in each $A_\tau $ , $\tau \in X^{\nu _0}_0$ , i.e., $n_\tau =0$ .

  2. (2) $n_\tau =1$ , for every $\tau \in X^{\nu _0}_1$ .

In particular, this condition forces some information about the Cohen functions. Namely that:

  1. (1) For $\tau \in X_0^{\nu _0}$ , $f^*_{\tau }\restriction \nu _0=h_{\nu _0,f_{\kappa ,\tau }(\nu _0)}$ .

  2. (2) For $\tau \in X_1^{\nu _0}$ , .

We would like to find a condition in $\mathcal {A}$ which is below these decided parts of the Cohen. By the previous paragraph, there is $d\in N_{\nu _0}\cap \operatorname {\mathrm {Cohen}}(\kappa ,Y)=\operatorname {\mathrm {Cohen}}(\nu _0,\phi "\nu _0)$ , which is extended by ${\langle } h_{\nu _0,f_{\kappa ,\tau }(\nu _0)}\mid \tau \in \phi "\nu _0{\rangle }$ . As before we will need to pick $\nu _0,\nu _1$ so that $d^{\nu _0}\in G^*$ .

Let

be a name in V for $B_0$ . We fix a condition $m_0\in G_\kappa *F_\kappa $ which forces that if

then there is

which is extended by

, and

. Recall that by the construction of $G_{\kappa _2}$ , we have $m_0\in G_{\kappa _2}*F_{\kappa _2}$ . Let $ m_0\leq t\in G_{\kappa _2}*F_{\kappa _2}$ be a condition such that

By the construction of $G_{\kappa _2}*F_{\kappa _2}$ , t has the form:

$$ \begin{align*}t={\langle} t_{<\kappa}, t_{\kappa},t_{(\kappa,\kappa_1)},\underset{t_{\kappa_1}}{\underbrace{{\langle} t_{\kappa_1}^0,t_{\kappa_1}^1{\rangle}}},t_{(\kappa_1,\kappa_2)},t_{\kappa_2}{\rangle}.\end{align*} $$

Distinguishing from the case of $\kappa ^+$ , we now have that $f_{\kappa _2,j_2(\alpha )}(\kappa _1)=j_1(\alpha )\cdot 2+1$ for every $\alpha <\kappa ^+$ ; this will hold for every $\alpha \in \phi "\kappa $ as well. Also, recall that $Y\in V$ , hence $\phi \in V$ . Thus $j_2(\phi )\in M_2$ and $j_2(\phi )"\kappa \in M_2$ . Also, for $(t_{\kappa _2})_{G_{\kappa _2}}\in M_2[G_{\kappa _2}]$ ,

$$ \begin{align*}j_2^{\prime\prime}\kappa^{++}\cap \operatorname{\mathrm{Supp}}((t_{\kappa_2})_{G_{\kappa_2}})\in M_2[G_{\kappa_2}]\end{align*} $$

and $(t_{\kappa _2})_{G_{\kappa _2}}\restriction \kappa \times \{j_2(\alpha )\}\subseteq f_{\kappa ,\alpha }$ . We also fix $X\in V$ , $X\subseteq \kappa ^{++}$ , $|N_0|\leq \kappa $ such that $\operatorname {\mathrm {Supp}}((t_{\kappa _2})_{G_{\kappa _2}})\subseteq j_2(N_0)$ .

Therefore, we can extend if necessary t such that:

$$ \begin{align*}(2a)\ \ t_{<\kappa_2}\Vdash (\kappa\cup\{\kappa_1\})\times j_2(\phi)"\kappa \subseteq \operatorname{\mathrm{dom}}(t_{\kappa_2})\wedge (0,\kappa_1)\in\operatorname{\mathrm{dom}}(t_{\kappa_2})\wedge \operatorname{\mathrm{Supp}}(t_{\kappa_2})\subseteq j_2(N_0),\end{align*} $$
$$ \begin{align*}(2b) \ \ t_{<\kappa_2}\Vdash \ t_{\kappa_2}(\kappa_1,j_2(\alpha))=j_1(\alpha)\cdot 2+1,\text{ for every }j_2(\alpha)\in j_2(\phi)"\kappa\text{ and } t_{\kappa_2,\kappa_1}(0)=\kappa,\end{align*} $$

Next consider $t_{\kappa _1}={\langle } t_{\kappa _1}^0,t_{\kappa _1}^1{\rangle }$ ; it is a ${\mathcal P}_{\kappa _1}$ -name for a condition in $F_{\kappa _1}\times H_{\kappa _1}$ . By the construction of the generic $F_{\kappa _1}\times H_{\kappa _1}$ , for every $\alpha <\kappa ^{++}$ , we made sure that, $h_{\kappa _1,j_1(\alpha )2+1}\restriction \kappa =f_{\kappa ,\alpha }$ . Also, $(j_1(\phi )"\kappa )\cdot 2+1\in M_2$ Footnote 7 . Let

$$ \begin{align*}\mu_1=\{{\langle} j_1(\alpha)\cdot2+1,\alpha{\rangle}\mid \alpha\in \phi"\kappa\}\in M_1.\end{align*} $$

The fact that for every $\beta <\kappa ^{++}$ , $f_{\kappa _2,j_2(\beta )}(\kappa _1)=j_1(\beta )\cdot 2+1$ implies

$$ \begin{align*}\operatorname{\mathrm{dom}}(\mu_1)=(j_1(\phi)"\kappa)\cdot 2+1=\{f_{\kappa_2,\gamma}(\kappa_1)\mid \gamma\in j_2(\phi)"\kappa\}, \ \operatorname{\mathrm{rng}}(\mu_1)=\phi"\kappa\subseteq\kappa^{++}.\end{align*} $$

Extend if necessary $t_{<\kappa _1}$ , and assume that

As for the lower part, due to the Easton support, we have

$$ \begin{align*}(4) \ \ t_{<\kappa}\in V_\kappa.\end{align*} $$

Fix functions $r,\Gamma _1$ which represent $t,\mu $ resp. in the ultrapower $M_{E^2}$ , namely for some $\vec {\xi }\in [\kappa _1^{++}]^{<\omega }$ , $j_2(r)(\vec {\xi })=t, \ j_2(\Gamma _1)(\vec {\xi })=\mu $ . Without loss of generality, suppose that both $\kappa $ and $\kappa _1$ appear in $\vec {\xi }$ , $\kappa =\min (\vec {\xi })=\vec {\xi }(0)$ and $\kappa _1=\vec {\xi }(i_0)$ . Then the functions $\vec {\nu }\in [\kappa ]^{|\vec {\xi }|}\mapsto (\vec {\nu }(0),\vec {\nu }(i_0))$ represent $(\kappa ,\kappa _1)$ . Without loss of generality, suppose that for every $\vec {\nu }$ , it takes the form

$$ \begin{align*}r(\vec{\nu})={\langle} r_{<\vec{\nu}(0)}, r_{\vec{\nu}(0)},r_{(\vec{\nu}(0),\vec{\nu}(i_0))},{\langle} r_{\vec{\nu}(i_0)}^0,r_{\vec{\nu}(i_0)}^1{\rangle},r_{(\vec{\nu}(i_0),\kappa)},r_\kappa{\rangle}.\end{align*} $$

Reflecting some of the properties of t we obtain a set $B'\in E(\vec {\xi })$ such that for every $\vec {\nu }\in B'$ :

  1. $(1)_{\vec {\nu }}$ .

  2. $(2a)_{\vec {\nu }}$ $ r_{<\kappa }\Vdash (\vec {\nu }(0)\cup \{\vec {\nu }(i_0)\})\times \phi "\vec {\nu }(0) \subseteq \operatorname {\mathrm {dom}}(r_{\kappa })\wedge $ ${\langle }0,\vec {\nu }(i_0){\rangle }\in \operatorname {\mathrm {dom}}(r_{\kappa }) \wedge \operatorname {\mathrm {Supp}} (r_{\kappa })\subseteq N_0.$

  3. $(2b)_{\vec {\nu }}$ $r_{<\kappa }\Vdash \forall \alpha \in \phi "\vec {\nu }(0).r_{\kappa ,\alpha }(\vec {\nu }(i_0))$ is odd and $r_{\kappa ,\vec {\nu }(i_0)}(0)=\vec {\nu }(0)$ .

  4. $(3)_{\vec {\nu }}$ $ r_{<\vec {\nu }(i_0)}\Vdash \vec {\nu }(0)\times \operatorname {\mathrm {dom}}(\Gamma _1(\vec {\nu }))\subseteq \operatorname {\mathrm {dom}}(r^1_{\vec {\nu }(i_0)})$ and for every .

  5. $(4)_{\vec {\nu }}$ $r_{<\vec {\nu }(0)}=t_{<\kappa }\in V_{\vec {\nu }(0)}$ .

Let

$$ \begin{align*}B"=\{\nu(i_0) \mid\exists\vec{\nu}\in B'. r(\vec{\nu})\in G_\kappa*F_\kappa\}.\end{align*} $$

Since $B'\in E(\vec {\xi })$ we have that $\vec {\xi }\in j_2(B')$ and since $j_2(r)(\vec {\xi })=t\in j^*_2(G_\kappa *F_\kappa )=G_{\kappa _2}*F_{\kappa _2}$ , we conclude that $B"\in W$ . Also, $B"\subseteq B_0$ by clause $(1)$ .

We proceed by a density argument, and recall that by the definition of $G_2$ , we have that ${\langle } t_{<\kappa },t_\kappa {\rangle }\in G_\kappa *F_\kappa $ .

Claim 3.7. Let D be the set of all conditions $q\in \mathcal {P}_{\kappa +1}$ , such that there exist $\vec {\nu }_0,\vec {\nu }_1\in B', \ \vec {\nu }_1(0)>\vec {\nu }_0(i_0)$ , and a $\mathcal {P}_{\vec {\nu }_0(i_0)}$ -name such that:

  1. (a) $r(\vec {\nu }_0),r(\vec {\nu }_1)\leq q$ .

  2. (b) .

  3. (c)

Then D is dense (open) above ${\langle } t_{<\kappa },t_\kappa {\rangle }$ and thus $D\cap G_\kappa *F_\kappa \neq \emptyset $ .

Proof Work in V, and let ${\langle } t_{<\kappa },t_\kappa {\rangle }\leq p:={\langle } p_{<\kappa },p_\kappa {\rangle } \in {\mathcal P}_{\kappa +1}$ . We will define two extensions $p\leq q\leq q^*$ as before such that $q^*\in D$ . By definition of $\mathcal {P}_{\kappa +1}$ , $p_{<\kappa }\Vdash p_\kappa \in \operatorname {\mathrm {Cohen}}(\kappa ,\kappa ^{++})$ , by $\kappa -$ cc of $\mathcal {P}_\kappa $ , for some $Z\subseteq \kappa ^{++},\ Z\in V$ , $|Z|<\kappa $ and some $\gamma <\kappa $ , $p_{<\kappa }\Vdash \operatorname {\mathrm {dom}}(p_\kappa )\subseteq \gamma \times Z$ . The same argument as before indicates that

$$ \begin{align*}p_{<\kappa}\Vdash Z\supseteq \operatorname{\mathrm{Supp}}(t_\kappa)\wedge \forall \beta\in Z.j_2(p_{\kappa})_{j_2(\beta)}\geq t_{\kappa,\beta},\end{align*} $$

Denote $\mu =(j_2\restriction (Z\cup N_0))^{-1}\in M_2$ , then

$$ \begin{align*}\operatorname{\mathrm{dom}}(\mu)=j_2(Z)\cup j_2^{\prime\prime}N_0, \ \operatorname{\mathrm{rng}}(\mu)=Z\cup\theta, \ \mu\text{ is 1--1},\end{align*} $$

and we can reformulate

$$ \begin{align*}p_{<\kappa}\Vdash \mu"j_2(Z)\supseteq \operatorname{\mathrm{Supp}}(t_\kappa)\wedge \forall \beta\in j_2(Z).j_2(p_{\kappa})_{\beta}\geq t_{\kappa,\mu(\beta)},\end{align*} $$

Also, find $\delta <\kappa $ such that $t_{<\kappa }\Vdash \phi "(\delta ,\kappa )\cap Z=\emptyset .$ We have that

$$ \begin{align*}\phi"(\delta,\kappa)=\mu_1^{\prime\prime}\{f_{\kappa_2,\gamma}(\kappa_1)\mid \gamma\in j_2(\phi)"(\delta,\kappa)\},\text{ and }\mu"\operatorname{\mathrm{Supp}}(j_2(p_\kappa))=Z.\end{align*} $$

Therefore in $M_2$ we will have that

Let $\Gamma $ be such that $j_2(\Gamma )(\vec {\xi })=\mu $ , and there is a set $\overline {B}_0\subseteq B'$ , $\overline {B}_0\in E(\vec {\xi })$ such that for every $\vec {\nu }\in \overline {B}_0$ :

$$ \begin{align*}(i) \ \ p_{<\kappa}\Vdash \Gamma(\vec{\nu})"Z\supseteq \operatorname{\mathrm{Supp}}(r_{\vec{\nu}(0)})\wedge \forall\beta\in Z. p_{\kappa,\beta}\geq r_{\vec{\nu}(0),\Gamma(\vec{\nu})(\beta)},\end{align*} $$

Find $\vec {\nu }_0,\vec {\nu }_1\in \overline {B}_0$ such that $r(\vec {\nu }_0),r(\vec {\nu }_1)$ are compatible, $\vec {\nu }_0(0)>\delta ,\gamma ,\sup (\operatorname {\mathrm {Supp}}(p_{<\kappa }))$ , and $\vec {\nu }_1(0)>\vec {\nu }_0(i_0),\sup (\operatorname {\mathrm {Supp}}(r_{<\kappa }(\vec {\nu }))$ . Denote

$$ \begin{align*}r^0:=r(\vec{\nu}_0)={\langle} r^0_{<\vec{\nu}_0(0)},r^0_{\vec{\nu}_0(0)},r^0_{(\vec{\nu}_0(0),\kappa)},r^0_\kappa{\rangle},\end{align*} $$
$$ \begin{align*}r^1:=r(\vec{\nu}_1)=(r^1_{<\vec{\nu}_1(0)},r_{\vec{\nu}_1(0)},r_{(\vec{\nu}_1(0),\vec{\nu}_1(i_0))},{\langle} r^{0,1}_{\vec{\nu}_1(i_0)},r^{1,1}_{\vec{\nu}_1(i_0)}{\rangle},r^1_{(\vec{\nu}_1(i_0),\kappa)},r^1_\kappa{\rangle}.\end{align*} $$

As before, q has the form: $q=p_{<\kappa }{}^{\smallfrown }q_{\vec {\nu }_0(0)}{}^{\smallfrown }r^0_{(\vec {\nu }_0(0),\kappa )}{}^{\smallfrown }q_\kappa $ . We have $q_{\vec {\nu }_0(0)}$ is a $\mathcal {P}_{\vec {\nu }_0(0)}$ -name for a condition with $\operatorname {\mathrm {Supp}}(q_{\vec {\nu }_0(0)})= \Gamma (\vec {\nu }_0)"Z$ and $q_{\nu ^{\prime }_0,\Gamma (\nu ^{\prime }_0,\nu _0)(\beta )}= p_{\kappa ,\beta }$ . As for $q_\kappa $ , we set it to be a $\mathcal {P}_\kappa $ -name for $r^0_\kappa \cup p_\kappa $ .

The argument that $r^0\leq q$ is the same as in the case of $\kappa ^+$ .

The choice of is possible since $r^0\leq q$ and .

Define the final condition $q\leq q^*$ ,

$$ \begin{align*}q^*=q_{<\kappa}{}^{\smallfrown}q^*_{\vec{\nu}_1(0)}{}^{\smallfrown}r^1_{(\vec{\nu}_1(0),\kappa)}{}^{\smallfrown}q^*_\kappa.\end{align*} $$

Again we have that $r^0\Vdash X_1^{\vec {\nu }_0(i_0)}\subseteq \phi "(\vec {\nu }_0(0),\vec {\nu }_0(i_0))\subseteq \phi "(\vec {\nu }_0(0),\vec {\nu }_1(0))$ and by $(iii)$

Now for the code of

, let

and

and $q^*_\kappa =q_\kappa \cup r^1_\kappa $ . We conclude that $r^0\leq q\leq q^*$ , $r^1\leq q^*$ , namely $(a)$ . Finally, for every $\tau \in X^{\vec {\nu }_0(i_0)}_1$ ,

and by $(3)_{(\vec {\nu }_1)}$ we have that $q^*$ forces that

Then $p\leq q^*$ and $q^*\in D.$

The rest of the argument remains unchanged.

4 On the Extender-based Prikry forcings and adding subsets to $\kappa $

H. Woodin asked in the early 90s whether, assuming that there is no inner model with a strong cardinal, it is possible to have a model M in which $2^{\aleph _\omega }\geq \aleph _{\omega +3}$ , GCH holds below $\aleph _\omega $ , there is an inner model N such that $\kappa =(\aleph _\omega )^M$ is a measurable and $2^{\kappa }\geq (\aleph _{\omega +3})^M$ . His question was natural given the results known back then: Magidor [Reference Magidor26] proved that it is consistent relative to a supercompact cardinal and a huge cardinal above it to have $2^{\aleph _\omega }\geq \aleph _{\omega +m}$ and $GCH_{<\aleph _\omega }$ using the supercompact Prikry forcing with collapses. Woodin, in an unpublished work which can be found in [Reference Cummings11] reduced Magidor’s large cardinal assumption to get $2^{\aleph _{\omega }}=\aleph _{\omega +2}+GCH_<\aleph _{\omega }$ to a strong cardinal (actually to a $p_2\kappa $ -hypermeasurable). Later, Gitik and Magidor [Reference Gitik and Magidor21] proved using the Extender-based Prikry forcing with collapses that starting from the optimal large cardinal assumption, it is possible to obtain $\aleph _{\omega +m}=2^{\aleph _\omega }$ and $GCH_{<\aleph _\omega }$ . However, Woodin’s question remained unanswered.

A natural approach to answer Woodin’s question is to force with the Extender-based Prikry forcing over $\kappa $ and then argue that in some intermediate where $\kappa $ is measurable we added $\lambda \geq \kappa ^{++}$ many subsets to $\kappa $ .

Our purpose will be to show that this direction is doomed. More precisely, we will prove that in any intermediate model of the Extender-based Prikry forcing where $\kappa ^{++}$ -many subsets of $\kappa $ were introduced, $\kappa $ is singularized (and in particular not measurable). We will analyze the situation in both the original version of Gitik and Magidor from [Reference Gitik and Magidor21] and Merimovich version of the Extender-based Prikry forcing from [Reference Merimovich29Reference Merimovich31]. We will rely on the following theorem from [Reference Benhamou and Gitik6, Theorem 6.7]:

Theorem 4.1. Suppose that $\mathbb {U}={\langle } U_a\mid a\in [\kappa ]^{<\omega }{\rangle }$ is a tree of P-point ultrafilters. Let $G\subseteq P(\mathbb {U})$ be V-generic, then for every set of ordinals $A\in V[G]\setminus V$ , $cf^{V[A]}(\kappa )=\omega $ .

Note that if U is any $\kappa $ -complete ultrafilter, then the forcing $\operatorname {\mathrm {Prikry}}(U)$ which we use in this paper is forcing equivalent to $P(\mathbb {U})$ where $\mathbb {U}={\langle } U_a\mid a\in [\kappa ]^{<\omega }{\rangle }$ is such that $U_a=U$ for every a.

Assume $2^{\kappa }=\kappa ^+$ . Let E be an extender over $\kappa $ . We consider two sorts of Extender-based Prikry forcings—the original one (see [Reference Gitik and Magidor21] or [Reference Gitik, Foreman and Kanamori17]) and a more elegant version of Merimovich [Reference Merimovich29Reference Merimovich31].

Let us start with the Merimovich version, but in which the measures of E are P-points as in [Reference Gitik and Magidor21].

4.1 The Merimovich version with P-points

Suppose that there is $h:\kappa \to \kappa $ such that all the generators of E are below $j_E(h)(\kappa )$ .

For example, if E is a $(\kappa , \kappa ^{++})$ -extender, this holds with $h(\nu )=\nu ^{++}$ , $\nu <\kappa $ . This is sufficient to ensure that for every $\alpha <\lambda $ , $U_\alpha $ is a P-point ultrafilter.

Denote by $\mathbb {P}_E$ the Merimovich Extender-based Prikry forcing with E, as defined in [Reference Merimovich31] (or see Definition 1.5).

Theorem 4.2. Let $G\subseteq \mathbb {P}_E$ be a generic. Suppose that $A\in V[G]\setminus V$ is a subset of $\kappa $ . Then $\kappa $ changes its cofinality to $\omega $ in $V[A]$ .

Proof Work in V. Suppose that is a name of a subset of $\kappa $ and some $p\in \mathbb {P}_E$ forces that it is a new subset.

Let us use $\kappa ^+$ -properness of the forcing $\mathbb {P}_E$ (see [Reference Merimovich31, Claim 2.7] or [Reference Merimovich29, Claim 3.29]). Pick now $N\preceq H_\chi $ , for some $\chi $ large enough such that:

  1. (1) $|N|=\kappa $ ,

  2. (2) $N\supseteq {}^{\kappa>}N$ ,

  3. (3) .

The properness implies that there is $p^*\geq ^* p$ which is ${\langle } N, \mathbb {P}_E{\rangle }$ -generic, i.e.,

In particular, for every $\nu <\kappa $ , the dense open set

is definable from

and $\nu $ , hence in N and it is dense open by elementarity.

Consider $X=\cup _{p\in \mathbb {P}_E\cap N}\operatorname {\mathrm {Supp}}(p)$ , since $\operatorname {\mathrm {Supp}}(p),N$ are of size $\kappa $ , and we have that $|X|\leq \kappa $ . There exists $\alpha ^*<\lambda $ such that for some $f\in V$ , $j_E(f)(\alpha ^*)=(j\restriction X)^{-1}$ (see, for example, [Reference Gitik, Foreman and Kanamori17, Lemma 3.3]).

Denote $Y=X\cup \{\alpha ^*\}$ and fix a set $R\in E_{Y}$ such that if $\mu \in R$ , then $f(\mu (\alpha ^*))=\mu \restriction X$ . Such a set exists since $j_E(f)(j^{-1}(j(\alpha ^*)))=(j\restriction X)^{-1}$ , hence

$$ \begin{align*}(j\restriction Y)^{-1}\in j_E(\{\mu\in ob(Y)\mid f(\mu(\alpha^*))=\mu\restriction X\}).\end{align*} $$

Find a condition $p_*\in G$ such that $Y\subseteq \operatorname {\mathrm {Supp}}(p)$ and $A^{p_*}\restriction Y\subseteq R$ . Define $G\restriction Y=\{p\restriction Y \mid p\in G/p_*\}$ . Then by genericity of $p^*$ and definition of Y, for every $\alpha <\kappa $ there is $p_\alpha \in G\cap D_\nu \cap N$ , hence $\operatorname {\mathrm {Supp}}(p_\alpha )\subseteq Y$ and we can find $p_\alpha \leq p^*_\alpha \in G\restriction Y\cap D_\nu $ . It follows that $A\in V[G\restriction Y]$ . Let $G_{\alpha ^*}=\{p\restriction \{\alpha ^*\}\mid p\in G/p_*\}$ , in particular, $p_0:=p_*\restriction \{\alpha ^*\}\in G_{\alpha ^*}$ . Note that $G_{\alpha ^*}$ is essentially a Prikry generic filter for $\operatorname {\mathrm {Prikry}}(U_{\alpha ^*}).$

Claim 4.3. $V[G\restriction Y]=V[G_{\alpha ^*}]$ .

Proof Inclusion from right to left is clear as $\alpha ^*\in Y$ . For the other direction, let $p_0={\langle } t_0,B_0{\rangle }\leq q={\langle } t, B{\rangle }\in G_{\alpha ^*}$ . For every $|t_0|<i\leq |t| \ t(i)\in B\subseteq B_0$ , by the property of R, we have that $\mu _i:=f(t(i))\smallfrown t(i)\in A^{p^*}$ such that $\mu _i(\alpha ^*)=t(i)$ . Now define $q'={\langle } f,B'{\rangle }$ as follows: $\operatorname {\mathrm {dom}}(f)=Y$ and

$$ \begin{align*}f=f^{p^*}{}^{\smallfrown}\mu_{|t_0|+1}{}^{\smallfrown}...{}^{\smallfrown}\mu_{|t|}.\end{align*} $$

In particular $f(\alpha ^*)=t\geq f^{p_*}(\alpha ^*)$ . Also, let $B'=\{\mu \mid \mu (\alpha ^*)\in B', \ f(\mu (\alpha ^*))=\mu \restriction X\}$ . We claim that $G\restriction Y=\{q'\mid q\in G_{\alpha ^*}/p_0\}$ . Indeed if $p\in G/p_*$ then $q=p\restriction \{\alpha ^*\}\in G_{\alpha ^*}$ and it is straightforward to check that $q'=p\restriction Y$ . It follows that $G\restriction Y$ is definable in $V[G_{\alpha ^*}]$ .

By our assumption $U_{\alpha ^*}$ is a P-point ultrafilter. Now, Theorem 4.1 applies, so

$$ \begin{align*}V[A]\models \operatorname{\mathrm{cof}}(\kappa)=\omega.\\[-42pt]\end{align*} $$

4.2 The original version

The difference here from the forcing of the previous section is that the order $\leq ^*$ is not $\kappa ^+$ -closed. However, we will show that the forcing is still $\kappa ^+$ -proper.

Assume for simplicity that E is a $(\kappa , \kappa ^{++})$ -extender and the function $\nu \mapsto \nu ^{++}$ represents $\kappa ^{++}$ in the ultrapower.

Let ${\mathcal P}_E$ be the forcing of [Reference Gitik and Magidor21] with E.

Lemma 4.4. Assume $p\in {\mathcal P}_E$ . Let $N\preceq H_\chi $ , for some $\chi $ large enough such that:

  1. (1) $|N|=\kappa $ ,

  2. (2) $N\supseteq {}^{\kappa>}N$ ,

  3. (3) $E, {\mathcal P}_E, p \in N$ .

Then there is $p^*\geq p$ which is ${\langle } N, {\mathcal P}_E{\rangle }$ -generic.

Proof Let ${\langle } D_\nu \mid \nu <\kappa {\rangle }$ be an enumeration of all dense open subsets of ${\mathcal P}_E$ which are in N. Proceed by induction and define a $\leq ^*$ -increasing sequence ${\langle } p_\nu \mid \nu <\kappa {\rangle }$ of extensions of p such that, for every $\nu <\kappa $ :

  1. (a) $p_\nu \in N$ .

  2. (b) $\min (A_\nu ^0)>\nu $ , where $A_\nu ^0=\{\rho ^0\mid \rho \in A_\nu \}$ is the projection of $A_\nu $ to the normal measure.

  3. (c) There is $k<\omega $ such that for every ${\langle } \rho _1,..., \rho _k{\rangle } \in [A_\nu ]^k$ , $p_\nu {}^\frown {\langle } \rho _1,..., \rho _k{\rangle } \in D_\nu $ .

It is natural now to move now to a coordinate $\eta $ which is above everything in N and to take the diagonal intersection $\Delta ^*$ of the pre-images of $A_\nu $ ’s according to the normal measure. However, in order to have the property (c) above, something more is needed. Namely, we would like to have the following:

  1. (d) for every ${\langle } \xi _1, ..., \xi _m{\rangle } \in [\min (A_\nu ^0)]^{<\omega }$ , if $p_\nu {}^\frown {\langle } \xi _1, ..., \xi _m{\rangle }\in {\mathcal P}_E$ then there is $k<\omega $ such that:

    $$ \begin{align*}\text{for every }{\langle} \rho_1,..., \rho_k{\rangle} \in [A_\nu]^k, \ p_\nu{}^\frown {\langle} \xi_1, ..., \xi_m{\rangle}^\frown {\langle} \rho_1,..., \rho_k{\rangle} \in D_\nu.\end{align*} $$

Given (d), as we will see, the idea above works fine. Let us construct a sequence which satisfies the conditions (a)–(d).

Pick $p_0\in N$ such that $p_0\geq ^* p$ and (d) is satisfied. To define $p_1$ , use the strong Prikry property to pick a condition $p_1^{\prime } \in N$ , $p_1^{\prime }\geq ^*p_0$ and

$$ \begin{align*}\text{ there is } k<\omega \text{ such that for every } {\langle} \rho_1,..., \rho_k{\rangle} \in [A_1^{\prime}]^k, p_1^{\prime}{}^\frown {\langle} \rho_1,..., \rho_k{\rangle} \in D_1.\end{align*} $$

Let $\eta _0=\min ((A_1^{\prime })^0)$ , by definition of $\pi _{\alpha ,\kappa }$ it follows that $\eta _0$ is an inaccessible cardinal.

Let ${\langle } \vec {\xi }_i \mid i<\eta _0{\rangle } $ be an enumeration of $[\eta _0]^{<\omega }$ .

Define $\leq ^*$ -increasing sequence ${\langle } q_i\mid i<\eta _0{\rangle }$ .

Consider $p_1^{\prime }{}^\frown \vec {\xi }_0$ . If it does not extend $p_0$ , then set $q_0=p_1^{\prime }$ . Otherwise, pick (inside N) $r_0\geq ^* p_1^{\prime }{}^\frown \vec {\xi }_0$ such that

$$ \begin{align*}\text{ there is } k<\omega \text{ such that for every } {\langle} \rho_1,..., \rho_k{\rangle} \in [A(r_0)]^k, r_0{}^\frown {\langle} \rho_1,..., \rho_k{\rangle} \in D_1.\end{align*} $$

Let $q_0={\langle } f^{q_0},A^{q_0}{\rangle }$ be obtained from $r_0$ by removing $\vec {\xi }_0$ from all coordinates which appear in $p_1^{\prime }$ (and leaving at new ones), and then, adding a larger maximal coordinate. Namely, $\operatorname {\mathrm {dom}}(f^{q_0})=\operatorname {\mathrm {dom}}(f^{r_0})\cup \{\alpha _0\}$ where $\alpha _0$ is $\leq _E$ strictly above all the ordinals in $\operatorname {\mathrm {dom}}(f^{r_0})$ . Let t be such that $\pi _{\alpha ,\kappa }"t=f^{p^{\prime }_1}(\kappa )$ and for every $\gamma \in \operatorname {\mathrm {dom}}(f^{q_0}),$

$$ \begin{align*}f^{q_0}(\gamma)=\begin{cases}f^{p^{\prime}_1}(\gamma), & \gamma\in Supp(p^{\prime}_1),\\ f^{r_0}(\gamma), &\gamma\in Supp(r_0)\setminus Supp(p^{\prime}_1),\\ t, & \gamma=\alpha_0.\end{cases}\end{align*} $$

Let $A^{q_0}=\pi _{\alpha _0,mc(r_0)}^{-1}[A^{r_0}]$ . Then $q_0\in N$ and also $q_0\in {\mathcal P}_E$ . By shrinking $A^{q_0}$ a bit more (as in [Reference Gitik, Foreman and Kanamori17, Lemma 3.10]) we secure condition (6), and $p_1^{\prime }\leq ^* q_0$ .

Define $q_1$ in the exact same fashion only replacing $p_1^{\prime }$ by $q_0$ and $\vec {\xi }_0$ by $\vec {\xi }_1$ .

Continue similarly for every $i<\eta _0$ , and finally, let $q_{\eta _0}$ be a $\leq ^*$ -extension of all $q_i$ ’s.

If $\eta _0=\min ((A(q_{\eta _0}))^0)$ , then set $p_1=q_{\eta _0}$ . Otherwise, let $\eta _1=\min ((A(q_{\eta _0}))^0)$ . Repeat the process above with $\eta _1$ replacing $\eta _0$ and $q_{\eta _0}$ replacing $p_1^{\prime }$ . Continuing in a similar fashion, we hope to reach some $\eta $ which is a fixed point, i.e., $\eta =\min ((A(q_{\eta }))^0)$ . However, we need to do this a bit more carefully at limit stages. Let us pick an elementary substructure $N'\prec V_{\mu }$ for sufficiently large $\mu $ of cardinality $\kappa ^+$ , closed under $\kappa $ -sequences, including $p_1^{\prime },p_0,{\mathcal P}_E,E,...$ . We can find some $\alpha <\kappa ^{++}$ such that for every $p\in N'\cap {\mathcal P}_E$ and every $\gamma \in \operatorname {\mathrm {Supp}}(p)$ , $\gamma <_E\alpha $ . Define a sequence of condition ${\langle } q_{\eta _i}\mid i<\eta {\rangle }$ of conditions of $N'$ .

We start with $q_{\eta _0}$ which is already defined. Let $Y_0\in U_\alpha $ such that the commutativity requirement from Definition 1.6(6) holds with respect to $\operatorname {\mathrm {Supp}}(q_{\eta _0})$ . If $\eta _0=\min (Y_0^0)$ we are done. Otherwise, let $\eta _1=\min (Y_0^0)$ and construct $q_{\eta _1}$ in a similar fashion going over all possible $\vec {\xi }\in [\eta _1]^{<\omega }$ , and construct $Y_1\in U_\alpha $ to satisfy (6) with respect to $\operatorname {\mathrm {Supp}}(q_{\eta _1})$ . At a general successor step, we are given $\eta _i, q_{\eta _i}$ , and $Y_i$ . Check if $\eta _i=\min (Y_i^0)$ , if yes, stop the construction, set $p_1=q_{\eta _i}$ , and we are done. Otherwise, let $\eta _{i+1}=\min (Y^0_{i})$ , construct $q_{\eta _{i+1}}$ above $q_{\eta _i}$ as we did with $q_{\eta _0}$ , going over all possible $\vec {\xi }\in [\eta _{i+1}]^{<\omega }$ , then find $Y_{i+1}\in U_\alpha $ satisfying (6) with respect to $\operatorname {\mathrm {Supp}}(q_{\eta _{i+1}})$ . At limit stages $\delta $ take $\eta _\delta =\sup _{i<\delta }\eta _i$ , check if $\eta _\delta =\min ((\cap _{i<\delta }Y_i)^0)$ , if yes, stop the construction and consider the condition $p_1=q_{\eta _\delta }$ with maximal coordinate $\alpha $ , putting $\cap _{i<\delta }Y_i$ as his measure one set. Then $q_{\eta _\delta }$ will be as desired. Otherwise, we find any $q_{\eta _\delta }\in N'$ above all the previous $q_{\eta _i}$ , and construct $Y_{\delta }\in U_\alpha $ with respect to $\operatorname {\mathrm {Supp}}(q_{\eta _\delta })$ . We can further require that $\pi _{\alpha ,mc(q_{\eta _i})}^{\prime \prime } Y_i\subseteq A(q_{\eta _i})$ and that $\min (A(q_{\eta _i})^0)>i$ .

Assume toward a contradiction that no suitable $q_{\eta _\delta }$ was found and that the process goes all the way up to $\kappa $ . Consider $Y^*=\Delta _{i<\kappa }^*Y_i\in U_\alpha $ and let $\mu $ be any limit point of $Y^*$ . Consider step $\mu ^0$ of the construction, and we have $\eta _{\mu ^0}=\sup _{i<\mu ^0}\eta _i$ . For every $i<\mu ^0$ , we have that $\mu \in Y_i$ , hence $\mu \in \cap _{i<\mu ^0}Y_i$ and $\mu ^0\in (\cap _{i<\mu ^0}Y_i)^0$ , and it follows that $\eta _{\mu ^0}\geq \mu ^0\geq \min ((\cap _{i<\mu ^0}Y_i)^0)\geq \eta _{\mu ^0}$ . This means that $\eta _{\mu ^0}=\mu ^0=\min ((\cap _{i<\mu ^0}Y_i)^0)$ which indicates that the construction should have terminated at step $\mu _0$ , contradiction.

We conclude that $p_1$ is defined. The further construction of $p_\nu $ ’s is similar, exploiting the $\kappa $ -closure of $\leq ^*$ .

Pick now some $\alpha \geq _E \beta ,$ for every $\beta \in N\cap \operatorname {\mathrm {dom}}(E)$ which exists since $|N|=\kappa $ . Set

$$ \begin{align*}A=\Delta^*_{\nu<\kappa}\tilde{A}(p_\nu)=\{\rho<\kappa\mid \forall \nu<\rho^0 (\rho\in \tilde{A}(p(\nu)))\},\end{align*} $$

where $\tilde {A}(p_\nu )$ is the pre-image of ${A}(p_\nu )$ under the projection from $\alpha $ to $mc(p_\nu )$ . Define a condition $p^*={\langle } f^*,A^*{\rangle }$ from the sequence ${\langle } p_\nu \mid \nu <\kappa {\rangle }$ as follows: $\operatorname {\mathrm {Supp}}(p^*)=\cup _{\nu <\kappa }\operatorname {\mathrm {Supp}}(p_\nu )\cup \{\alpha \}$ , from the way we defined $p_\nu $ there is no problem defining $f^*=\cup _{\nu <\kappa }f^{p_\nu }\cup \{{\langle }\alpha ,t{\rangle }\}$ where t is any sequence such that $\pi _{\alpha ,\kappa }^{\prime \prime }t=f^*(\kappa )$ . Then we take $A^*=A$ . It follows that $p^*\in {\mathcal P}_E$ , and it has the property that for every $\nu <\kappa $ and any sequence

$$ \begin{align*}\xi_1<\ldots,\xi_k<\min(A^0_\nu)\leq\xi_{k+1}<\ldots<\xi_n\end{align*} $$

of ordinals from A, $p_\nu ^{\smallfrown }{\langle }\xi _1,...,\xi _n{\rangle }\leq p^*{}^{\smallfrown }{\langle } \xi _1,...,\xi _n{\rangle }$ .Footnote 8 Let us argue that it is ${\langle } N,{\mathcal P}_E{\rangle }$ -generic. Let G be generic with $p^*\in G$ . We need to prove that $G\cap N\cap D_\nu \neq \emptyset $ for every $\nu <\kappa $ . By density, pick any $p^{\smallfrown }{\langle }\xi _1,...,\xi _{k_1}{\rangle }\leq ^*q\in D_\nu \cap G$ , and let m be such that $\xi _1,...,\xi _m<\min (A(p_\nu ))\leq \xi _{m+1}<\cdots <\xi _{k_1}$ . By condition (d), there is $k_2$ such that any ${\langle }\nu _1,...,\nu _{k_2}{\rangle }\in [A_\nu ]^{k_2}$ extension $p_\nu ^{\smallfrown }{\langle }\xi _1,...,\xi _m{\rangle }^{\smallfrown }{\langle } \nu _1,...,\nu _{k_2}{\rangle }\in D_\nu $ . If necessary, extend q to

$$ \begin{align*}q{}^{\smallfrown}{\langle} \xi_{k_1+1},...,\xi_{k_1+k_2}{\rangle}\in G\cap D_\nu,\end{align*} $$

and suppose without loss of generality that $k_1\geq m+k_2$ . Since $\nu <\min (A(p_\nu )^0)\leq \xi _{m+1}$ , by definition of $\pi _{\alpha ,\kappa }$ , it follows that $\nu <\xi _{m+1}^0$ , and by diagonal intersection, $\xi _{m+1},...,\xi _{k_1}\in A_\nu $ . It follows that

$$ \begin{align*}p_{\nu}^{\smallfrown}{\langle}\xi_1,...,\xi_m{}{\rangle}^{\smallfrown}{\langle}\xi_{m+1},...,\xi_{m+k}{\rangle}\in D_\nu.\end{align*} $$

Also, $p_{\nu }^{\smallfrown }{\langle }\xi _1,...,\xi _m{}{\rangle }^{\smallfrown }{\langle }\xi _{m+1},...,\xi _{m+k}{\rangle }\leq q$ hence in G. Hence

$$ \begin{align*}p_{\nu}^{\smallfrown}{\langle}\xi_1,...,\xi_m{\rangle}{}^{\smallfrown}{\langle}\xi_{m+1},...,\xi_{m+k}{\rangle}\in G\cap D_\nu\cap N\end{align*} $$

as wanted.

Now, as in the previous section, the following holds.

Theorem 4.5. Let $G\subseteq {\mathcal P}_E$ be a generic. Suppose that $A\in V[G]\setminus V$ is a subset of $\kappa $ . Then $\kappa $ changes its cofinality to $\omega $ in $V[A]$ .

4.3 The Merimovich version

The previous subsection implies in particular that ${\mathcal P}_E$ and $\mathbb {P}_E$ with P-points cannot add $\kappa ^{++}$ -many mutually generic Cohen functions. In this subsection, we will provide the general argument that the Extender-based Prikry forcing $\mathbb {P}_E$ cannot add $\kappa ^{++}$ -many distinct subsets of $\kappa $ which preserves even the regularity of $\kappa $ .

Theorem 4.6. Assume GCHFootnote 9 and let E be an extender over $\kappa $ . Let G be a generic subset of $\mathbb {P}_E$ and let ${\langle } {A}_\alpha \mid \alpha <\kappa ^{++} {\rangle }$ be different subsets of $\kappa $ in $V[G]$ . Then there is $I\subseteq \kappa ^{++}, I\in V, |I|=\kappa $ such that $\kappa $ is a singular cardinal of cofinality $\omega $ in $V[{\langle } {A}_\alpha \mid \alpha \in I{\rangle }]$ . In particular, there is no intermediate model of $V[G]$ where $\kappa $ is measurable and $2^{\kappa }>\kappa ^+$ .

Proof Let be $\mathbb {P}_E$ -names of subsets of $\kappa $ . We will confuse them sometimes with their characteristic functions. Work in V, and for every $\alpha <\kappa ^{++}$ , let $N_\alpha $ be an elementary submodel of $H_\theta $ of cardinality $\kappa $ such that ${}^{\kappa>}N_\alpha \subseteq N_\alpha $ , .

Let $f_\alpha \in \mathbb {P}^*_E$ be $N_\alpha $ -completely generic, i.e., $f_\alpha ^\frown {\langle } \vec {\nu }_1,...,\vec {\nu }_n{\rangle }\in \mathbb {P}^*_E$ is $N_\alpha $ -generic.

Using $\Delta $ -system-like arguments, we can assume that ${\langle } f_\alpha \mid \alpha <\kappa ^{++}{\rangle }$ form a $\Delta $ -system such that for every $\alpha , \beta <\kappa ^{++}$ ,

  1. (1) $otp(\operatorname {\mathrm {dom}}(f_\alpha ))=otp(\operatorname {\mathrm {dom}}(f_\beta ))$ , and the order isomorphism between $\operatorname {\mathrm {dom}}(f_\alpha )$ and $\operatorname {\mathrm {dom}}(f_\beta )$ , $\sigma _{\alpha ,\beta }$ is constant on the intersection $\operatorname {\mathrm {dom}}(f_{\alpha })\cap \operatorname {\mathrm {dom}}(f_{\beta })$ .

  2. (2) for every $\rho \in \operatorname {\mathrm {dom}}(f_\alpha )$ , $f_\alpha (\rho )=f_\beta (\sigma _{\alpha \beta }(\rho ))$ .

Attach to each $\alpha <\kappa ^+$ an $E(\operatorname {\mathrm {dom}}(f_\alpha ))$ -large tree $T_\alpha $ . Define $T_\alpha $ level by level as follows. Set $Lev_1(T_\alpha )=S_\alpha ^0\cup S_\alpha ^1$ , where:

  1. (1) for every $\vec {\nu }\in S_\alpha ^0$ , $\operatorname {\mathrm {dom}}(\vec {\nu })$ contains elements in $\operatorname {\mathrm {dom}}(f_\alpha )\setminus \operatorname {\mathrm {dom}}(f_0)$ , if $\alpha>0$ ,

  2. (2) if $\alpha =0$ , then $S_\alpha ^0= S_\alpha ^1$ ,

  3. (3) $S_\alpha ^1\hspace{-0.5pt}=\hspace{-0.5pt} \{\vec {\nu }\hspace{-0.5pt}\mid\hspace{-0.5pt} \vec {\nu } \text { is an increasing partial function from} \operatorname {\mathrm {dom}}(f_0)\hspace{-0.5pt}\cap\hspace{-0.5pt} \operatorname {\mathrm {dom}}(f_\alpha ) \text { to } \kappa \}$ , if $\alpha>0$ ,

  4. (4) for every $ \vec {\nu } \in S_\alpha ^0$ , the following holds:

    ${\langle } f_\alpha {}^\frown \vec {\nu },B_{\vec {\nu }}{\rangle }$ decides for some $E(\operatorname {\mathrm {dom}}(f_\alpha ))$ -tree $B_{\vec {\nu }}$ and such that the decision depends only on $\vec {\nu }(\kappa )$ .

In order to find such a tree, we will use the fact that $f_\alpha \in \mathbb {P}_E^*$ is $N_\alpha $ -generic, and the set

being dense open in $\mathbb {P}_E^*$ . This implies the existence of an $E(\operatorname {\mathrm {dom}}(f_\alpha ))$ -tree $B_{\vec {\nu }}$ such that

Next, in order to make the decision to depend only on $\vec {\nu }(\kappa )$ , we use ineffability: Suppose that ${\langle } f_\alpha {}^\frown \vec {\nu }, B_{\vec {\nu }}{\rangle }$ forces that

. Let g be the function $g(\vec {\nu })=A_\alpha (\vec {\nu })$ . It follows that

$$ \begin{align*}X_\alpha({\langle}{\rangle}):=j(g)( (j\restriction\operatorname{\mathrm{dom}}(f_\alpha))^{-1} )\subseteq \kappa.\end{align*} $$

Also, since $crit(j)=\kappa $ , it follows that $j(X_\alpha ({\langle }{\rangle }))\cap \kappa =X_\alpha ({\langle }{\rangle })$ . Combine this together with the fact that

$$ \begin{align*}j"\operatorname{\mathrm{dom}}(f_\alpha)\text{ contains elements not in }j(\operatorname{\mathrm{dom}}(f_0))\end{align*} $$

to find an $E(\operatorname {\mathrm {dom}}(f_\alpha ))$ -large set $S^0_\alpha $ , such that $(1)$ holds and for all $\vec {\nu }\in S^0_\alpha $ ,

$$ \begin{align*}A_\alpha(\vec{\nu})=X_\alpha({\langle}{\rangle})\cap \vec{\nu}(\kappa).\end{align*} $$

Finally, we let $Lev_1(T_{\alpha })=S^0_\alpha \cup S^1_\alpha $ . Note that if $\alpha>0$ , then $S^0_\alpha $ and $S^1_\alpha $ are disjoint and therefore $S_\alpha ^1\not \in E(\operatorname {\mathrm {dom}}(f_\alpha ))$ . In general, we define by induction on n, then $n^{\text {th}}$ level of $T_\alpha $ . So let ${\langle } \vec {\rho }_1,...,\vec {\rho }_n{\rangle } \in Lev_n(T_\alpha )$ and let us define $\operatorname {\mathrm {Succ}}_{T_\alpha }({\langle }\vec {\rho }_1,...,\vec {\rho }_n{\rangle })=S_{\alpha ,{\langle } \vec {\rho }_1,...,\vec {\rho }_n{\rangle }}^0\cup S_{\alpha , {\langle } \vec {\rho }_1,...,\vec {\rho }_n{\rangle }}^1$ , where:

  1. (1) For every $\vec {\nu }\in S_{\alpha ,{\langle } \vec {\rho }_1,...,\vec {\rho }_n{\rangle }}^0\cup S_{\alpha ,{\langle } \vec {\rho }_1,...,\vec {\rho }_n{\rangle }}^1$ , $\vec {\nu }(\kappa )> \sup (\operatorname {\mathrm {rng}}({\rho }_n))$ .

  2. (2) $S_{\alpha ,{\langle } \vec {\rho }_1,...,\vec {\rho }_n{\rangle }}^0\subseteq Suc_{B_{{\langle } \vec {\rho }_1,...,\vec {\rho }_n{\rangle }}}({\langle } \vec {\rho }_1,...,\vec {\rho }_n{\rangle })$ .

  3. (3) If $\alpha>0$ , then for every $\vec {\nu }\in S_{\alpha ,{\langle } \vec {\rho }_1,...,\vec {\rho }_n{\rangle }}^0$ , $\operatorname {\mathrm {dom}}(\vec {\nu })$ contains elements in $\operatorname {\mathrm {dom}}(f_\alpha )\setminus \operatorname {\mathrm {dom}}(f_0)$ .

  4. (4) If $\alpha =0$ , then $S_{\alpha ,{\langle } \vec {\rho }_1,...,\vec {\rho }_n{\rangle }}^0= S_{\alpha ,{\langle } \vec {\rho }_1,...,\vec {\rho }_n{\rangle }}^1$ .

  5. (5) If $\vec {\rho }_n\in S_{\alpha ,{\langle } \vec {\rho }_1,...,\vec {\rho }_{n-1}{\rangle }}^0$ and $\alpha>0$ , then $S_{\alpha ,{\langle } \vec {\rho }_1,...,\vec {\rho }_n{\rangle }}^1=\emptyset $ .

  6. (6) If $\vec {\rho }_n\in S_{\alpha ,{\langle } \vec {\rho }_1,...,\vec {\rho }_{n-1}{\rangle }}^1$ and $\alpha>0$ , then

    $$ \begin{align*}S_{\alpha,{\langle} \vec{\rho}_1,...,\vec{\rho}_n{\rangle}}^1 =\{\vec{\nu}\mid \vec{\nu} \text{ is an increasing partial function from }\end{align*} $$
    $$ \begin{align*}\operatorname{\mathrm{dom}}(f_0)\cap \operatorname{\mathrm{dom}}(f_\alpha) \text{ to } \kappa, \vec{\nu}(\kappa)>\sup(\operatorname{\mathrm{rng}}(\vec{\rho}_n))\}.\end{align*} $$
  7. (7) For every $ \vec {\nu } \in S_{\alpha ,{\langle } \vec {\rho }_1,...,\vec {\rho }_n{\rangle }}^0$ , the following holds:

    ${\langle } f_\alpha {}^\frown {\langle } \vec {\rho }_1,...,\vec {\rho }_n{\rangle }^\frown {\langle } \vec {\nu }{\rangle }, B_{{\langle } \vec {\rho }_1,...,\vec {\rho }_n{\rangle },\vec {\nu }}{\rangle }$ decides and the decision depends only on ${\langle } \vec {\rho }_1,...,\vec {\rho }_n{\rangle }{}^\frown \vec {\nu }(\kappa )$ ,

    for some $E(\operatorname {\mathrm {dom}}(f_\alpha ))$ -tree $ B_{{\langle } \vec {\rho }_1,...,\vec {\rho }_n{\rangle }, \vec {\nu }}$ , which is a subtree of $B_{{\langle } \vec {\rho }_1,...,\vec {\rho }_n{\rangle }}$ .

Denote by $T_\alpha ^0$ the tree $T_\alpha $ with $S^1_{\alpha \vec {\nu }_1,...,\vec {\nu }_n}$ removed from $\operatorname {\mathrm {Succ}}_{T_\alpha }({\langle }\vec {\nu }_1,...,\vec {\nu }_n{\rangle })$ Footnote 10 . Clearly, $T_\alpha ^0$ is still $E(\operatorname {\mathrm {dom}}(f_\alpha ))$ -tree.

The tree $T^0_{\alpha }$ has the property that for every ${\langle }\vec {\nu }_1,...,\vec {\nu }_n{\rangle }\in T_{\alpha }$ (!), and every $\vec {\nu }\in \operatorname {\mathrm {Succ}}_{T^0_\alpha }({\langle }\vec {\nu }_1,...,\vec {\nu }_n{\rangle })$ , item $(2)$ above ensures that $(T^0_\alpha )_{{\langle }\vec {\nu }_1,...,\vec {\nu }_n,\vec {\nu }{\rangle }}\subseteq B_{{\langle }\vec {\nu }_1,...,\vec {\nu }_n,\vec {\nu }{\rangle }}$ and by item $(7)$ we obtain

By shrinking if necessary, we can assume that the trees are isomorphic under the obvious isomorphism induced by the $\Delta $ -system. Moreover, by $GCH$ , there are only $\kappa ^+$ -many possible decisions on a fixed isomorphism-type of trees, and therefore we can stabilize the decisions, so they do not depend on a particular choice of $\alpha $ . Let us now take $\kappa $ elements and combine them into a single condition. Namely, we consider ${\langle }{\langle } f_\alpha , T_\alpha {\rangle } \mid 0<\alpha <\kappa {\rangle }$ and define a condition ${\langle } f^*,T^*{\rangle }$ as follows:

Let $f^*=\bigcup _{0<\alpha <\kappa }f_\alpha $ . Define an $E(\operatorname {\mathrm {dom}}(f^*))$ -tree $ T^*$ . It will be a sort of a diagonal intersection of $T_\alpha , 0<\alpha <\kappa $ . Set

$$ \begin{align*}X=\{\vec{\nu}\mid \vec{\nu} \text{ is an increasing partial function from } \operatorname{\mathrm{dom}}(f^*) \text{ to } \kappa,\end{align*} $$
$$ \begin{align*}\operatorname{\mathrm{dom}}(\vec{\nu})\subseteq \bigcup_{\xi<\vec{\nu}(\kappa)}\operatorname{\mathrm{dom}}(f_\xi), (\forall \xi<\vec{\nu}(\kappa)) |\operatorname{\mathrm{dom}}(\vec{\nu})\cap \operatorname{\mathrm{dom}}(f_\xi)|=\vec{\nu}(\kappa)\}.\end{align*} $$

To see that $X\in E(\operatorname {\mathrm {dom}}(f^*))$ , note that

$$ \begin{align*}\operatorname{\mathrm{dom}}( (j\restriction \operatorname{\mathrm{dom}}(f^*))^{-1})=j"\operatorname{\mathrm{dom}}(f^*)\subseteq \cup_{\xi<\kappa}\operatorname{\mathrm{dom}}(j(f_\xi)).\end{align*} $$

Also, for every $\xi <\kappa $ , $|j"\operatorname {\mathrm {dom}}(f^*)\cap \operatorname {\mathrm {dom}}(j(f_\xi ))|=|j"\operatorname {\mathrm {dom}}(f_\xi )|=|\operatorname {\mathrm {dom}}(f_\xi )|$ and since $f_\xi $ is completely generic we conclude that this cardinality must be $\kappa $ . Hence $(j\restriction \operatorname {\mathrm {dom}}(f^*))^{-1}\in j(X)$ . Define the first level of the treeFootnote 11

$$ \begin{align*}Lev_1(T^*)=\operatorname{\mathrm{Succ}}_{T^*}({\langle}{\rangle}):=X \cap \Delta^*_{\xi<\kappa}\pi^{-1}_{\operatorname{\mathrm{dom}}(f^*) \operatorname{\mathrm{dom}}(f_\xi)} \operatorname{\mathrm{Succ}}_{T^0_{\xi}}({\langle}{\rangle}).\end{align*} $$

Then $Lev_1(T^*)\in E(\operatorname {\mathrm {dom}}(f^*))$ . To see this, it suffices to prove that the $E(\operatorname {\mathrm {dom}}(f^*))$ is closed under the diagonal intersection $\Delta ^*$ , so if ${\langle } X_\alpha \mid \alpha <\kappa {\rangle }\subseteq E(\operatorname {\mathrm {dom}}(f^*))$ , we claim that $(j\restriction \operatorname {\mathrm {dom}}(f^*))^{-1}\in j(\Delta ^*_{\alpha <\kappa }X_\alpha )$ . Indeed, for every $\alpha <\kappa =(j\restriction \operatorname {\mathrm {dom}}(f^*))^{-1}(j(\kappa ))$ , $j(\alpha )=\alpha $ and the $\alpha ^{\text {th}}$ element in the sequence $j({\langle } X_\alpha \mid \alpha <\kappa {\rangle })$ is $j(X_\alpha )$ . Since $X_\alpha $ is assumed to be in $E(\operatorname {\mathrm {dom}}(f^*))$ we conclude that $(j\restriction \operatorname {\mathrm {dom}}(f^*))^{-1}\in j(X_\alpha )$ . By the definition of $\Delta ^*$ , and elementarity of j, we conclude that $(j\restriction \operatorname {\mathrm {dom}}(f^*))^{-1}\in j(\Delta ^*_{\alpha <\kappa }X_\alpha )$ .

We continue to define inductively the level of $T^*$ . Let now ${\langle }\vec {\rho }_1,...,\vec {\rho }_n{\rangle } \in Lev_n(T^*)$ , and define $\operatorname {\mathrm {Succ}}_{T^*}({\langle }\vec {\rho }_1,...,\vec {\rho }_n{\rangle })$ . As above, we consider first the set

$$ \begin{align*}X_{{\langle}\vec{\rho}_1,...,\vec{\rho}_n{\rangle}}=\{\vec{\nu}\mid \vec{\nu} \text{ is an increasing partial function from } \operatorname{\mathrm{dom}}(f^*) \text{ to } \kappa, \vec{\nu}(\kappa)>\sup(\operatorname{\mathrm{rng}}(\vec{\rho}_n)),\end{align*} $$
$$ \begin{align*}\operatorname{\mathrm{dom}}(\vec{\nu})\subseteq \bigcup_{\xi<\vec{\nu}(\kappa)}\operatorname{\mathrm{dom}}(f_\xi), (\forall \xi<\vec{\nu}(\kappa)) |\operatorname{\mathrm{dom}}(\vec{\nu})\cap \operatorname{\mathrm{dom}}(f_\xi)|=\vec{\nu}(\kappa)\}.\end{align*} $$

Clearly, $X_{{\langle }\vec {\rho }_1,...,\vec {\rho }_n{\rangle }}\in E(\operatorname {\mathrm {dom}}(f^*))$ . Let $\operatorname {\mathrm {Succ}}_{T^*}({\langle }\vec {\rho }_1,...,\vec {\rho }_n{\rangle })$ be the set

$$ \begin{align*}X_{{\langle}\vec{\rho}_1,...,\vec{\rho}_n{\rangle}} \cap \Delta^*_{\xi<\kappa}\pi^{-1}_{\operatorname{\mathrm{dom}}(f^*) \operatorname{\mathrm{dom}}(f_\xi)} \operatorname{\mathrm{Succ}}_{T_\xi^0}({\langle}\vec{\rho}_1\restriction \operatorname{\mathrm{dom}}(f_\xi),...,\vec{\rho}_n\restriction \operatorname{\mathrm{dom}}(f_\xi){\rangle}).\end{align*} $$

Once we ensure that for every $\xi <\kappa $ , $\operatorname {\mathrm {Succ}}_{T_\xi ^0}({\langle }\vec {\rho }_1\restriction \operatorname {\mathrm {dom}}(f_\xi ),...,\vec {\rho }_n\restriction \operatorname {\mathrm {dom}}(f_\xi ){\rangle })$ is well defined, then $T^*$ will form an $E(\operatorname {\mathrm {dom}}(f^*))$ -fat tree. Namely, we need to prove that:

Claim 4.7. For every $\xi <\kappa $ , ${\langle }\vec {\rho }_1\restriction \operatorname {\mathrm {dom}}(f_\xi ),...,\vec {\rho }_n\restriction \operatorname {\mathrm {dom}}(f_\xi ){\rangle }\in Lev_n(T_\xi )$ . Moreover, $\xi <\vec {\rho }_1(\kappa )$ iff ${\langle }\vec {\rho }_1\restriction \operatorname {\mathrm {dom}}(f_\xi ),...,\vec {\rho }_n\restriction \operatorname {\mathrm {dom}}(f_\xi ){\rangle }\in Lev_n(T^0_\xi )$ .

Proof of Claim 4.7

For every $\xi <\vec {\rho }_1(\kappa )$ , we have

$$ \begin{align*}\vec{\rho}_1\in \pi^{-1}_{\operatorname{\mathrm{dom}}(f^*)\operatorname{\mathrm{dom}}(f_\xi)}(Lev_1(T^0_\xi))\end{align*} $$

and therefore $\vec {\rho }_1\restriction \operatorname {\mathrm {dom}}(f_\xi )\in Lev_1(T^0_\xi )$ . If $\xi \geq \vec {\rho }_1(\kappa )$ , then since $\vec {\rho }_1\in X$ , the $\Delta $ -system ensures that $\operatorname {\mathrm {dom}}(\vec {\rho }_1)\cap \operatorname {\mathrm {dom}}(f_\xi )=\operatorname {\mathrm {dom}}(\vec {\rho }_1)\cap \operatorname {\mathrm {dom}}(f_0)\subseteq \operatorname {\mathrm {dom}}(f_0)\cap \operatorname {\mathrm {dom}}(f_\xi )$ . It follows from the definition that $\vec {\rho }_1\restriction \operatorname {\mathrm {dom}}(f_\xi )=\vec {\rho }_1\restriction \operatorname {\mathrm {dom}}(f_0)\in S^1_\alpha \subseteq Lev_1(T_\alpha )$ . Suppose that ${\langle }\vec {\rho }_1\restriction \operatorname {\mathrm {dom}}(f_\xi ),...,\vec {\rho }_n\restriction \operatorname {\mathrm {dom}}(f_\xi ){\rangle }\in Lev_n(T_\xi )$ , and let $\vec {\rho }_{n+1}\in Succ_{T^*}({\langle }\vec {\rho }_1,...,\vec {\rho }_n{\rangle })$ . Then for every $\xi <\vec {\rho }_{n+1}(\kappa )$ , $\vec {\rho }_{n+1}\restriction \operatorname {\mathrm {dom}}(f_\xi )\in \operatorname {\mathrm {Succ}}_{T^0_\xi }({\langle }\vec {\rho }_1\restriction \operatorname {\mathrm {dom}}(f_\xi ),...,\vec {\rho }_n\restriction \operatorname {\mathrm {dom}}(f_\xi ){\rangle })$ by the definition of the diagonal intersection. If $\xi \geq \vec {\rho }_{n+1}(\kappa )$ , then, as before, $\vec {\rho }_{n+1}\restriction \operatorname {\mathrm {dom}}(f_\xi )\in S^1_{\xi ,{\langle }\vec {\rho }_1,...,\vec {\rho }_n{\rangle }}$ .

$Lev_1(T^*)$ has the property that for all $\vec {\rho }\in Lev_1(T^*)$ and $\alpha <\vec {\rho }(\kappa )$ ,

$$ \begin{align*}{\langle} f^*{}^\frown\vec{\rho}, (T^*)_{\vec{\rho}}{\rangle}\geq^* {\langle} f_\alpha{}^\frown\vec{\rho} \restriction \operatorname{\mathrm{dom}}(f_\alpha), (T^0_{\alpha})_{\vec{\rho} \restriction \operatorname{\mathrm{dom}}(f_\alpha)}{\rangle}.\end{align*} $$

Hence, by $(*)$ , ${\langle } f^*{}^\frown \vec {\rho }, (T^*)_{\vec {\rho }}{\rangle }$ also forces . In addition, if we have $\alpha ,\beta < \vec {\rho }(\kappa )$ , then , depends only on $(\vec {\rho }\restriction \operatorname {\mathrm {dom}}(f_\alpha ))(\kappa )=\vec {\rho }(\kappa )=(\vec {\rho }\restriction \operatorname {\mathrm {dom}}(f_\beta ))(\kappa )$ , and since the isomorphism $\sigma _{\alpha ,\beta }$ fixes $\kappa $ (as $\kappa \in \operatorname {\mathrm {dom}}(f_\alpha )\cap \operatorname {\mathrm {dom}}(f_\beta )$ ) it follows that are decided to be the same set.

Next consider ${\langle }\vec {\rho }_1,...,\vec {\rho }_n{\rangle }\in T^*$ , by Claim 4.7, and we have that for all $\alpha <\vec {\rho }_n(\kappa )$ ,

$$ \begin{align*}(**) \ \ \ \ \ \ \ {\langle} f^{*}{}^{\smallfrown}{\langle}\vec{\rho}_1,...,\vec{\rho}_n{\rangle},(T^*)_{{\langle}\vec{\rho}_1,...,\vec{\rho}_n{\rangle}}{\rangle}\geq\end{align*} $$
$$ \begin{align*}\geq{\langle} f_\alpha^{\smallfrown}{\langle}\vec{\rho}_1\restriction\operatorname{\mathrm{dom}}(f_\alpha),...,\vec{\rho}_n\restriction\operatorname{\mathrm{dom}}(f_\alpha){\rangle},(T^0_\alpha)_{{\langle}\vec{\rho}_1\restriction\operatorname{\mathrm{dom}}(f_\alpha),...,\vec{\rho}_n\restriction\operatorname{\mathrm{dom}}(f_\alpha){\rangle}}{\rangle}.\end{align*} $$

However, since the decision about depends now on ${\langle }\vec {\rho }_1,...,\vec {\rho }_{n-1}{\rangle }^{\smallfrown }\vec {\rho }_n(\kappa )$ , then if $\alpha $ or $\beta $ are below $\vec {\rho }_{n-1}(\kappa )$ , then $\vec {\rho }_{n-1}\restriction \operatorname {\mathrm {dom}}(f_\alpha )$ (or $\vec {\rho }_i\restriction \operatorname {\mathrm {dom}}(f_\alpha )$ for $i<n$ ) might include in its domain ordinals which are moved under the isomorphism $\sigma _{\alpha ,\beta }$ and therefore we are not guaranteed that the decision about is the same (up to $\vec {\rho }_{1}(\kappa )$ it is still the same decision). However, if both $\alpha ,\beta \in [\vec {\rho }_{n-1}(\kappa ),\vec {\rho }_n(\kappa ))$ , we have the following claim:

Claim 4.8. If $\alpha ,\beta \in [\vec {\rho }_{n-1}(\kappa ),\vec {\rho }_n(\kappa ))$ then ${\langle } f^*{}^{\smallfrown }{\langle }\vec {\rho }_1,...,\vec {\rho }_n{\rangle },(T^*)_{{\langle }\vec {\rho }_1,...,\vec {\rho }_n{\rangle }}{\rangle }$ decides the values of and to be the same.

Proof of Claim 4.8

By definition, for every $1\leq i<n$ , $\vec {\rho }_{i}\in X_{{\langle }\vec {\rho }_{1},...,\vec {\rho }_{i-1}{\rangle }}$ . Since $\alpha ,\beta \geq \vec {\rho }_{n-1}(\kappa )\geq \vec {\rho }_i(\kappa )$ ,

$$ \begin{align*}\vec{\rho}_{i}\restriction\operatorname{\mathrm{dom}}(f_\alpha)=\vec{\rho}_i\restriction\operatorname{\mathrm{dom}}(f_\beta)\text{ and }\operatorname{\mathrm{dom}}(\vec{\rho}_i\restriction \operatorname{\mathrm{dom}}(f_\alpha))\subseteq\operatorname{\mathrm{dom}}(f_\alpha)\cap\operatorname{\mathrm{dom}}(f_0).\end{align*} $$

Since the isomorphism $\sigma _{\alpha ,\beta }$ fixes the kernel of the $\Delta $ -system, we have that the decision of

$$ \begin{align*}{\langle} f_\alpha^{\smallfrown}{\langle} \vec{\rho}_1\restriction \operatorname{\mathrm{dom}}(f_\alpha),...,\vec{\rho}_n\restriction \operatorname{\mathrm{dom}}(f_\alpha){\rangle},(T^0_\alpha)_{{\langle} \vec{\rho}_1\restriction \operatorname{\mathrm{dom}}(f_\alpha),...,\vec{\rho}_n\restriction \operatorname{\mathrm{dom}}(f_\alpha){\rangle}}{\rangle}\end{align*} $$

about and the decision of

$$ \begin{align*}{\langle} f_\beta^{\smallfrown}{\langle} \vec{\rho}_1\restriction \operatorname{\mathrm{dom}}(f_\beta),...,\vec{\rho}_n\restriction \operatorname{\mathrm{dom}}(f_\beta){\rangle},(T^0_\beta)_{{\langle} \vec{\rho}_1\restriction \operatorname{\mathrm{dom}}(f_\beta),...,\vec{\rho}_n\restriction \operatorname{\mathrm{dom}}(f_\beta){\rangle}}{\rangle}\end{align*} $$

about is the same. By $(**)$ , the condition ${\langle } f^*{}^{\smallfrown }{\langle }\vec {\rho }_1,...,\vec {\rho }_n{\rangle },(T^*)_{\vec {\rho }_1,...,\vec {\rho }_n}{\rangle }$ decides the values the same way.

Using density arguments we can assume that such defined condition ${\langle } f^*,T^*{\rangle }$ is in the generic subset G of $\mathbb {P}_E$ . Denote by ${\langle } \kappa _n \mid n<\omega {\rangle }$ the Prikry sequence for the normal measure $E_\kappa $ .

It follows that the sets ${\langle } A_\alpha \mid \alpha <\kappa {\rangle }$ have the following property in $V[G]$ :

$$ \begin{align*}(***)\ \ \forall n<\omega.\forall \alpha,\beta\in[\kappa_{n-1},\kappa_n). A_\alpha\cap \kappa_n=A_\beta\cap\kappa_n.\end{align*} $$

Now, let us turn to the model $M^*=V[{\langle } A_\alpha \mid \alpha <\kappa {\rangle }]$ and prove that $cf^{M^*}(\kappa )=\omega $ . Let us define in $M^*$ an $\omega $ -sequence ${\langle }\zeta _n\mid n<\omega {\rangle }$ as follows:

First, let $\zeta ^{\prime }_0$ be the least such that for some for some $\alpha ,\beta <\kappa $ , $A_\alpha \cap \zeta _0^{\prime }\neq A_\beta \cap \zeta _0^{\prime }$ . There exists such $\zeta _0^{\prime }$ since the sets in the sequence ${\langle } A_\alpha \mid \alpha <\kappa {\rangle }$ are distinct. Let $\zeta ^{\prime \prime }_0$ be the least such that for some $\alpha <\zeta ^{\prime \prime }_0$ , $A_\alpha \cap \zeta _0^{\prime }\neq A_{\zeta _0^{\prime \prime }}\cap \zeta _0^{\prime }$ . Define $\zeta _0=\max (\zeta _0^{\prime },\zeta _0^{\prime \prime })$

Claim 4.9. $\zeta _0\geq \kappa _0$ .

Proof of Claim 4.9

If $\zeta _0^{\prime }\geq \kappa _0$ then we are done. Otherwise, suppose $\zeta _0^{\prime }\leq \kappa _0$ , then by $(***)$ for every $\alpha <\beta <\kappa _0$ , we have $A_\alpha \cap \zeta _0^{\prime }=A_\beta \cap \zeta ^{\prime }_0$ . Hence by the definition of $\zeta ^{\prime \prime }_0$ , we have $\zeta ^{\prime \prime }_0\geq \kappa _0$ and also $\zeta _0\geq \kappa _0$ .

Suppose that $\zeta _n<\kappa $ was defined. Then the sequence ${\langle } A_\alpha \mid \zeta _n<\alpha <\kappa {\rangle }$ consists of $\kappa $ -many distinct subsets of $\kappa $ . Since $\kappa $ is strong limit in $V[G]$ , $2^{\zeta _n}<\kappa $ , hence there must be $\zeta _n<\alpha <\beta <\kappa $ such that $A_\alpha \setminus \zeta _n+1\neq A_\beta \setminus \zeta _n+1$ . Let $\zeta ^{\prime }_{n+1}$ be the minimal such that for some $\zeta _n<\alpha <\beta <\kappa $ , $A_\alpha \cap \zeta ^{\prime }_{n+1}=A_{\beta }\cap \zeta ^{\prime }_{n+1}$ . Finally, let $\zeta _n<\zeta ^{\prime \prime }_{n+1}$ be the minimal such that for some $\alpha <\zeta ^{\prime \prime }_{n+1}$ , $A_\alpha \cap \zeta ^{\prime }_{n+1}\neq A_{\zeta ^{\prime \prime }_{n+1}}\cap \zeta ^{\prime }_{n+1}$ and $\zeta _{n+1}=\max (\zeta ^{\prime }_{n+1},\zeta ^{\prime \prime }_{n+1})$ . To conclude that $cf^{M^*}(\kappa )=\omega $ is suffices to prove the following lemma:

Claim 4.10. For every $n<\omega $ , $\zeta _n\geq \kappa _n$ .

Proof of Claim 4.10

By induction, for $n=0$ this is just the previous claim. Suppose that $\zeta _n\geq \kappa _n$ , and toward a contradiction suppose that $\zeta _{n+1}<\kappa _{n+1}$ . Then by definition, there is $\alpha $ , such that $\kappa _n\leq \zeta _n<\alpha <\zeta ^{\prime \prime }_{n+1}<\kappa _{n+1}$ such that $A_\alpha \cap \zeta ^{\prime }_{n+1}\neq A_{\zeta ^{\prime \prime }_{n+1}}\cap \zeta ^{\prime }_{n+1}$ . However, since $\zeta ^{\prime }_{n+1}<\kappa _{n+1}$ we reached a contradiction to $(***)$ , since we found two indices $\alpha ,\beta \in [\kappa _n,\kappa _{n+1})$ such that $A_\alpha \cap \kappa _{n+1}\neq A_\beta \cap \kappa _{n+1}$ .

The sequence ${\langle } \zeta _n \mid n<\omega {\rangle }$ will be a cofinal sequence in $\kappa $ which belongs to $V[{\langle } A_\alpha \mid \alpha <\kappa {\rangle }]$ .

It turns out that $\mathbb {P}_E$ can add $\kappa ^+$ -many mutually generic over V Cohen functions, for specially chosen extender E.

Theorem 4.11. Assume $GCH$ and suppose that E is a $(\kappa ,\kappa ^{++})$ -extender. Then after the preparation of Theorem 2.10, there exists an extender $E'$ such that $\mathbb {P}_{E'}$ adds $\kappa ^+$ mutually generic over V Cohen functions.

Proof Let $j=j_E:V\rightarrow M$ be the natural ultrapower by the $(\kappa ,\kappa ^{++})-$ extender E, then $j(\kappa )>\kappa ^{++}$ , $crit(j)=\kappa $ , and ${}^{\kappa } M\subseteq M$ . Recall that the preparation forcing in Theorem 2.10 is an Easton support iteration

such that

is trivial unless $\beta $ is inaccessible in which case if $\beta <\kappa $ then

is a ${\mathcal P}_\beta $ -name for $\operatorname {\mathrm {LOTT}}(\operatorname {\mathrm {Cohen}}(\beta ,\beta ^+),\operatorname {\mathrm {Cohen}}(\beta ,\beta ^+)^2)$ . At $\kappa $ ,

is a name for $\operatorname {\mathrm {Cohen}}(\kappa ,\kappa ^+)$ . Let $G_\kappa *g_\kappa $ be V-generic for

. In $V[G_{\kappa }*g_\kappa ]$ we can construct an M-generic filter for

by taking $G_\kappa *g_\kappa $ to be the generic up to $\kappa $ , including $\kappa $ and choosing that the lottery sum forces $\operatorname {\mathrm {Cohen}}(\kappa ,\kappa ^+)$ (this forcing is the same in $V[G_\kappa ]$ and $M[G_\kappa ]$ since $(\kappa ^+)^{M[G_\kappa ]}=\kappa ^+$ and $M[G_\kappa ]$ is closed under $\kappa $ -sequences of $V[G_\kappa ]$ ). Above $\kappa $ we have sufficient closure, from the point of view of $V[G_\kappa *g_\kappa ]$ , and by $GCH$ there are not too many dense open subsets of the tail forcing ${\mathcal P}_{(\kappa ,j(\kappa )]}$ to meet, hence the embedding j lifts to

$$ \begin{align*}j\subseteq j^*:V[G_{\kappa}*g_\kappa]\rightarrow M[j(G_{\kappa})*j(g_\kappa)].\end{align*} $$

Since the cardinals in all the models are preserved, it follows that [Reference Cummings, Foreman and Kanamori12, Proposition 8.4]

$$ \begin{align*}(\kappa^{++})^{M[j(G_{\kappa})*j(g_\kappa)]}=\kappa^{++}<j(\kappa)\text{ and }{}^{\kappa}M[j(G_{\kappa})*j(g_\kappa)]\subseteq M[j(G_{\kappa})*j(g_\kappa)].\end{align*} $$

So in $V[G_\kappa *g_\kappa ]$ the extender E extends to an extender $E'={\langle } E^{\prime }_a\mid a\in [\kappa ]^{<\omega }{\rangle }$ defined by $E^{\prime }_a=\{X\subseteq \kappa ^{|a|}\mid a\in j^*(X)\}$ .

Let W be the non-Galvin, $\kappa $ -complete ultrafilter over $\kappa $ with preparation for adding $\kappa ^+$ -many Cohens (See Theorem 2.11).

Combine $E',W$ together as follows. First take an ultrapower with $E'$ . Let $j_{E'}:V \to M_{E'}$ be the corresponding embedding. Denote $j_{E'}(\kappa )$ by $\kappa _1$ and let $W'=j_{E'}(W)$ . Then take an ultrapower of $M_{E'}$ with $W'$ . Let $j_{W'}:M_{E'} \to M$ be the corresponding embedding.

Consider $j_*=j_{W'}\circ j_{E'}:V\to M$ . Let $E^*$ be the derived $(\kappa ,\lambda )$ -extender for some $\kappa _1<\lambda \leq j_*(\kappa )$ .

Note that $E^*(\kappa _1)=W$ , since for any $X\subseteq \kappa $ ,

$$ \begin{align*}X\in E^*(\kappa_1) \Leftrightarrow \kappa_1 \in j_*(X)\Leftrightarrow\kappa_1\in j_{W'}( j_{E'}(X))\Leftrightarrow j_{E'}(X)\in W'\end{align*} $$
$$ \begin{align*}\Leftrightarrow j_{E'}(X)\in j_{E'}(W)\Leftrightarrow X\in W.\end{align*} $$

The Prikry forcing with W adds $\kappa ^+$ -many Cohens over V. This forcing is a part of $\mathbb {P}_{E^*}$ , since W appears as one of the measures of $E^*$ , which implies the theorem.

4.4 Cohen subsets of $\kappa ^+$

Let us argue here that both versions add $\kappa ^{++}$ -many (or $\lambda $ -many if the extender has $\lambda $ generators for a regular $\lambda>\kappa $ ) Cohen subsets of $\kappa ^+$ mutually generic over V.

Start with ${\mathcal P}_E$ of [Reference Gitik and Magidor21].

Theorem 4.12. Let $G\subseteq {\mathcal P}_E$ be a generic. Then in $V[G]$ there is a sequence ${\langle } Z_\xi \mid \xi <\kappa ^{++}{\rangle }$ of mutually generic over V Cohen subsets of $\kappa ^+$ .

Proof Let ${\langle } t_\alpha \mid \alpha <\kappa ^{++}{\rangle }$ be the Prikry sequences added by G.

Split, in V, $\kappa ^{++} $ into disjoint intervals ${\langle } I_\xi \mid \xi <\kappa ^{++}{\rangle }$ order type of each $\kappa ^+$ . Denote by $\sigma _\xi $ the order isomorphism between $I_\xi $ and $\kappa ^+$ .

Now, in $V[G]$ , set

$$ \begin{align*}Z_\xi=\{\sigma_\xi(\alpha)\in I_\xi \mid t_\alpha(0) \text{ is even }\}.\end{align*} $$

Let us argue that such a sequence is as desired.

Work in V. Let $p\in {\mathcal P}$ and let D be a dense open subset of $\operatorname {\mathrm {Cohen}}(\kappa ^+, \kappa ^{++})$ .

Let us find $q\geq p$ such that

Extend first p to some r such that for every $\gamma \in \operatorname {\mathrm {Supp}}(r), r^\gamma $ is not equal to the empty sequence. Now, using $I_\xi ,\sigma _\xi $ ’s turn ${\langle } r^\gamma (0) \mid \gamma \in \operatorname {\mathrm {Supp}}(r){\rangle }$ into a condition in $\operatorname {\mathrm {Cohen}}(\kappa ^+, \kappa ^{++})$ . Extend it to one in D and move back to ${\mathcal P}$ using $I_\xi ,\sigma _\xi ^{-1}$ ’s. Finally, turn the result into a condition q in ${\mathcal P}$ stronger than r. It will be as desired.

The situation in the case of the Merimovich version is very similar:

Theorem 4.13. Let $G\subseteq \mathbb {P}_E$ be a generic. Then in $V[G]$ there is a sequence ${\langle } Z_\xi \mid \xi <\kappa ^{++}{\rangle }$ of mutually generic over V Cohen subsets of $\kappa ^+$ .

Proof Proceed as in Theorem 4.12 and define ${\langle } Z_\xi \mid \xi <\kappa ^{++}{\rangle }$ .

Work in V. Let $p\in {\mathcal P}$ and let D be a dense open subset of $\operatorname {\mathrm {Cohen}}(\kappa ^+, \kappa ^{++})$ .

Let us find $q\geq p$ such that

A slight difference here is that the support of $p={\langle } f, T{\rangle }$ , i.e., $\operatorname {\mathrm {dom}}(f)$ may have $\kappa $ many places $\gamma $ with $f(\gamma )={\langle }{\rangle }$ .

As a result, for such $\gamma $ , $t_\gamma (0)$ will be determined only after an element of the corresponding set of measure one is picked, and there are $\kappa $ -many such $\gamma $ ’s.

However, we do not need the exact value of $t_\gamma (0)$ , but rather to know whether it is even or odd. This is determined (on a set of measure one) by $\gamma $ itself. Namely, in this situation, $t_\gamma (0)$ will be even iff $\gamma $ is even.

The rest of the argument is as in Theorem 4.12.

Acknowledgements

The authors would like to thank the referee for their insightful remarks and the improvement of the content of the paper. Also, they would like to thank Mohammad Golshani for his crucial correction in the first draft of the paper. Finally, they would like to thank the participants of the Set Theory Seminar of Tel-Aviv University, and in particular to Menachem Magidor, Carmi Merimovich, and Sittinon Jirattikansakul for their wonderful comments and suggestions.

Funding

The work of the first author was supported by the National Science Foundation under Grant No. DMS-2246703. The work of the second author was partially supported by ISF grants (Nos. 1216/18 and 882/22).

Footnotes

1 We would like to thank Mohammad Golshani for reminding us of the exact formulation of Woodin’s question.

2 Recall that $k:M_1\rightarrow M_2$ is the factor map satisfying $j_2=k\circ j_1$ defined by $k([f]_U)=j_2(f)(\kappa )$ .

3 Since over V, at $\kappa $ we forced one copy of Cohen’s, i.e., $\operatorname {\mathrm {Cohen}}(\kappa ,\kappa ^+)$ , over $M_U$ we need to force only one copy of $\operatorname {\mathrm {Cohen}}(\kappa _1,\kappa _1^+)$ , thus we only need the generic $F_{\kappa _1}$ .

4 Explicitly, one can define in $V[G]$ the function $f(\alpha )=f_{\kappa ,\alpha }(\alpha )$ . Then $j^*_2(f)(\kappa _1)=f_{\kappa _2,\kappa _1}(\kappa _1)=\kappa $ .

5 Since the tail forcing $\mathcal {P}_{[\nu _0,\kappa ]}$ is $\nu _0$ -closed, if there is such $d^{\nu _0}\in V[G_{\kappa }*F_\kappa ]$ then $|d^{\nu _0}|<\nu _0$ , hence $d^{\nu _0}\in V[G_{\nu _0}]$ .

6 An easy transfinite induction proves that if an ordinal $\gamma =\beta \cdot 2$ or $\gamma =\beta \cdot 2+1$ , then $\beta $ is unique, and we denote $\beta =\lfloor \frac {\gamma }{2}\rfloor $ .

7 For a set of ordinals A, let $A\cdot 2+1=\{\alpha \cdot 2+1\mid \alpha \in A\}$ .

8 Although $\xi _1,...,\xi _k\notin A_\nu $ , the condition $p_\nu ^{\smallfrown }{\langle }\xi _1,...,\xi _n{\rangle }$ is a legitimate condition which is simply not above $p_\nu $ .

9 $2^{\kappa }=\kappa ^+$ is enough, since $\kappa $ is a measurable, and so $2^\nu =\nu ^+$ on relevant sets.

10 Even if ${\langle }\vec {\nu }_1,...,\vec {\nu }_n{\rangle }\in T_\alpha \setminus T^0_\alpha $ the set $\operatorname {\mathrm {Succ}}_{T^0_\alpha }({\langle }\vec {\nu }_1,...,\vec {\nu }_n{\rangle })$ is still defined.

11 We define the diagonal intersection for the ultrafilter $E(d)$ as follows: for ${\langle } X_\alpha \mid \alpha <\kappa {\rangle }\subseteq E(d)$ , $\Delta ^*_{\alpha <\kappa }X_\alpha =\{\vec {\nu }\in Ob(d)\mid \forall \xi <\vec {\nu }(\kappa ). \vec {\nu }\in X_\xi \}.$

References

Abraham, U. and Shelah, S., Forcing closed unbounded sets, this Journal, vol. 48 (1983), no. 3, pp. 643–657.Google Scholar
Baumgartner, J. E., Ha̧jņal, A., and Mate, A., Weak saturation properties of ideals , Infinite and Finite Sets (Colloq., Keszthely, 1973; Dedicated to P. Erdős on His 60th Birthday) , vol. I , Colloquia Mathematica Societatis János Bolyai, 10, North-Holland, Amsterdam, 1975, pp. 137158.Google Scholar
Benhamou, T., Prikry forcing and tree Prikry forcing of various filters . Archive for Mathematical Logic , vol. 58 (2019), pp. 787817.CrossRefGoogle Scholar
Benhamou, T., Garti, S., and Poveda, A., Negating the Galvin property . Journal of the London Mathematical Society , vol. 108 (2023), pp. 190237.CrossRefGoogle Scholar
Benhamou, T., Garti, S., and Shelah, S., Kurepa trees and the failure of the Galvin property . Proceedings of the American Mathematical Society , vol. 151 (2023), pp. 13011309.CrossRefGoogle Scholar
Benhamou, T. and Gitik, M., Sets in Prikry and Magidor generic extensions . Annals of Pure and Applied Logic , vol. 172 (2021), no. 4, p. 102926.CrossRefGoogle Scholar
Benhamou, T. and Gitik, M., Intermediate models of Magidor–Radin forcing—part II . Annals of Pure and Applied Logic , vol. 173 (2022), p. 103107.CrossRefGoogle Scholar
Benhamou, T. and Gitik, M., Intermediate models of Magidor–Radin foring—part I . Israel Journal of Mathematics , vol. 252 (2022), pp. 4794.CrossRefGoogle Scholar
Benhamou, T., Gitik, M., and Hayut, Y., The variety of projections of a tree-Prikry forcing. Journal of Mathematical Logic , 2021, to appear. Preprint available from arXiv:2109.09069.Google Scholar
Ben-Shalom, Y., On the Woodin construction of failure of GCH at a measurable cardinal, preprint, 2017, arXiv:1706.08143.Google Scholar
Cummings, J., A model in which GCH holds at successors but fails at limits . Transactions of the American Mathematical Society , vol. 329 (1992), no. 1, pp. 139.CrossRefGoogle Scholar
Cummings, J., Iterated forcing and elementary embeddings , Handbook of Set Theory (Foreman, M. and Kanamori, A., editors), Springer, Dordrecht, 2010, pp. 775883.CrossRefGoogle Scholar
Garti, S., Weak diamond and Galvin’s property . Periodica Mathematica Hungarica , vol. 74 (2017), no. 1, pp. 128136.CrossRefGoogle Scholar
Garti, S., Tiltan. Comptes Rendus de l’Académie des Sciences , vol. 356 (2018), no. 4, pp. 351359.Google Scholar
Gitik, M., The negation of the singular cardinal hypothesis from $o\left(\kappa \right)={\kappa}^{++}$ . Annals of Pure and Applied Logic , vol. 43 (1989), pp. 209234.CrossRefGoogle Scholar
Gitik, M., The strength of the failure of the singular cardinal hypothesis . Annals of Pure and Applied Logic , vol. 51 (1991), no. 3, pp. 215240.CrossRefGoogle Scholar
Gitik, M., Prikry-type forcings , Handbook of Set Theory (Foreman, M. and Kanamori, A., editors), Springer, Dordrecht, 2010, pp. 13511447.CrossRefGoogle Scholar
Gitik, M., On density of old sets in Prikry type extensions . Proceedings of the American Mathematical Society , vol. 145 (2017), no. 2, pp. 881887.CrossRefGoogle Scholar
Gitik, M., On $\kappa$ -compact cardinals . Israel Journal of Mathematics , vol. 237 (2020), pp. 457483.CrossRefGoogle Scholar
Gitik, M., Kanovei, V., and Koepke, P., Intermediate models of Prikry generic extensions, 2010. Available at http://www.math.tau.ac.il/gitik/spr-kn.pdf.Google Scholar
Gitik, M. and Magidor, M., Extender based forcings, this Journal, vol. 59 (1994), no. 2, pp. 445–460.Google Scholar
Hamkins, J. D., The lottery preparation . Annals of Pure and Applied Logic , vol. 101 (2000), nos. 2–3, pp. 103146.CrossRefGoogle Scholar
Jech, T., Set Theory , the third millennium edition, revised and expanded, Springer Monographs in Mathematics, Springer, Berlin, 2003.Google Scholar
Kanamori, A., The Higher Infinite , Springer, Berlin, 1994.Google Scholar
Koepke, P., Rasch, K., and Schlicht, P., Minimal Prikry-type forcing for singularizing a measurable cardinal, this Journal, vol. 78 (2013), pp. 85–100.Google Scholar
Magidor, M., On the singular cardinals problem II . Annals of Mathematics , vol. 106 (1977), no. 3, pp. 517547.CrossRefGoogle Scholar
Maharam, D., An algebraic characterization of measure algebras . Annals of Mathematics , vol. 48 (1947), pp. 154167.CrossRefGoogle Scholar
Mathias, A. R. D., On sequences generic in the sense of Prikry . Journal of Australian Mathematical Society , vol. 15 (1973), pp. 409414.CrossRefGoogle Scholar
Merimovich, C., Prikry on extenders, revisited . Israel Journal of Mathematics , vol. 160 (2003), pp. 253280.CrossRefGoogle Scholar
Merimovich, C., Supercompact extender based Prikry forcing . Archive for Mathematical Logic , vol. 50 (2011), pp. 591602.CrossRefGoogle Scholar
Merimovich, C., Mathias like criterion for the extender based Prikry forcing . Annals of Pure and Applied Logic , vol. 172 (2021), p. 102994.CrossRefGoogle Scholar
Mitchell, W. J., The core model for sequences of measures, I . Mathematical Proceedings of the Cambridge Philosophical Society , vol. 95 (1984), no. 2, pp. 229260.CrossRefGoogle Scholar
Mitchell, W. J., On the singular cardinal hypothesis . Transactions of the American Mathematical Society , vol. 329 (1992), no. 2, pp. 507530.CrossRefGoogle Scholar
Sacks, G. E., Forcing with perfect closed sets, axiomatic set theory . Proceedings of Symposia in Pure Mathematics , vol. 13 (1971), no. 1, pp. 331355.CrossRefGoogle Scholar